Sie sind auf Seite 1von 366

2011

Secondary School
Examination
Papers

Secondary One Express


Mathematics
Paper 1 & 2

1 Admiralty Secondary School SA2


2 Manjusri Secondary School SA2
3 Ang Mo Kio Secondary School SA2
4 Ping Yi Secondary School SA2
5 Bukit Batok Secondary School SA2
6 Bukit Merah Secondary School SA2
7 Siglap Secondary School SA2
8 Tanjong Katong Secondary School SA2
9 Beatty Secondary School SA2
10 Clementi Town Secondary School SA2
11 Geylang Methodist Secondary School SA2
12 Jurong West Secondary School SA2
13 Shuqun Secondary School SA2
14 Whitley Secondary School SA2
15 Yuhua Secondary School SA2
16 Northland Secondary School SA2
17 Chung Cheng High School SA2
I
NAME: NO: CLASS:

ADMIRALTY SECONDARY SCHOOL

Admlfalty
1··1arc... ,.. ....."""

,, END-OF-YEAR EXAMINATION 2011 II


SUBJECT Mathematics
PAPER 1
LEVEL/STREAM 1 Express
DATE 11 October 2011
TIME : 0750 - 0850
DURATION :1h
Instructions to candidates:

1. Write your name, class and index number.

2. Answer ALL questions in the spaces provided.

3. Use an electronic calculator to evaluate explicit numerical expressions. If


the degree of accuracy is not specified in the question, and if the answer
is not exact, give the answer to three significant figures.

Give answers in degrees to one decimal place. For 7r, use either your
calculator value or 3 .142, unless the question requires the answer in terms
of :ir.

4. Essential working must be shown. Omission of essential workings and


illegible handwriting will lead to loss of marks.

50
DO NOT TURN OVER THIS PA GE UNTIL YOU ARE TOLD TO DO SO.

This question paper consists of!! printed pages including this cover page.
1. Express the following in index notation, where all the bases are prime numbers.
~a) 81 x 156
(b) 22 x 440

Answer: (a) .... ... . ........... ...... ... [4)

(b) . ... ... ... . ... . ............ [4)

2. Express your answers in index notation.


(a) Find the square of 46 x 193 .
(b) Find the cube root of the result in (a).

Answer: (a) .. .. . .... .. . . . .. . .. . .. .. . . . [1]

(b) ··························· [2]

2
3. Evaluate each of the following.
(a)(- 9-7)+ 13x4
(b) [SO - (28 - 32)) x 6.

Answer: (a) ............ :.............. [2]

(b) ..... ... .. ........ ...... . .. [2]

4. The price of a cake of radius r cm and height h cm is given by the formula I'=__!_ r 2 h.
25
(a) Find the price for a cake of radius 35 cm and height of 11 cm.

(b) If the price of cake is $240 and the height is 15 cm, calculate the radius of the cake.

Answer: (a)$............. ... ...... ... . [2]

(b) ........................cm [3]

3
5. Expand and simplify the following.
(a) 3(2a +4b)+5(b-a),
(b) 6(b-a)+4b,

c+ I c- 2
(c) - +-.
3 2

Answer: (a) . . . . . . . . .. . . . . . . . . . . . . . . . . . [2]

(b) ···· ···· ················ .. . [2]

(c) . . . . . . . . . . . .. . . . . . . . . . . . . . . [2)

6. A pail was half filled with water. An amount of water equivalent to _!_ of the volume of
4

the pail was poured out. Given that the remaining volume of water in the pail is 300 ml,
find the volume of the pail.

Answer: ........... ... ............... ml [2)

4
7. Convert the following speed. Leave your answers in 3 significant figures.
(a) 223 km/h to mis,
(b) 16 m/s to km/h.

Answer: (a) ...................... m/s [2]

(b) .... .... ... ......... . km/h [2]

8. A mobile phone shop sold 21 mobile phones a day. This is 40% below the daily target'.
What is the target number of mobile phones to be sold a day?

Answer: . ........... mobile phones [2]

5
2
9. (a ) The general term of a sequence is T. = n - 3n + 4. Find its 4th term.

(b) Consider the sequence 38, 36, 34, 32 ... .


(i) Find the next term.
(ii) Find its general term.
(iii) Find the sothterm.

Answer: (a) .. . ..... .. ... , . . . . . .. . . . . . . [2]

(bi) . .. . ······ · ····· ........... [1]

(bii) . . . . . . . . . . . .. . . . . . . . . . . . . . . [1]

(biii) .. . . . . . . . . . . . . .. . . . . .. . . . . . [1)

6
10. A club needs to raise $52 000 to build some indoor squash courts. The club needs to
recruit 500 members to raise the funds.
(a) If each member pays an equal amount of membership fee, how much is the
membership fee in order to meet the target?
(b) If each court costs $8600, find the largest possible number of courts that can be
built with $52 000.

Answer: (a)$......... ... ........ . .. .. [2]

(b) ..................courts {2]

7
11. The figure shows a diagram of a building. The top part is a triangle. The bottom part is
a trapezium. These two parts are joined by a rectangle and two semicircles in the
middle. The perpendicular height of the triangle and trapezium is 12 and 19
respectively. Taken:= 3.14.
(a) Calculate the perimeter of the figure.
(b) Calculate
(i) the area of triangle,
(ii) the area of trapezium,
(iii) the total area of the diagram.
- -10
7t.- -
I
I
I
I
I
I

:19
I
I
..!,-
15

Answer: (a) ..................... units [2]


2
(bi) ..................... units [1]

(bii) ....................units2 [1]

(b ...
111
) ••••••••••••.•••••• uni
•ts2 [3]

© End Of Paper © 8
Sec 1 Express End of Year Exam 2011
Mathematics Paper 1 Marking Scheme

1a)
81 x 156 240 = -1 (r)(15) [M1J
25
240 + 15x25=r2 [M1]
81 = 34 [M1]
=
156 z2 x 3 x 13 [M1]
r =
J400
81x156 r=20cm [A11
= 3 4 x z2 x 3 x 13 [M 1]
= 22 x 35 x 13 [A11
5a)
3(2a +4b) + 5(b - a)
1b) =6a + 12b + 5b - 5a [M1]
22 x 440 =a+17b[A1l
=
22 2 x 11 [M1] 5b) - 6(b - a) + 4b
=
440 23 x 5 x 11 [M1]
= - 6b + 6a + 4b [M 1J
22 x 440 = 6a - 2b _ffi1}
= 2 x 11 x 23 x 5 x 11 [M1]
= 4 2
2 x 5 x 11 [A 1] 5c)
c+l c- 2
-- + --·
3 2
2a)
(46 x 193)2 _ 2(c+l) 3(c-2)
- --- + ---
=4 12 x 196 fA1] 2x3 3x 2
2c + 2 + 3c - 6
2b) = [M1]
6
V~
4-1 2 -x-19
_6_ (M1]
=4 4 x 192 [A1]
= ,?_c::..=.:!_ [A 1J
6

3a) 6)
(-9-7)+13x4
I
=( - 16) + 52 (M1] - = 300 ml
= 36 [A1l 4
4
-4 =300 x 4 [M1]
3b) = 1200 ml [A1]
[ 50 - ( 28 - 32 ) ] x 6
=[ 50 - ( - 4)] x 6 [M1]
= 54x 6
7a)
= 324 [A1]
223 km = 223 x 1000 m [M1]
1h 1x60x60s
4a) = 61.9 mis [A1J
I
P= - r2h
25
= __!_ (352)( 11) [M1)
25
=$539 [A1J
4b)
Sec 1 Express End of Year Exam 2011
Mathematics Paper 1 Marking Scheme

7b)
16 m = 16 + 1000 km
1s 1 + 3600 h 11 bi)

Area of triangle = .!. (10)(12)


16 x 3600 [M1] 2
1000 = 60 [A1]
= 57.6 km/h [A 1) 11bii)

Area of trapezium = .!. (10 + 15) 19


2
8)
= 237.5 [A1]
60% 7 21
100% 7 21 + 60 x 100 [M1]
11 biii) ..
= 35 [A1 J
Area ofrectangle = 10 x 12 [M1]
= 120
9a) 2
Area of circle= 3.14 x 6 [M1]
T4 = 42 - 3 (4) + 4 [M1]
= 113.04
=§....l61]
Total area= 60 + 120 + 113.04 + 237.5
= 530.54 [A1]
9bi)
Ts= 30 (81]

9bii)
I,. = 40-2n (81)

9biii)
Tso= 40 - 2 (50)
=
T50 - 60 fA1]

10a)
52 000 + 500 [M1)
= $104 [A1)

10b)
52 000 + 8600 [M1)
=
= 6.04 6 courts [A1J

11a)
Perimeter =
13+13+(3.14x12)+20+20+15 [M1]
=26 + 37.68 + 55
= 118.68 [A1]
I
I
NAME: NO: CLASS:

ADMIRALTY SECONDARY SCHOOL

4
1\
Admiralty
- nri<:L"')'>rl'lol~·

II END-OF-YEAR EXAMINATION 201 1 II


SUBJECT Mathematics
PAPER 2
LEVEL/STREAM 1 Express
DATE 12 October 2011
TIME : 0750 - 0920
DURATION : 1 h 30 min
Instructions to candidates:

1. Write your name, class and index number.

2. Answer ALL questions in the spaces provided.

3. Use an electronic calculator to evaluate explicit numerical expressions. If the


degree of accuracy is not specified in the question, and if the answer is not
exact, give the answer to three significant figures.

Give answers in degrees to one decimal place. For TC, use either your
calcu lator value or 3.142, unless the question requires the answer in terms of
TC.

4. Essential working must be shown. Omission of essential workings and


illegible handwriting will lead to loss of marks.

50

DO NOT TURN OVER THfS PA GE UNTIL YOU ARE TOLD TO DO SO.

This question paper consists of!! printed pages including this cover page.
1. Solve the following equations.
(a) 2(6x+5) = 5(3x - l)

(b) 3(3x - 1) - 6(x - 2x) =-57

Answer: (a) . .. . .. . . .. . .. . .. . ....... [2]

(b) .. .. ... . .. ... ... ........ [3]

2. r-actorise the following completely.


(a) 8qx+ 6px + 2 l py + 28qy

(b) 7xy+z(5x-7y)-5z 2

Answer: (a) . .. . . .. . . . . . . . . . . . . . . . . . [2]

(b). .... ... .. . ... .......... [3J

2
3. Eve bought x number of books at $14.50 each and twice as many books at
$12.50 each. The total cost of all books bought is $1975. Find the value of x.

Answer: [3]

3
4. Ally, Bond, Chad and Day are participants in a talent competition in a
secondary school. The following lists the votes received by the four candidates
from students in the school.

Bond Day Bond Ally Day Bond Bond Day Ally


Day Ally Chad Day Chad Chad Day Ally Bond
Ally Bond Day Chad Bond Day Ally Chad Day
Chad Chad Bond Day Ally Chad Day Bond Chad
Day Ally Day Bond Chad Day Chad Bond Day

(a) Fill in the frequency table below that represents the data given.
(b) What is the total number of students who voted?
( c) Represent the data by a bar graph in the space below.
(d) Who is the most popular participant?
(a) Frequency Table

r~:::ero-f-4~~~~~1--~~~~+-~~~~-+--~~~~~
Ivotes
(c) Show your bar graph in the space below.

Answer: (a) Frequency table [2]


(b) ..................students [1]
(c) Bar graph [3]
(d)................... ........... [1]

4
5. A toy puzzle block looks like the diagram below.

•-«-- .;i.,
I I

2cm
IL'
---i
2cm
I __ }
1 cm

l 4cm

The block is made up of a cuboid and two cylinders as protrusions. All


protrusions have a thickness of 0 .5 cm.
Take rr = 3.14.
(a) Calculate
(i) . the volume of cuboid,
(ii) the volume of 2 cylinders and;
(iii) the total volume of the puzzle block.

(b) Calculate the total surface area of the puzzle block.

3
Answer: (ai) ........................ ... cm [1]
(aii)..........................cm3 [2]
3
(aiii)........... .............. cm [1]
2
(b)........................ ..cm [4]

5
6. (a) Mark the points A( - 3, 6 ) and B( 3, - 4 ) on the Cartesian plane
provided and draw a line to connect them.
(b) What is the gradient of the line?
(c) Write down the coordinates of the point at which the line cuts the y - axis.

-3 -2 -1 0 2 3 x
-1

-2

-3

-4

Answe r: (a) Line AB [2]


(b)...... ... ..................... [2]
(c)....... ....................... [1]

6
7. (a) Complete the following table.
[4]

(b) Draw the graph of y = 2x - 5 for values of x from - 2 to 5 in the grid [2]
provided.

x
-2 -1 0 2 3 4 5
-l

-2

-3

-4

-5

-6

-7

-8

-9

7
8. (a) Construct a quadrilateral ABCD where AB = 9 cm , BC = AD = 8 cm ,
L'.ABC = 120° and L'.DAB =60°.
(b) Measure L'.BCD and L'.CDA.
(c) State the special name for this quadrilateral.
(d) Construct a perpendicular bisector of line AB.
(e) Construct the angle bisector of L'.CDA.

Answer: (a) Quadrilateral ABCD. [4}


(b)................ ' .. ... ..... ..... [2]
(c). ....... ........................... [1)
(d) Perpendic ular bisector [2]
(e) Angle bisector [2]

© End Of Paper ©

8
Sec 1 Express End of Year Exam 2011
Mathematics Paper 2 Marking Scheme

1a) 2(6x+5)=5(3x - I)
12x+ 10 = J5x - 5 [M1)
3x = 15
x = 5_.ffi1.]

1b) 3(3x - 1)-6(x - 2x) = -57


9x-3-6x+ 12x = - 57 [M1]
15x-3 = -57
I 5x = - 54 [M1]
x= -3~ or - 3.6 [A1]
5

2a) 8qx+6px+2 1py+28qy


= 2x(4q + 3p) + 7y(3p + 4q) [M1]
= (2x+1y)(3p+4q).J8.1.]

2b) 7xy+z(5x-1y) - 5z 2 or 1.xy+z(5x - 7y) - 5z 1


= 7xy + 5zx - 7zy - 5z 2 [M1] = 7.xy + 5zx - 1 zy - 5z [M1]
2

= 7y(x - z) + Sz(x - z ) [M1] = x(7y+5z ) -z(7y+5z ) [M1]


=.(x - z)(7 y + 5z).J8.1] =(x - z)(7y + 5z).JA1]

3) (x x l4-50)+(2xxl2.50) = $1975 [M1J


39.50x = $1975 [M1]
x = 50.JA1]

4a
Bond Chad Day
----
11 11 15

4b) 45 students [B1J

4c) correct x-axis [B1}, correct y-axis [B1l. all correct bars [B1]

Ally Bond Chad Day

4d) Day [B1]


Sec 1 Express End of Year Exam 2011
Mathematics Paper 2 Marking Scheme

Sai) volume of cuboid= Ix bx h = (4)(2)(2) = 16 cm 3 [M1]


aii) volume of 2 cylinders= 2 x n x r2 x h = (2)(3.14)(0.52)(0.5) (M1]
= 0.785cm3 [M1]
aiii) total volume= 16 + 0.785 = 16.785 cm 3 [A1J

Sb) surface area of 2 cylinders =2ndh = (2)(3.14)(1 )(0.5) = 3.14 cm2 [M1]
surface area of cuboid= (4)(2)(4) + (2)(2)(2) [M1]
= 40 cm2 [M1]
total surface area = 40 + 3.14 = .4 3.1 4 cm 2 [A1]

6a)

·Y

A (-3,6) 6

-3 -2 -I 0 2 3 x
-I

-2

-3
B (3,-4)
-4

[1 mark for correct A and B plot, 1 mark for correct line]


Sec 1 Express End of Year Exam 2011
Mathematics Paper 2 Marking Scheme

6b) Gradient= 2:'_i_-Ji 6c) (0. 1) [B 11


Xz -x1
= - 4- 6 [M1]
3 - (- 3)
= .:_!_0
6
= - 1-2 _[Af}
3

7b) [1 mark for all correct plots and 1 mark for correct line]
y
5

4 y · 2x - 5

+-~~-+-~~-+~~~+-~~-+-~.,,__-+~~~-t--~~--t-•
x
-2 -1 0 2 3· 4 5
-I

-2

-3
8.
(a) line AB [1J
arc BC= Scm & ang le 8=·120 [1 J
arc AD=Bcm & angle A=60 (1)
line DC [1)
-7 (b) BCD= 60de!d [1]. CDA = 120 deg [1 J
-8 (c) parallelogram [1J

-9 (d) 2 pairs of arcs (1)


perpendicular bisector [1 )

(e) 2 arcs on AD and CD + 1 pair of


arcs [1 J
angle bisector (1 J
Name Register Number Class

MANJUSRI SECONDARY SCHOOL

~7-*- o/~ '.

END-OF-YEAR EXAMINATION 2011


Subject: Mathematics
Paper: 1
Level: Secondary 1 Express
Date: 6 October 2011
Duration: 1 hour
Setter: Mrs Teo Hwee Ping

INSTRUCTIONS TO CANDIDATES

Write your name, register number and class in the space at the top of this page.
Answer all questions.
Write your answers in the spaces provided on the question paper.
If working is needed for any question, show it in the space below the question.
Omission of essential working will result in loss of marks.

ELECTRONIC CALCULATORS MUST NOT BE USED IN THIS PAPER.

INFORMATION TO CANDIDATES

The number of marks is given in brackets ( ] at the end of each questions or part question.

The total marks for this paper is 40.

Marks Obtained

40

This paper consists of~ printed pages, including the cover page.
2

Answer all questions.


1. Evaluate 52.3 - (-20.1)

>.

Ans: [1]

2. Estimate

(a) 73481 - 51901, correct to 2 significant figures,

(b) 0.059 + 1.02, correct to 1 significant figure.

Ans: (a) [I}

(b) [I]

3. In a school election for the head prefect, there were 3 candidates A , B, and C.

Given that there were 1200 voters and the votes for the 3 candidates were divided in the ratio

of 11 : 7 : 2 . Calculate

(a) the number of votes that the winning candidate received,

(b) the difference between the highest number and lowest number of votes.

Ans: (a) [I J

(b) [J)

2011 EOY Exam I Sec I Express/ Paper I


3

4. Given that 2x < 17, state the largest possible value of x if

(a) x is a prime number,

(b) x is a multiple of 3.

. ..
Ans: (a) (1]

(b) [1]

5. Given that K = 26 x 7 2 , find the

(a) positive square root of K,

(b) cube root of 7K.

Ans: (a) [1]

(b) [ 1]

6. Factorise

(a) l 3bc + l 3c ,

(b) 3(x - 2y) - 7a(2y-x).

Ans: (a) [1]

(b) [ 1]

20 I I EOY Exam I Sec I Express I Paper l


4

7. Simplify

(a) -12a+(- 3a),

2x -1 x - I
(b) - --
3 2

' ..

Ans: (a) [I]

(b) [2]

8. (a) The exterior angle of a regular polygon is 40°. How many sides docs this polygon have?

(b) Given that the interior angles of a pentagon arc x 0 , (x + 30)0 , 3x 0 , (3x + 10)0 and (x + 50) 0 .

Find the largest interior angle.

Ans: (a) [I]

(b) [2]

2011 EOY Exam I Sec l Express I Paper l


5

9. The figure below shows a net of an open cuboid.


The open cuboid has a square base of side x cm. x
Write and simplify an algebraic expression for

(a) the height of the cuboid,

(b) the perimeter of the net of the cuboid,


. ) . 5x+2

Ans: (a) (I]

(b) (2J

10. In the figure below, Dis a point inside triangle ABC such that LACD = 80°, LDCB = x 0 ,
c
L.CBA = 3x0 , and LBAD = L.DAC = 20°.
Find,

(a) reflex LCDA, stating reason,

(b) the value ofx.

Ans: (a) reflex L CDA _ _ __ _ [I](reason: _ _ _ _ _ _ _ _ _ ) [l]

(b) x= - --- [2]

2011 EOY Exam I Sec I Express I Paper I


6

11 . The result of a health survey conducted with 200 adults aged 18 to 69 years old is shown in the
pie chart below.

I-fig h Total
Cholesterol
Diabetes
\ .
126°

Obesity

(a) Calculate the number of adults in the survey who have high total cholesterol.
(b) Besides the 3 categories shown, it was also found that 50 adults in the survey have
hypertension and the rest in the survey have lung disease.
(i) Calculate the angle of the sector representing the number of adults who ha ve
hypertension.
(ii) Complete the pie chart using protractor and a ruler, labeling clearly each sector.

(c) State an advantage of representing this health survey result in a pie chart.

Ans: (a) [l l

(b) (i) [1]

(ii) on pie chart [1]

(c) [I]

2011 EOY Exam I Sec I Express I Paper I


7

12. In the space provided below,

(a) construct MBC in which AB= I 0 cm, BC= 7.2 cm and AC= 6.4 cm. [2]

(b) measure and write down L.ABC.

(c) on the same diagram, draw and label clearly, [2]


' .
(i) the perpendicular bisector of BC,

(ii) the angle bisector of LBA C.

(d) The two lines drawr.i in part (c) meet outside the triangle at a point P.

Measure and write down the distance of P from B.

Ans: (b) LABC = - - -- -- - [I]

(d) PB= [l]


- - - -- ----

2011 F.OY Exam I Sec I Express / Paper l


8

13. Two points A and Bare marked in the Cartesian plane below.

r- - - - - r- -- -- r-- - --r -- -3-


• t • '

'
I I I I B I I I t
~ - -- -- ~ -- - - - ~ -- -- -~ -- - z- ---- -T - -- - - l- --- - ~ - - -- - ,
• I t I

I t I I '.
. .:---.. . --:------:------:----r-------:------:------ ~ -----:
t I I t I I

- :4 ~3 - :2 -:1 0 ~ 2 j 4!
t I I ' I J' '
I I'
__ ___ i __ __ _ i _ _ __ _ J __ _ __ J
I

~ -- - - - ~ -- - - - ~ -- - - - t ----= t I I I I

' ' '


~- - - - - ~---- - ~ -- -- - ~- -.=.~
. .
- - - - -L - -- - - L- -- -- ~ - -- -- J
I I J I

I I A . .
~ - -- -- ~ -- -- -><- -- - - ~ - -=--}
I I I I

' .
' .
~ - - - -- ~ - -- - - ~ --- - - ; --.:..-4

' '

(a} Write down the coordinates of A and B.

(b} On the graph, plot and label clearl y, the point D(l , -3).

(c) Write down the coordinates of the point C such that ABCD is a parallelogram .

(d) Find the area of ABCD.

(e) Write down the coordinaks of a point E such that triang le BDE and triangle ABD have the

same area.

Ans: (a) A= ( __ ) [I]

B=( [!]

(b) Mark on graph above [I]

(c) C= ( , __ ) [I]

(d) [I j

(e) £ = ( _) [ I]

+ end ofpaper +
2011 EOY Exam I Sec I Express I Paper I
END-OF-YEAR EXAMINATION 2011
Sec One Express Mathematics Paper 1
Mark Scheme
'
l. 52.3- (- 20. l) = 52.3 + 20. I

= 72.4 Bl

2. (a) 73500-5 1900 = 22000 Bl


(b) 0.06 + 1 = 0.06 Bl

3. (a) 660 Bl
(b) 540 Bl

4. x < 8.5 -7 (a) 7 Bl


(b) 6 Bl

5. (a) .JK = 2 3
x 7 = 56 Bl 1

2 Bl
(b) V11< = 2 x7 =28

6. (a) 13c(b + I) Bl
(b) (x -2y)(3+7a) Bl

7. (a) --J5a BI
4x -2 3x -3
(b) - - - - - -
6 6
4x - 2 - 3x+ 3 MI
-
6
x+ I Al
= --
6
' - - - - -·

8. (a) Number of sides = 360 .;- 40 = 9 Bl


(b) 9x+90 = 540 MI
x = 50
Largest interior angle= 160° Al

9. (a) 2x+ I BI
(b) Perimeter = 2(5x + 2) + 6x = I 6x + 4 M I,A I
10. (a) reflex L.CDA = 360 - 80 = 280°
- Bl
(reason: angles at a point) Al
(b) x + 3x + 20 + 280 = 360 or 80 + x + 3x + 40 = 180 Ml
.. ) . Al
x = l5

72 Bl
11. (a) - x200 = 40
360

(b) (i) so x 360 = 90° Bl


200
(ii) Correct angle and label Bl
(c) It can compare the parts with the whole. Bl

12. (b) LABC = 40° ( ± 1°) Bl

(d) PB = 3.9 ± 0. I cm Bl

~
I?
,
,

A" ·a

(a) Construction of triangle:


Correct construction lines [Ml]
Accuracy [Al]

(c) (i) perpendicular bisector [Bl]


angle bisector (BI]
13.
y
.
,.. - - - - - r - - - - - r - - - - - r - - - -3- - - - - - ;- - - - - ~ ... - - - - ~ - - -.~ ·)~'
C I : : I ' t t

! !
t "'l
B: t
!
I
~· :
4
~ - - - - - ~ - - - - - ~ - - - - - r - - - :z:.-~ ~- - - - - ~ .. - - - - ~ - - - - - ' - ... ... - - ~
t I I I
: : I t I I I I

f I t : : : : :
-'- -- -- -·------:-- .. - - -,- - - -l -- -----,.--- - -.- - -- - -.- - ---·.
: I I I t I I

'-~~--'!--~~-:!--~~-+~~~-r~~~f,~~~-;-~~~-;-~~--jr--,. x
-:4 ~3 -:2 -:. 1 0 :1 l 3 4:
I I I f I
__ _ _ _ j _ _ _ _ _ J
I ' t t
_ ___ _ J __ _ __ J
~-----~ ----- ~ --- --~ ----=t I I I I
I I O

' '
t I I f
L - - - - - L ....... .... L ....... ... .. l ...... .=.,.J').
I t I : ~
t
___ __ i _ _ ___
I

.
__ __ _

~
I

'
I
J __ __ _ J

'

: . A I : I I I' I

~ - - - - - ~ ---- -':'-- -- - ~ - - ~3 -- - --~-D


- - --~- ----~-----~
t I I I I
I

. I
I
I
I

'
I
I

'
t t

~-----~-----~--- - -;--=--4
'
I

.
I

'
.
-- ---· - - ---4 -- -- -4- ----~ . .
' .
------------------------ . ' --------
-- ---·----------
(a) A= (- 2, -3)
Bl
B = (0, 2)
Bl
(b) point Don graph
Bl
(c) C = (3, 2)
Bl
2
(d) 3x5 = I5units Bl
{e) either (4, -3) or (-3, 2) Bl
Name Register Number Class

MANJUSRI SECONDARY SCHOOL

END-OF-YEAR EXAMINATION 2011 '


Subject: Mathematics
Level: Secondary 1 Express
Paper: 2
Date: 11 October 2011
Duration: 1 hour 30 mins
Setter: Mrs Teo Hwee Ping

ADDITIONAL MATERIALS
Electronic Calculator

INSTRUCTIONS TO CANDIDATES
Write your Name, Register Number and Class in the spaces provided above.
Write your answers and working in the papers provided.
/\II necessary working must be shown.
Submit this question paper at the end of the session.
Answer all questions.

INFORMATION FOR CANDIDATES


The number of marks is given in brackets []at the end of each question or part question.
You should not spend too much time on any one question.
You are expected to use an electronic calculator for this paper.
If the degree of accuracy is not specified in the question and if the answer is not exact, the answer
should be given to three significant figures. Answers in degrees should be given to one decimal
place.

For 1t, use either your calculator value or 3.142.

The total marks for this paper is 60.

Marks Obtained

60

- - -·--- - -
This question paper consists of~ printed pages, including the cover page
[Turn over
2

Answer all questions.

1. With the use of a calculator,


172.68 . .fi ti [])
(a) evaluate , correct to 4 stgnt tcant 1gures,
/ 2
vl2.9 -34.326

9 . d ec1ma.
. I
(b) express - as a recumng • I ' [l)
]]

2. Hazel, rrene and Justin visit the National Library every 6 days, 18 days and 3 weeks
51
respectively. If the 3 good friends meet one another on 31 August 2011, what is the
date that they will next meet at the library again? [3)

3. A sequence of patterns formed by dots and lines are shown below.

x Pattern I
)<X
Pattern 2
X>V< Pattern 3

(a) Find the number of dots


(i) in the s'" pattern, [IJ
(ii) in the n'" pattern. (1 )

(b) Pattern n has 288 dots. Find the value of n. [2]

4. (a) Express 240 millilitres (m A.) as a percentage of3 litres (A.). (I J


(b) From 12.30 pm to 3 pm, the volume of water in an open container decreased
27% to 3 litres.
(i) Calculate the original volume of water. [2)
3
(ii) Calculate the rate of decrease in volume of water in cm per hour. [2]

5. A car travels at 90 km/h for30 minutes and then travels 90 km for I hour 30 minutes.
Find
(a) the distance travelled for the first part of the journey, [l]
(b) the speed of the car for the second part of the journey, [I J
(c) the average speed of the whole journey in m etres per seconds. [3]

2011 EOY Exam I Sec I Express I Paper 2


3

6. (a) Simplify algebraic expression - 4(a + b) +[a - 2(b - a)]. [2]

. . 5 4x-3 I
(b) Solve the a1gebraic equation - - - - ::::_- - x. [3]
2 3 2

7. The bar graph below shows the weight of Mary and Jane over a period of 5 months.

75

.
6 - --------·

55

January February March April May

Legend
~Mary

D Jane

Use the bar graph to ans\ver the following questions:


(a) What is Mary's weight in April? [I]
(b) What is the difference in weight between Mary and Jane in January? [1]
(c) Calculate the average rate of Mary's weight loss per month. [2]
(d) If Mary's ideal weight is 46kg, can she reach her ideal weight by October?
Explain your answer, supported by calculations. [2]

· - -- - -- -- - - - -- - -- ·- - - - - - -- - - · - - .
2011 EOY Exam / Sec I Express / Paper 2
4

8. In the diagram, ABCD is a square and AGFD is a paraUelogram. The lines BD, GC
and CE are parallel lines. AG and BC meet at the point H.

Given that LGFD = 36°,


(a) name two angles that are equal to L GFD, [2]
(b) find L AHC, stating the reason, [2]
(c) find L CED. [2]

9. In a class test, Alice scored x marks, Alvin scored 25% more than Alice and Alex
scored 50% less than Alvin.
(a) Write down, in tenns of x, an algebraic expression for
(i) Alvin's marks, (1]
(ii) Alex's marks. [l]

(b) Given that the sum of Alice's and Alex's marks is 9 more than Alvin's marks,
form an equation in x . Solve it to find the value of x. [3]

(c) Given that Alvin scored 18 marks in the previous test, calculate the percentage
increase in Alvin's marks. [2]

(d) Given that there was a drop of 40% in Alex's marks as compared to the
previous test, calculate Alex's marks in the previous test. [2}

- - ----- - - - - -- - - -
2011 EOY Exam I Sec I Ex press I Paper 2
5

10. A cylindrical container (hollow) of radius 8 cm and a height of 12 cm is mounted in a


wooden prism solid with the same height as shown in the diagram below.

24cm

Calculate,
(a) the volume of the cylindrical container, leaving your answer in 3 decimal
places, [2] /
(b) the volume of wood used in the solid, [3]
(c) the total surface area of the wooden solid (exclude inner cylinder). [3]

11 . Answer tire whole nfthis question 011 graph paper.

(a) A table of values for the equation y =- 3x+ 2 is shown below.

x -2 -I 0 2
y 8 a 2 -1 b

(i) Find the values of a and of b. [ I]

(ii) Using a scale o f 4 cm to represent I unit, draw a horizontal x-axis for - 2 s x s 2 .


U sing a scale of 2 cm to represent 2 units, draw a vertical y-axis for - 6 sy s l 0.

Plot the points in the table and draw the graph of y =- 3x + 2. [3]

(b) (i) On the same graph paper, plot the 3 points (-2, -4), (0, - 2) and (2, 0) and join.
them to form a straight line. [2]

(ii) Write down an equatio n for this straight line graph. [1]

+ end ofpaper +
2011 EOY Exam I Sec I Express I Paper 2
END-OF-YEAR EXAMINATION 2011
Sec One Express Mathematics Paper 2
Mark Scheme
.. t'

1. (a) 15.03 Bl
.. Bl
(b) 0.81

2. LCM of 6, 18, 21 = 3 x 2 x l x 3 x 7 = 126 days Ml, Al

Date they meet again -7 4lh January2012 Al


--
3. (a) (i) 12 B1

(ii) 2n + 2 Bl

2n +2 = 288 Ml
(b)
n = 143 Al

Bl
4. (a) 240 xl00=8%
3000
·.
Ml, Al
(b) (i) JOO x3 =4.l IA.
73

(ii) .!.J..!. = 0.444 x l 000 = 444 cm3 per hour Ml, Al


2.5

5. (a) Distance= 90 x 0.5 = '15 km Bl

90 Bl
(b) Speed= - = 60 km/h
1.5
45+90 Ml, Al
(b) Average speed = - - - = 67.5 km/h
2
= 18.75 mis Al
6. (a) - 4(a + b) +[a - 2(b-a)J = - 4a - 4b +[a - 2b+ 2a) Ml
= - 4a - 4b + 3a - 2b
= -a-6b Al
: '.
15 8x - 6 3 6x
(b) ---- =- - -
6 6 6 6 MI
'
15 - 8x + 6 = 3 - 6x
- 2x =- 18 Ml

x=9 Al

7. (a) 65 kg BI
(b) 75 - 65 = 10 kg Bl
15 Ml, Al
(c) average rate = - = 3 kg I month
I 5
(d) (60 - 46) .;- 3 = 4.67 months BI (relevant calculations)
Mary needs about 5 months after May to reach her ideal BI (logical reasoning
weight assu!ning she lose 3kg I month. By end of October, connected to their
she should be able to reach her ideal weight. calculations)
...

8. (a) LGAD, L.BHA , L.GHC Bl, Bl (any two)


i (b) L AIIC = 180 - 36 = 144° Bl
(either Opposite interior angles of parallel lines Al I
or Angles on a straight line)
(c) L.BDE = 144 - 45 =99° Ml o.e.

L.Ct"D = 180 - 99 = 81° Al

·-

2
9. (a) (i) l.25x Bl o.e.

c11) 50
-xl.25x = 0.625x ..
Bl o.e.
100
(b) x + 0.625x = 9 +I .25x Bl
0.375x = 9 Ml
x = 24. Al
. 30-18 Ml,AI
(c) percentage increase = - · - x l 00 = 66. 7%
18
(d) 60% -7 15 marks

Alex' s marks m
. previous
. 100 x 15 =25
test = ~ Ml, Al
60
3 Ml,AI
10. (a) Volume of cylinder = nx8 2 x12 = 2412.743cm
1
(b) Volume of wood= -(20+ 24)(12) x 16- 2412.743
2 Ml (volume of prism),
Ml (difference)
= 1811.3 ~ 1810 cm3 Al
- - • • •• - - - w

(c) Total surface area =


l MI (at least 2 correct)
20x 16 + - (20+24)(12)x2+13x16 x 2 + (24 x 16- rrx 8 2 )
I 2
Ml (all surfaces considered)
I
Al
Ir-----· = 1446.938 ~ 1450 cm
2

I l I. Graph Paper (P.T.O.)


'- · - "

3
y (a)(i) a = S,b = -4 [Bl}

(ii) Scale on x-axis [Bl]


1O Scale on y-axis (BI]
' Points and line (B 1]

(b) (i) Points [Bl}


8 Line [Bl)

(ii) y = x-2 [Bl]

y = - 3x -l 2

- - -- - - -- - - - - - -...+-----:...--- -- - -- - · --=·------I>- x
-2 -1 2

-6

4
I lnde' Numbe• IName

ANG MO KIO SECONDARY SCHOOL


FINAL EXAMINATION 2011
SECONDARY ONE EXPRESS

MATHEMATICS
Part 1

MONDAY 1 O OCTOBER 2011 1 hour

Candidates an~wer on the Question Paper.

Setter : Mr Eric Lee

READ THESE INSTRUCTIONS FIRST

Write your class, index number and name on all the work you hand in.
Write in dark blue or black pen in the spaces provided on the Question Paper.
You may use a pencil for any diagrams or graphs.
Do not use staples, paper clips, highlighters, glue or correction fluid.

Answer all questions.

The number of marks is given in brackets [] at the end of each question or part question.
If working is needed for any question it must be shown in the space below that question.
Omission of essential working will result in loss of marks.

If the degree of accuracy is not specified in the question and the answer is not exact, the
answer should be given to three significant figures. Answers in degrees should be given to
one decimal place.

The total of the marks for this paper is 50.

ELECTRONIC CALCULATORS CAN BE USED IN THIS PAPER.

For Examiner's Use

50
This document consists of 10 printed pages (Turn over
2

Part J (50 marks)


Answer ALL the questions.

1_ Cons ider the seven numbers stated below.

1 , !!__ '
11
-!"43 ' - 149 ' 31

Write down the


(i) integers,

(ii) prime numbers,


(iii) irrational numbers.

Ans: (i) [I J

(ii) [I]

(iii) [1)

2. Express
(a) 0.010199 to 4 significant figures,
(b) 36 seconds as a percentage of 5 hours,
(c) 1.05% as a fraction in its lowest term.

Ans: (a) [1 J

(b) % [IJ

(c) [1]
3

3. Arrange the following numbers in descending order

0.9 , 0.909 , 99% , J_Q


11

Ans: [1]
- -- - - - - - -- - - --· -- - -

4. Mr Ronnie competed in a triathlon of total distance 75 km. After finishing the 2 km swim
segment in I h 15 min, he proceeded on to complete the cycling segment at a speed of
15 km/h in I h 48 min before finishing the whole race with the running seg1J1ent at
9 .2 km/h. Find
(j) the distance of the running segment,
(ii) his average speed for the whole journey in km/h.

Ans: (i) km [21

(ii) km/h (2)


4

5. (a) Find the smallest value of p, when 630p is a cube.


(b) In the figure shown, ABCD is a kite. Find the value of x 0 •

Ans: (a) [2]

(b) - - - - - - -- - - [ll

6. (a) lf P: R = 5: 11 and Q: R = 4: 5, find P: Q: R


(h) If the original cost of a teddy bear was $200, find its price if il costs 12% less today.

Ans: (a) (I]

(b) [2J
5

7. If x = - 3, y =4 and z = 1.5 , evaluate

x
(b)
yz

J\ns: (a) [2]

(b) [ 1]

8. (a) Simplify 2p - [3(q + 2p)].

(b) Solve the equation 2d +3 = _!_(4d +3).


3

Ans: (a) [2]

(b) - - -- - -- - - [3)
6

9. Charlie draws the following diagram during his art class. It consists of a serni -circle insi<le a
trapezium with a right angled triangle at the bottom as shown in the figure. If the area of

2 22
trapezium ABCE is 144 cm , taking n = , find
7
(i) CE,
(ii) the area of the shaded region.

18cm
B

9cm

9cm

------- ---------- - - - ---- ---------])

Ans: (i) ~~~~~~~~~~~~~~


cm [2]

(ii) cm2 [2)


7

I0. The graph below shows 3 points P, Q and R.


(a) Write down the gradient of PR.
(b) Write down the gradient of QR.
(c) Find the gradient of PQ.
(d) Write down the coordinates of point S such that PQRS is a parallelogram.

t f-· '.'
+-H-++++-'-i- r-!- '~. ~1 .,.4 1-1 Hj
. • . n w·
l ! ?l f r't·~++-+-l''+-~"i..f-.:-JH.-
.d . I I '-'--+;- "t -'-81.-r
~ H , . ·-'
-+- -j · ·tj··'
f- ·.I-:- - r -rt ~; - ~ R • - . : --:-+ .. • [- -1--l.:
l
'·r -•1 ~f:
,
'•I;

• l • .
I:
' I • ..__~,

'
. ·
. i !I
~ + -· i 1-- -!-' ..... l ... 'I ~
1 ~: dt l-'-,-- -t-11
I • :,
•-:t-'--J --!....;_ 1 ~
. , i . +-i-...,.. .• Ir
;-l i i ~j •.
1,

L~ ~
1
l
..; -1 ~
I'

·t -+·.- .-, .,:-,'


• ~·
->-t '.~-- .i.~r~
't-lj_I

~. l~ +-· -~ I
+. f-1 ! ' ! l I I l li'
· - r.-r- , ' · 'I=1-.
I ;
.L ; L _,I +1-1-r' -· '·· -· '• 1; I·-:-r;-
!':
1-1--1-++--1--1~ I

H-f-+j+f .-1-1
t
·-r--
tf ! I·
1r
- :-r- 'J
· t·t r1
'l'• -·
f :i· 1 · : ! l1 ·- J-
l ' • ' ' I
1 :.1-rffff·
fI
r~-·i·-:- -1t -~-i--l--:--t
·-
1
~ rtf·

Lf- ±J~I' -I· •; :r
1

I - -:t l~.r
:t I'- 11·-.
.._...._
r~. ..·~·· ':
' -t .:..lt
l •
·:
. L~
.
i,

,~
;
.-r-i
I ;

1 ~~1 ··• :, .~ ;i., lt:~i--L 1 -··- '+-----rt+ fX:r :: ct,


1
!·1· i 1
1
:r i I•i~'11· ---1-t ~ '. .i :. LI- +j -H--i--...,
. , ,
_ ,!_ !
- -+-•4
; l 1 1
- i i ,
t. l ~ ! -~-· r • - ~ -· --1 L._ •
1 1 +~- ~j j -- - •r-·-· ; __ ; :
·-:+ fr -f - : I ~_L ...'..L.; r r 11 -Hi ·f i ,1- --!+'. ~ ~ r 1 I ; '1 -1-- ;-, H

_:::;__• ~
--+t l ' ~ : ~. : ~: i .:.. lt.
·- !1--· t.f-t·:tt+f --I :~l_ L'._ .1 -~j ! ["J
-'--~J -t-. --+.+j;.. i i t '
t
L1 : ~ .. ' , l . : : I
... ~--·-
,,- - -j--t' "i-+-t ' r IT ! h f • ~' '""'. !-- ,,- , I l - rl -;-- - " t' ~ ,. '
-=F+ HI I+ I j _,__; ~- ~·-'-1--'-i..J_!__,-
· r · T ·H·J-.L i!~ ....:.....\....-:--+!-r-+-~-Ll:_._tr
__~ 1 -J-+
±-4--l
!- ~ ...,. j I; !i 1' ' • ! )' 'J' t t
1'.~t , 1 • _ , .,~ Q . . :
·· 1+ r t , , 1 1 ' i ·; • · 1 • ~ - t · ! ,- - · 1- -tr 1

-H-·
' .. ! : . ----, : t ·_; 4c ~I : ! - .... r- •--;- r -L- - · .
-·' 1-,_ ., -1 ---t
: I ! . . -I - •. ; :-· ' ·1 •
1 j ! t • ! I . ! ~ I i . . r. ; t H .• f t ! ! . I .. i I I ; I ;
"f'' .. .
i·";-
~ ,! .: ,i i:
~
t-1 :- .. :-· "T-,_...__ ...... ~,-~ ~ t I
l t- ~ lr.
f t .
~ f I _. . J. i ~ • .. I 1 r - t--t-t-'· r r - !- t-4- - • ~ I f t t
I ; ' • • I •

• · -!- -.-r-..o- · · ·•
t - t I_
. ---- ... , ~ -"-·r --..,.-r- --1-..f-+----~- :-

Ans: (a) [1 j

(b) [ I]

(c) [I]

(d) _J__=_,_(_ __,____,__ _ __ [1 l


l I. T he pie chart shows the dis tribution of 150 fishes in an aquarium. Given that there are 45
seahorses and that there are as many eels as there are the total number of sharks and
stingrays,
(i) find the number of guppies in the aquarium,
(ii) find the percentage of eels in the aquarium,
(iii) find the angle of the sector in the pie chart which represents the number of sharks if
the ratio of sharks to stingrays is 2 : 7.

Ans: (i) [I )

(ii) % [2]

( iii) {2]
9

12. In the diagram, ABCD is a parallelogram with L ABD = 25°, L DYQ = 110° and

L.BCD = 115°. Given that PQ II SR. calculate


(i) L DRX,
(ii) LBQY.

Ans: (i) 0
[3]

(ii)
- -- -- - - -
0
f21
10

13. Construct triangle ABC such that AB = 8.5 cm, LABC = 65° and BC = 7 .7 cm. The line
[2]
AB has been drawn for you.
(i) Measure and write down the length of AC.
(ii) Construct the perpendicular bisector ofAB. The perpendicular bisector of AB cuts
the line AB at a point P. Label point P. [2]
(iii) Construct the angle bisector of angle L BAC. [1 J

(iv) Mark and label the point of intersection of the bisectors in part (ii) and (iii) with
the letter Q. Measure and write down LAQP.

A 8.5 cm B

Ans: (i) AC= cm [I ]

(iv) LAQP~ 0
[1]

End ofPart 1
Sec IE Final Exam 2011 Mathematics Part 1

I Answer Marking
Scheme
l (i) I, -.J49,31 Bl

(ii) 31 Bl
(iii) 1[ .f,-- Bl
-- 43
11 '
-·--
2 (a) 0.01020 Bl

(b) 0.2 Bl
(c) 21 Bl
- - or0.0105
2000
-- t--·
. • • I0 Bl
3 0.9, 99%' 0.909' -
1I
4 (i) Distance of running segment

= 75 - (2)-(!Sxl.8) Ml

r0;)
=46km Al
-~-- - --
T 1me ta ken for runnmg. == -46 =Sh Ml
9.2
75
Ave speed =
- -- 5+1.8+ 1.25
51 Al
= 9.32 km/h or 9 - km/h
---- ·- 16 I
5 (a) 2 630
I 3 315
iI 3 105
5 35
I II 7 7
J
I
630 = 2x32 x5x7 Ml
II 630k =2 x 3 2 2 2
x 5 x 7 x (2 x 3 x 5 x 7
2
)
AI
I 2 2
. k = (2 x3x 5 x 7 ) = 14700 2

f--1
e---+---
(b) x= 58° Bl
(a)
<;) .
p Q R Ml
5 II
4 5
25 55
44 55
-
25 44 55 Al or 82
(b) 100% - $200 MI
1%- $2
88% - $176
Al
12

1-7 (a) -3(4 - 2.25) Ml


Al
= -5.25 ..
r-- (b) - 3
- - =- -
I
Bl
I 6 2
I x (a) 2p - (3(q+2p)]
- Ml
= 2p - [3q t- 6p ]
=-4 p - 3q Al

1
(b) 2d + 3 =- (4d + 3)
3 e-·-
1-
I 4
2d +3 = - d+ l Ml

~
3
- - -
Ml
2
-- d = - 2
l
!.- 3 .
I
I d =-3 Al
!---- - -
9 (i)
Area of trapezium = }_ x (AB+ CE)x Height
I 2
I
I

I I
I44 =- x (I 8 + CE) x 9 Ml
2
li CE == 14cm
Al

I
(ii) Arca oftriangle CDE = ~ x l4 x9 = 63cm
2

2
I 22 7 2 Ml
l Arca of semi - circle = _!_ x x 7 =77cm
2 7
Arca of shaded region =
63 + (144 - 77) = I 30cm 2 Al
-- -- ' - - - -·.
i JO (a)
(b)
PR = undefined
QR = O
Bl
Bl
- '--- .

I (c) - 4 - (- 3) Bl
PQ = - - -
2- 5
I 7
··- - -
I
1--
1
- -
II
(d)
(i)
-
122
3
S = ( - 1 , 4) Bl
81
I
I
.4 x I 50 = 5 1
360
(ii) I Ml
Eels = - (1 50 -5 1- 45) = 27
2
27
- x i 00 = 18% Al
- ·- -- 150 ·-
(ii i) 9 parts - 2 7 fishes
l part - 3 fishes
2 parts - 6 fishes Ml
6
- - x360° = 14.4° Al
150
13

12 (i) L BDR = L ABD = 25° Ml


LDXR =LDYQ= 110° Ml
LDRX = 180° - 110° - 25° = 45° Al
(ii) LYBQ = 180° - 115° - 25° = 40° Ml
LBYQ= 180°- 110° = 70°
LBQY =180° - 70° - 40° = 70° Al
13 See separate file.
(i) AC = 8.8± O. lcm Bl
(ii) See separate file.
--
(iii) See separate file.
--
--
(iv) See separate file.
LAQP = 63°± I 0 Bl
--
ANG MO KIO SECONDARY SCHOOL
FINAL EXAMINATION 2011
SECONDARY ONE EXPRESS

MATHEMATICS
PART TWO

THURSDAY 6 OCTOBER 2011 I HOUR -15 MIN

Name of Sc!ter: Mr Muhammad Lewin Sim

Instru~tions to candidates:

1. Answer ALL the questions in Section A and Section B.

2. All relevant workings must be clearly shown.

3. Silent non-programmable calculators may be used.

4. If the degree of accuracy is not specified in the question and the answer
is not exact, the answer should be given to three significant figures.
Answers in degrees should be given to one decimal place.

5. The intended marks for the questio ns or parts of the q uestions are
given in brackets [ ].

6. The total mark of this paper is 50.

This Question Paper consists of 6 printed pages (including this page)


2

Section A (22 marks)


Answer ALL the q uestions

(15.3) + J9Js
2
I. (a) EvaIuate
. d . I I
expressing your answer to 3 ec1ma p aces. [I]
3

(:)
(b) Arrange the following numbers in ascending order.
2 • • --
ii' 0.181, J o.0321, 0.4336
2
[I]

2. A cylindrical tank has ·a base radius 8 cm and height 18 cm. The d epth of water in
the tank is I 0 cm. Taking l[ = 3.14, calculate

(a) the total external surface area of the tank, [21

(b) how much more water is needed to fill the tank to the brim. [2]

3. (a) Solve the following equations

(i) 7x + [9 - 2 ( x + 8)] =13 , [2]

3x 2 (x-:- 1) [2]
(ii) - -1 = ·.
4 5

(b) Given r = __£ + 2 , find the value of p if q = 3 and r = 4 . [2]


p- q

2 AMKSS_ 20JJ_FY_ JE_ EM_ P2


3
4.
D __(_4,_x_+_3_)_c_m_~ C R

(3x - 8)cm
(5x + I) cm 2-ccm

A B p Q
(Sx - I) cm

AJJCD is a rectangle which measures (3x - 8) cm by (4.x +3) cm. PQR is a right
angled triangle in whi~h PQ = (5x - l) cm, QR= 2x cm and PR = (5x +I) cm.

(a) Given the perimeter of the rectangle ABCD is equal to the perimeter of the
triangle PQR, find the value of x.
[3]

tb) Hence, find the area of triangle PQR [2]

5. Alex prepares 180 chicken wings, 240 sandwiches and 300 fish balls for a party.
/u;;~:ming each guest eats the same quantity of each type of food,

(a) what is the largest number of guests he can invite, if no food of any kind is [2)
to be left over?
(b) how many sandwiches arc allocated for each guest? [ 1J

An zirticle is advertised for $326.70 inclusive of7% GST. Find the original price. [2)

3 AMKSS_201 J_ FY_ IJ:.-._ Ei,,l_ P2


4

Section B (28 marks)


Answer ALL the questions

7. Tom cycled ford km at a constant speed of 16 km/h and then walked for three-
quarter of an hour to reach his destination. The total time taken for the whole
d- 3
JOurncy was hours.
8

(a) Write down in terms of d, an expression for the time he spent cycling. (1]

(b) Form an equation ind and solve ford. [3]

(c) Hence, find in hours and minutes, to the nearest minute, the time taken for [l]
the whok journey.

8. Answer the whole of this question on a sheet of graph paper.


The table below shows the corresponding x and y values for the equation
2x ·- y - 5 = 0.

(i) Find the value ofp . [1]

(ii) Using a scale of 2 cm to repres1::nt I uni( on both axes, draw the graph of [3 j
2x - y - 5 =0 for 0 ~ x ~ 5.

(iii} Draw and label the line y = -1.5. [I]

(iv) From the graph drawn, write down the coordinates of the point of [Ij
intersection of y = - 1.5 and 2x - y - 5 = 0 .

9. (a) Simplify 2 J_:0.02: 5 2 [I]


4

(b) A polygon has n sides. Two of its exterior angles are 48° and 81° while the [2]
remaining exterior angles are each equal to 21°. Find the value of n .

4 AMKSS_ 20JJ_FY_ JE_EM_ P2


5
I 0. /\. salesman at Young Heart Jewellery recorded the number of customers who
visited his shop each day for 30 days. His records are as follows:

~I
Number ofcustomers per day at shop

6 7 8 8 12 7 9 8 JO
12 7 9 8 10 11 IO 6 11
7 10 6 7 9 7 8 8 10 8

(a) Represent the data by drawing a frequency table with tally. [2]

(b) What is the most frequent number of customers per day? [I]

( c) What fraction of the days of the month saw 8 or more customers at the shop? [ 1J

J}. ln the diagram, AC is parallel to PR and XB is parallel to QC.

Given that LBAX= 95° and /..AQC = 50°, find

(a) reflex LAQC, [I J

(b) LCQR, [l]

(c) LABX. f2]

A
R

5 AMKSS_ 201l _ FY_JE_EM_P2


6
12. A simple ornamental lamp consisting of a triangular prism with a cube as the base,

is shown in Figure I.

The lamp uses a special cylindrical light bulb consisting of an enclosed glass

22
envelope with a metal base, shown in Figure JI. (Take;r = )
7

+-- 30 cm __..

20 cm!_ __ _

r
Im
.. 14 cm
.
r
70cm
glass
envelope

l Metal
base

~- ~

~ 150 cm

Figure l Figure II

3
(a) Find the volume of the ornamental lamp. Express your answer in m . [3]

(b) Find the total surface area of the glass envelope of the special cylindrical [3]

2
light btilb in cm .

End of Paper

6 AMKSS_ 201 l_FY_ IE_EM_P2


I. (a)
562.515 (BI]

(b)
J"o.0321 , o. is i , 2-,0.4336 2 [Bl]
11

2 (a) TSA= 3. I 4 x 8 x 8 + 2x3. 14 x 8 x 18 [M 1]

= I I05.28cm 2 [Al]
(b) Volume= 3.I4x8x8x8 [MI]

= 1607.68cm 3 [Al]
3. 7x + [9 - 2(x+8)] = 13
(a)
7x + 9 - 2x - I 6 = I 3 [M 1]
(i)
5x= 20
x = 4(AIJ
3x_ 1 = 2(x~n.
4 5

15x - 8(x - 1) =l [Ml]


20

(ii) · 15x - 8x+8 =I


20
7x+ 8 = 20
7x = 12
5 [AI]
x = l-
7

(b) 4 = _E_ +2
p-3

2 = __!!___ [Ml]
p- 3
2p-6 = p
p=6 [Al}
4 (a) 2(4x+3)+2(3x-8)=5x+1+5x - 1+2x [Ml]

8x + 6 + 6x - I 6 = 1Ox+ 2x [ M I]
2x= IO
x = 5 [Al]
(b) 1
- x 24 x I 0 [MI]
2
= 120cm 2 [Al]

7 AMKSS_201 l_FY_ !E_EM_ P2


8
5. (a) 2
180 = 2 x 3 x 5 2

240= 2 4 x}x 5 [Ml]

300 = 2 2 x3 x5 2
2
HCF=2 x 3 x 5 =60 [A IJ
(b) 240 + 60 = 4 [Bl]

6. 326 70
· x100 [Ml]
107
= $305.33 [Al]
7. (a) .:!_ h
16
d-9
Or - [Bl}
8
d-3 3
O r -- - -
8 4
(b) cl 3 d - 3
- + - = - · - [Ml)
16 4 8
d+l2 d-3
16
= 8
8(d + l2) = 16(d-3) [M1]
8d + 96 = I 6d - 48
d = 18[Al]
(c) I Hour53minutes [Bl]

8. (i) p=1

(ii)

(iii)

(iv) (1.75,-1.5) [Bl]

(1.75 ± 0.05,- l.5)

(a) 225:2:2500 [BJ}


9.
(b) 360-48-81 = 231 ° [Ml]
231
- - =11
21
n = 13 sides [A 1J

8 AMKSS_ 1011_ FY_ l E_EM_ P2


9
10.

No of No of Tally
Customers days

6 3

7 6
(a)
8 7
-
9 4

JO .5

11 3
- - - - -·· - >--
12 2
-
(b) 8 [BI]

~= 2_ [Bl}
(c) 30 10

11. (a) LAQC = 360- 50

= 310° (Angles@a point) [BI J

(b) LCQR= \80 - 95 - 50(int angles)

= 35°[BIJ

LACQ = 180-50-95( Ls sum of~)

=35°[M 1J

LABX= LACQ (Corre.\· Ls)

= 35°[AIJ

12. (a) Volume=l.5xl.5xl.5 [Ml]


I
+ - x0.3x0.2xl [Ml ]
2
= 3.405m 3 [Al}
(b) 22 22
TSA= - x7x7+2x - x7x70 [MIJ
7 7

= 154 + 3080lMIJ

2
= 3234cm [A I]

9 AMKSS_ 201 l_ FY_JE_EM_P2


Register No Class

Name: - - - -- - - - - - - -- -
--- - --. - - - --'
'Perseverance Yields Success'

iP
Ping Yi. Secondary School
End of Year Examination 2011
Sec 1 Express
Mathematics 4016 / 01
Paper 1 1 hour 15 minutes

INSTRUCTIONS TO CANDIDATES I FOR EXAMINER'S USE


Do not open this booklet until you are told to do so. I

l "'" '
Write your name, class and register number in the spaces at the lop of this page.
Wr~e in dark ~e or black pen.
You may use a pencil for any diagrams or graphs.
/ so
Do not use staples. paper clips, highlighters, glue or correction fluid.

Answer all questions.


If working is needed for any question tt must be shown with the answer.
Omission of essential working will result in loss of marks.
Calculator should be used where appropriate.
If the degree of accuracy is oot specified in the question and ii the answer is not exact. give
the answer to three significant figures. Answers in degrees shollld be given to one
decimal place. For ;r, use either your calculator value or 3.142.

The number of marks is given in brackets ( Jat the end of each question or part question.

Expected Grad e--~ 0 AI _..I- u A2 o_ n_J_ _.__ _o


__]_ _ 84
_· _ _.___o c_s_ -1
Teacher's Comment

Student's Comment

Parent's Comment
and Signature
~----

[Turn over
This document consists ol 1Oprinted pages including the cover page.
2
Answc1· a ll lh c questions.

I. Represent the two numbers (- J4 )2 and 15 -:· (- \fTi) on th<.: number line below.

[2J

• I I I I I I I I I I I I I •
-6 - 5 -4 - 3 - 2 - 1 0 1 2 3 4 5 6

2.
------ - - --- - -
Exp ress the following word statemenL~ algebraically in their simplest fom1.
------------
(a} Subtr.ict the square of a from the quotient ofp divided by the cube of q.

Ans: (a)
- - - [I J

(b) Divide tl1e sum of 4a and Sa by the product of 3 and z.

Ans: (b) _ _ _ _ _ ___ l2J

3 Using a calculator, evaluate


(a) -4l x ( 5)2 + {[3-(- 3)) x 25},

Ans: (a) (I J
- - -- -- - -
(b)

Ans: (b) (I J

(c} 20% of0.385.

Ans (c) - [I]


[Turn over
This exam paper IS the property of Ping Yi Soolndary School. ll must not be dupfocated in part or wholP.
3
Answer all the ll ucstions.

4. Correct 907.5895 to
(i) the nearest whole number,

Ans: {i) ~---'--


. _ _ (1)

(ii) 3 decimal places.

Ans: (ii)-- - -- - - I IJ

{iii) 3 significant figures.

Ans: (iii) -- -- - - - ri l

). The results of a survey on the height of a c lass of 16 secondary school students arc shown in a
frequency table below.

(i) Pill in the blank under the Frequency column. [IJ

Rei hi h cm) Fre


-t--~-""-l

l60<h '!: l65


165 < " ~'---
170 --+--
170 < ,, $ 175 5
175 < " '!: 180 6
- --''---- -!
180 < " !O. 185 3

(ii) Whal percentage of the students in the class has a height greater than 170 cm?
Leave your answer lo I decimal place.

Ans: (ii) - - - - - - % [2]

[Tum over
This exam paper is the property or Ping Yi Se<:ondmy School. It must oot be duplicated in part or wnolc.
4
Answer all the questions.

6. (a) Three numbers arc given as follows.


I" number = 3 x 5 x 7
2...i number - 32 x 52
3"1number = 33 x 5
(i) Find the highest corrunon factor (HCF) of the three numbers.

A11s: (ai) _HCF = ___ __ [l)

(ii) Find the lowest common multiple (LCM) oflhe tlrree numbers.

Ans: (aii) LCM =


~~~~~~~~
[J]

(b) Find the smallest positive integer k such that 22 x 3 x 11 x k is a perfect cube. I .eave
your answer in index notation.

Ans: (b) k= [l J
[Turn over
This cxnm paper is the property of Ping Yl Secondary Sc.hoof. It must not be duplicale<I in part or whole.
5
Answer all the queslions.

4x-3 7x - 3
7. Solve the equation - - = - - +I.
2 9

..
,

Ans: ~----- l3J

(Turn over
Th<S exam paper 1s the prope<iy of Ping Yi Secon<lary School. It must not be ctuplicatecl in pan or whole.
6
Auswcr a ll th e lluestions.

8. (a) TI1e ratio of Aily's height to Bily's height is 3:5. The ratio of Aily's height to
Carny's height is 2:3. Find the ratio of Aily's height to Bily's height to Camy's
height.
' .

Ans: (a) - - - - - [2]

(h) A 4-minutc overseas call costs $20.60. f\.1r Lee make.~ an overseas call from
09 55 to I0 I0. Assuming that the cost per minute is the same, calculate the cost
of Mr Lee's call.

Ans : (b) -"s'-·_ __ _ _ (2)

(Turn over
This exam paper os tho property of Ping Yi Secondary School. It must not be duPlicated in part or whole
7
Answer all lbc questions.

9. (a) Express 12 cm as a percentage of 4 rn.

' '
Ans: (a) % [2]
~~~~-----'--=-

(b) 4% ofa number is 9.7. find the number.

Ans: (b) _ _ _ _ __ [2]

r:h+31J
l 0. (a) Evaluate the value of \;ff ";' - - - when a = R, h ~ 2 and c = 6. J.eavc your answer
1 a · - 1oc

to 3 signilicanl figure.

Ans: (aJ
--- - - (2]

(b) Given that y =4( ~ 1· !!.. ) -d11 2, lin<l thc value ofy when d = 6 and 11 • 3.
II d

Ans: (b) ~ - (3]


[Tur n over
This exam paper IS the prQPe<ty of Pmg Yi Socondary School II must no4 be OOplieated in P.111 or whole
8
Answer all the questions.

11. (a) Factorise completely Z.5ax- Sa - 35bx + 7b.

'.

Ans: (a) _ _ _ __ _ _ (2)

(b) Simplify(3y+5) - (5(4y - x ) - 8y].

Ans: (b) - - - · - - - - [3]

[Turn over
This exam paper is the prope<1y of Pin9 Yi Secondary School. It must not be duplicated in part or whole.
9
Am»>tr all the questions.

12. (a) (i) Write down a formula for an interior angle of a regular n-sidcd polygon,
in tcnns of 11.

Ans: (ai) _ _ [I)


~~--'-

(ii) Hence, if each interior angle of a regular n·sided polygon is 108°, find the
value of n.

Ans: (aii) _!!...:._ [2j

(b) A hexagon luL~ exterior angles 85°, 74° , 105°, 2y0 , 3y0 and Jy0 • Find the
value of y.

Ans: (b) y =
_ _ __ _ 13]

[Turn over
Tt11s e<am paper is lhn J>roperty of Ping Yi Secondary School. II musl not be duJ)lial1cd in par1 or whole
10
Answer nll the questions.

13. Every student in a school was asked to choose a book, a CD player or a set of computer
games as a birthday present. 1l1eir choices were presented on the pie chart as shown below

Calculate the
Computer Games
(i) value ofx,

CD
Player

Ans: (i) -'~"--·-


=_ _ _ - [2]

(ii) fraction of lhe students who dilln21 choose computer games,

Ans: (ii)
- -- - - - - [21

(iii) the number of stuctents who chose computer games if then: were 1440 student~
in the school.

Ans: (iii) _ _ _ _ stuctenls [2]

End of l'apcr
[Turn over
llis exam p.ipcr cs the property of Ping Yi Seoondary School. II must not be d<(llicated in part "'~
S econdary IE Fi nal Yea r Exam 2011 Ma rking Scheme

l'apcr I

Qn Marking J>oi n t M ark Re ma rks


Awarded
'1
15 ~(- V27) (- 14 )l Bl , Bl mark for each
correct dot at -5 and
... I
·6 • 5 •I .4
I I I I 1I I
·3 ·2 ·1 0 2
I
I
3 •4
I
I
5
I•
6
4. Minus JM for no
labelling.

Bl
'
.•.
2b 9a Ml
3z
3a
=- Al
z
- 5)2 + {[3 - (- 3)) x 25} - -250 Bl
-- Accept ONLY
- (- 2)2 0.0909.
5 0. I' 0.0909 Bl
19
3c 20%0 f 0.385 - 0.077 131 --
-
4i 908
·-·
I 131
4ii 907.590 Bl
4iii 908 RI
--
Si 14 Bl

Sii 1 Ml
~xl00%
36
8
=38 - %
9
=38.9o/c, Al
6ai llCF = 15 Gl
2
6aii I.CM= 3 3 x 5 x 7 " 4725 Bl Accept ONLY 4725.

6b k~ 2 x 32 x I 12 DI Accept ONLY index


-
form.

I
I -
2

7 4x-3 7x - 3
-
- - = - - +I (multiply by 18)
2 9
9(4x - 3)=2(7x-3) + 18 Ml
36x - 27 = I4x - 6 -~ 18

,.
36x - 14x = 12+27 Ml
.. ' .
22x = 39 I
17
x = 1- orx = 1.77 Al
22
~·--
8a A:B = 3:5 = 6:10 (x2); A:C ,,- 2:3 - 6:9 (x 3) Ml
A:R:C = 6:10:9
Al
. ... ..
8b 09 55 to 1010 - ISmin
-·-·-- --- - -

15
- x$20.60
15 min - 4 Ml
i =$77.25 Al
- -- -·- --· -·-·-- ··
9a
-
12
x l00% ! Ml
400
I Al I

'
-- .
= 3%
l!
9b 100 Ml
- -· x 9.7
4
=242.5 Al I'
I
l
.- ·-
!On J8x2+3 x8
- .1
I 8i - 10x6 I
I I
I
-- ~4 Ml '
I
j
= JIO
= 3.16 Al
II
>-·-
!Ob
·-- --
4 X (6 ~?
-- - --· - ·- - - - ------ ,
- + --3) - 6 X .;>- I
3 6 Ml
I
= 4 x 2.!. _ 54 Ml
2
= 10 - 54
I
I
= --44
Al

Ila 25ax - Sa - 35bx I- 7b


- - ----
Ml
- 5a(5x- l)-7b(5x - l)
~ (Sx 1)(5a ~ 7h) or(Sa - 7b)(5x - I)
Al
··-
l I b (3y+ 5) - (5(4y -x ) - 8yJ
- ·-· ---·- -- ~

2
3

= 3y+5 - [20y-5x 8y] Ml


= 3y 2y+ 5x+5 Ml
Accept
= 5x 9y -~ 5 or - 9y + Sx+ 5 Al equivalent fo
12ai (11-2)
- x 180. Bl
II

I 2aii (11- 2)
-
- - x 180 = 108
II
Ml

18011 - 360 = 10811


7211 = 360
II= 5 Al
12b 85°+7'1° +1 05°+2y 0 +3y0 +3y0
-- 360° -
Ml
264 + 8y = 360
&y = 96 Ml
y - 9618
- 12 Al
- -- - -

c_
13i 5x~2 10 - 360
Sx= 150
x - 30 I

l Ji i Sx30
360 Ml
5

L
' I 3iii
::· -
12

2 10xl'1'10
Al

Ml I
360
=840 _.___
Al l__ ______,

3
Register No. Class

'Pers everance Yields Success'


~-- J
..

Ping Yi Secondary School


j)
End of Year Examination 2011
Sec 1 Express
Mathematics 4016 / 02
Paper2 1 hour 15 minutes

INSTRUCTIONS TO CANDIDATES fOR EXAMINER'S USE


Do not open this booklet unt il you are told to do so. /
/
Wnte your name, class and register number in the spaces al the top of this page. Pa per 2 ;
Write in dark blue or black pen.
You may use a pencil for any diagrams or graphs.
/ 50
Do not use staples, paper dips, highlighters, glue or correction fluid. - ---- /
Answer all questions.
If wofl<ing is needed for any question ii must be shown with the answer.
Omission of essential wol1<ing will result in loss of marks.
Calculator should be used IWlefe appropriate.
If tile degree or accuracy is no1 specified in the question and ff tile answer is not exact. give
the answer to three significant figures. Answers in degrees should be given to one
decimal place.
Fr>r " · use e~hcr your calculator value or 3.142.

The number of marks is given in brackets [ ) at the end of each question or part question.

Expected Grade Al A2 BJ J\4 cs


0 0 0 0 0
Teac her's Comment

Student's Comm ent

I~nd
Parent's Comment
Signature
[Turn over

This document consists of 10 printed pages including the cover page.


2
Answer all the questions.

I. (a) Arrange the following in descending order.


-I {f. •
6' 0.67, ..,6, 0.6.

[2]

(b) Evaluate the following using a calculator. Leave your answer in fraction form .

[- :>- -3 - ( - 6. -' 'J'


J x v, c;

4 2

Ans: (b) _ _ __ _ f!l


.. - ------~ -- - - -- -
2. The diagram below shows the floor plan of two rooms in an apa11ment. Calculate the
to laI floor area nf the two rooms.
.....__ 6m -
2m•
t
t
4m

fl\2 [3)

I his P.xam pap~r b~ tho property of Ping Yi Secondary School It must not be d\1p1ic.-, tcd in pan or whole.
3
Answer all lh e {1uestions.

3. The table below s hows the first live tcnns of a number sequence,
T1 = 1, T2 = 2, T3 9
5, T4 = 10, Ts : 17, .....

(i) Fill in the table for T6 amt T20 . [2]


' ..

r.
"I l=O+l
2 2=1 I I -
3 5=4 + I
-
4 10-9 + l
5 17 - 16+1
6
---
~

20

(ii) Write an expression, in tem1s of 11, for 7~.

Ans: (1i) [I]

4. The diagram be low shows an open container, used as a vase, with a regular hexagonal
cross-section. Given that AB is 6 cm, ve1tical height is 30 cm and the area of the equilateral
triangle AOB is 15.6 cm 2, find the total surface area of the contniner.

30cm

,>---... ~
, ''
' '\
?
Ans: cm· [3]

1his exam paper 1s the property of Ping Yi Secondary School. ll must not bo duphcattld 1n pan or whole
4
Answer all the <111estions.

5. Mr X pla1mecl to travel from location A to location B. He walked at an average speed of


4 km/h for an hour. He then rented a bicycle and cycled the remaining 8 km of his journey.
His cycling speed is 12 km/h. Find the .;
(i) total time taken, in minutes, for Mr X's journey from location A to B,

Ans: (i) mm [2)

(ii) average speed for Mr X's journey in 111/min.

Ans: (ii) .!nimin (2)


----~-

This exam paper is the prooerty of Ping Yi Se<..-ondary School. It must not be dopticated in part or whole.
5
Answer all the questions.

6. (i) Fill in the missing numbers in the factor tree.

[3}

Q . ..

o;,,< () 455

(ii) Express 910 in pnme factorisation form.

An.~: (ii) _ _ _ _ _ _ fl )

This wcam paper is the prope<ty ol Ping Yi Sccond~ry School. It must not be duplicated in pall or wholtt
6
An swer a ll the questions.

7. Simpli fy

(a) ( 4a -3 +12c}( ~ )-2c(a +I),

' '

Ans: (a}
- -- -- - l2J

Jx 1 2 _2(x-I)
(b)
s 3

Ans: (h) _ __ _ _ _ _ _ (31

This exam paper is the property of Ping Yi ~ry School. II must nol be duplicated in p<J<1 or whole

.,
7
A.uswe1· all the questions.

8. In the figure below, A BCD is a square of side 21.2 cm. A, B, C and Dare points on the

circumJcrcnce of a circle with diameter 30 cm.


::. . ·.
(i) Find the area of the circle. (Taken = 3.142)
''
''
''
',30cm
''
'
'''
''

>
Ans: (i) cm· [21

(ii) Find the area of the unshaded region of the circle.

2
Ans: (ii) ~~~~~~~~
cm [I)

(iii) Find th<.: total area of the shaded portions.

Ans: (iii) err/ [2]

l his exam paper 1s the property of Ping Yi Secondary School. It rnust not be duplicated in part or \Vhole.
8
Answc1· all the questions.

9. Jn the diagram.below, BCDE and FGH are parallel lines, ACF and ADG a.re straight lines,
L.Ci!D =27° and L.ACD = 50°. Stating all appropriate reasons in your working, calculate
(i) LEDG,
H E

/
/_..;_/___
...
F /-· B
Diagram not drawn to scale

/
Ans (i) _ __ " [2]
----
(ii) , ,_ enc;,

Ans: (ii) • (2)


-------~

(iii) L.DGF.

Ans: (iii) _____ _____:__ 12]

This P.xam paper is the property of Ping Yi Secondary School. It must not be duplicated in part or whole
9
Answer all th e <1ucstio us.

10. (a) The price of an adult admission ticket 10 an amusement park is $23 more than the
price of a child admission ticket. A family of3 adults and 8 children paid a total of
$201 for their admis.~ion tick els. Find the price of an adult ticket.
..

i
Ans: (a) _s_ _ __ _ - [3]

(b) The lengths of two pieces of rihbons are (a(Sx -y) - 3cr] cm and 3c(9x - y) cm.
(i) Find the total length of the two pieces o f ribbons. Simplify your answer.

Ans: · -- ~(2J

(ii) Factorise the answer in b(i).

l\ns: b(ii) [21


---
This exam paper is the l)<opcrty of J>1ny Yi Secondary Sehool It must not be Ouplic.itcd 1n part or whole.
10
A.1Jswcr all the q uestions.

I I. In tl1e figure below, PQRS is a square and POR is a straight line. PQ • (a+ IO) cm,
QR = (6a - 5)cm andSO~ST. (a 1 IO)cm
(i) Write down the value of LOST. Pr - - - - -- - ; Q
Hence, find the aJ1gles m and n.
!
"I
(6a - 5) cml .0 /
'(,._
'--\··.
111 \ n

\.
s· '/'

Ans: (i) LOST= _ _ ._ [ !]

Ill = :. [1]

11 = • l lJ

(ii) Find tltc value ofa.

Ans: (ii) ~ (2]

(iii) Find the perimeter of the square.

_ _ Ans: (iii)
......:...;::..:.:.....~..'..-=======--o==-"=
cm (21
Jo;nd of Paper

lhis oxnm paper is the properly of Ping Yi SecondRry School. It must not h" duplicated in part or whole.
Seconda ry 1 E Fin al Year Exam 2011 Marking Sche m e

Paper 2

Qn !forking Point Mark Remarks


Awar ded
la • -I Bl, Bl · 1 mark for each
.J6, 0 .67,0.6, 6 . correct pair

Bl Accept ONLY this.


lh [-5 ~ - (-6.!.)]3 x~
2• = 3
3
4 2 8
2 Arca of trapezium = 0.5(6 + 8) x 6 = 42 m' Ml ·-
Arca of rectangle = 4 x 3 - 12 m2 Ml
2
Total = 54 m AI
·--
3i T6: 26 - 25 + I RI Accept only this
equation fonn
T10: 362 - 361 + I Bl
3ii 1;,=(n - 1)2+1 or 111 211 I 2 Ul Accept BOTH
.
4 Base area -6 x 15. 6 Ml
=93.6cm2
Area of 6 side faces ~ 6" (6 ><30) Ml
= 1080cm 2
Tomi = 93.6 I IORO
- 1173.6cm2 Al

Si 8 2
- -
Cycling time ~ dis= _ ,. h
12 3 Ml
Tota l time taken = ( I + 213) h = (60 + 4-0) min
= t 00 m in Al

Sii Walking distance = 4 km, Cycl ing distance - 8 km


. --
Total distance~ 12 km 12000m
12 000 Ml
A vernge speed = - 1
- 00-
120 m/min Al

6i 910, 5, 7 (top to bottom) B3


6 ii 2 x 5 x 7 x 13
- Bl Accept any
arrangement.
2

~
79
(4a -3+12c)( ) - 2c(a +I)

= ~a -~+2c-2nc-2c M
6 6
2 I
= - a - 2ac - - A
3 2

~---1---------
7b 3xi-2 _ 2(x- l)
- - - - - - - - - ·--t-- -
5 3

= 3(3x
-- 1 2) l O(x
_.__ -_ 1) M
15 15
9x+6 lOx-10
=- - -
15 15
= 9
__x +..;.
6_-_,_(l_O_x _
- _IO..:.) M
15
9x-l0x+6+10
= -
15
- x+ l 6

:-t ·-- ·~I


= --
15 A •

hi _A_r_ea_o_f-ci-rc-l-e -_-3-.1-4_2_x_(3_0/2_)~2-------+--~
~-- -~-- _._ ·_·-- _
2

_ _ _ _ _-_7_C_l6.9_5_c_:rn_i ·- - - -- - I -- -
1 8ii Area of unshaded rcgion=2 l.2 x 21.2 ~ 449.44 cm'
!- o···
nlll Arca of shaded region
~ 11rea of circle - arcn of square
~ 706.95 - 449.44 M
= 257.51 cm2 A
l-
9i .CADC - 180° - 50° - 27° (L sum of£\) M Deduct 1M from total
if there is(arc} wrong
"' 103° reason(s).
L EDG =L'.ADC (vt:rt. opp. Ls)
- 103° A

- -· - - - - -- -1-- - - -+- - - - - - - .
L CDG + l.'.EDG (orLADC) Deduct IM from total
if there is(are} wrong
~ I 80° (adj. Ls on str. Linc} reason(s).
L CDG • 180° ·- ~ M
*Accept error carried
- 77° Al
forward from IOi)
- - - - ·- - -
2
3

9iii Method t Deduct 1M from tota


if there is(are) w rong
L DGF + L CDG - 180'' (int. Ls, II lines) reason(s).
L DGF ~ 180° - 77° . Ml
Al • Accept error cam e d
= 103° forward.
Meth od 2
LDGF ~ L EDO (corr. L s, II lines) Ml
= 103° Al

IOa Let the price of an .adull ticket h e Sx.


Then the price of a chik1 ticket is$( x - 23).

i 3x + 8(x - 23) = 201 Ml Accept non-algcbr


method.
3x + 8x 184 = 201
1 lx ~ 385 Ml

385
I x= - 1-1
I =3s Al
--
I Ohi a(8x y ) - 3cx . ~ 3c(9x - y)
Accept other
- llw. - ay - 3cx + 27ex 3cy Ml
arrangem ent of the
I=Bax ay + 24cx 3cy Al final 4 terms
f--- - l -
l Obi i 8a.r - ny ~ 24cx - 3cy
- " -

- a(llx y) 1 3c(8x - y) or 8x(a I Jc) y(a " Jc) Ml

= (8.\ - y) (a + 3c) or (a 1 Jc)(8x- y) Al

11i LOST = 4 5° Bl
1

-1 180" - -4 5-. - -
I Jn - 2 (isos. ti)
-= 61.s• Bl

II - 90° - 67.5° - 22.5" Bl

11ii a + 10 = 6a 5
- Ml
-
.
5a = 15
a =3 Al

t I iii Perimeter= 2(6(3) 5) 1- 2(3 + 10) or 4(3 + 10) Ml


= 52 cm Al
_J
BUKIT BATOK SECONDARY SCHOOL
SECOND SEMESTRAL EXAMINATION 2011
Secondary One Express
Mathematics
PART1

Class: Sec 1

Dat e: 10 Oct 2011

Duration : 1 hour ( 10 15 - 11 15)

READ THESE INSTRUCTIONS FIRST

1. Write your name, class and index number in the spaces provided on this
page.

2. Answer all questions.

3. Write in dark blue or black pen.

4. Write your answers in the spaces provided on the question paper.

5. If working is needed for any question.show it in the space below that


question.

6. Omission of essential working will result in the loss of marks.

7. CALCULATORS MUST NOT BE USED IN THIS PART OF THE PAPER.

8. This paper consists of 14 questions.

9. The intended marks for each question or part of a question are given in
brackets [ ].

10. The total marks for this paper is 40.

This question paper consist s of 8 printed pages, including this page.

Do n ot turn over until yo u are told t o do so.


Bukit Batok Secondary School
Mathematics Secondary One Express S.42 201 I Part 1

CALCULATORS MUST NOT BE USED IN THIS PART OF THE PAPER.

Consider the following numbers:

5 r;:;:;
81, 0.13, 17, I, 64, 125, J[, 11, v 23
13 '

Write down
(a) two prime numbers,
(b) two cube numbers,
(c) two irrational numbers.

Ans: (a) _ __ _ _ _ _ _ _[IJ

(b) _ _ __ _ _ _ _[I]

(c) " - -- - - - -- - [I J

2. Given that 198 = 2 x 32 x 11 and


60 = 2 2 x 3 x 5 ,
find
a) the LCM of J 98 and 60. Express your answer in index notation.
b) the smallest integer, k, such that I 98k is a perfect square.

Ans: (a) _ _ _ __ __ _ _ [J]

(b) _ _ _ __ _ __ [l]

3 Terry weighs 60kg, while Sidney weighs 72 kg.


By what percentage is Sidney heavier than Terry?

Ans _ _ _[2]

2
Striving for accuracy & precision
Buki1 Bu1ok Secondary School
11!fathemarics Secondary One Express SA2 201IPart1

4 .
G 1ven l hat -Sx ;:;; -Jy , fim d the vaI ue o f x: y.
2 10

Ans ._ _ ___ [!]

5 The table shows the average temperature of four different countries on I st January
2009.

Country Sin a ore Greenland Norwa


Temperature 30°C - 22°c

(a) Which country was the coldest?


(b) What was the difference in temperature between Hong Kong and Norway?
(c) What was the average temperature of the four countries?

Ans: (a) - - - - -- -· -- - -[ I J

(b) [I)

(c) _ _ _ __ ___ _ [I]

. . 2
6. {a) SoIve the mequahty - x 2'.' 11.
3
(b) Use your answer in part (a) to find the smallest value ofx if xis an integer.

Ans: (a) _ _ _ _ _ __ _ _ [l]

(b) - - _ _ _ _ _ _[I]

3
Striving /or accuracy & precision
H11kit Batok Secondwy School
Mathematics Secondmy One Express SA2 2011 Part I

7 Evaluate
(a) (-2) 3 - ( - 9x4)-;-(2x3 2 )

Ans_ _ __ __ __ [2]

2 I 1 I
(b) 2 - x(- - )---;- -
3 4 12 2

Ans_ _ _ _ _ _ [2]

8 Expand and simplify the following expressions.


(a) 7m-5k-3(2k+3m)

Ans _ _ _ _ __
_l!J

4
Striving for acc11racy & precision
Bukit Batvk Secondary Schoof
Mathemati<.:s Seconda1y One £\press SA2 201 I Part I

Ans_ _ _ _ _ __[2J

9 Write down the missing terms in the following number sequenc es. [2]

(a) 5, 7, 11 , 19, 35,

I 3 7 21
(b) , 10,,
2. 4, 6,,

4 - 2y 6y + I . ti . . . . fi
I0 E xprcss - - - - - - as a s ingle ract10n m its simplest onn.
s 3

Ans_ _ _ __ __ [3]

5
Strfri11g/or accuracy & precision
Bukit Bat0k Secondary School
Mathematics Secondary One Express SA2 2011 Parr I

11 In the figure, ABCD is a parallelogram.


If the area of parallelogram ABCD is 240 c m 2 , find the length of BC.

A
B
Ans- - - _ _ [2]

12 The back to back stem and leaf diagram shows the numbers o f s it-up completed in
o ne minute by 13 boys and 17 girls in Sec I E5.
-
Boxs Girls
,_. -- I 9
I
·- - ·- -
9-
·--
2
..
3
- 37 6 7
- - ·-
7 3 I 4 2 5 7 8 9
9 8 66 5 4 5 3 4 5 6 8
7 5 3 6 2- 8
Key: 314 means 43 Key: 415 means 45

(a) What is the m edian number o f sit-up completed by the g irls?

Ans: (a) _ _ __ _ [I ]

(b) What is the mode o f the number o f s it-up completed by all the students?

Ans: (b) _ _ _ _ _ [ IJ

(c) Distinction is awarded to the best 20% of the students. What is the least
number of s it-up has to be completed by this group of students in order to
achieve distinction grade?

Ans: (c) _ _ _ [ I]

6
Striving for accuracy & precision
Bukit Batuk Secundary Schuol
Mathematics Secondmy One Express SAl 201 1 Part I

13 In the diagram shown, BDE is a straight line and line AE is parallel to line BC.
T riangle ABC is an isosceles triangle where AC = BC.

LAEB = 37° and LACB = 50°.


Calculate (a) L.EDC,
(b) L.BAC,

(c) LA.BE.

B---+----~-' C
State your reason clearly for each angle property used.

(a)

A.ns(a)_ _ _ __ .l2 J
(b)

A.ns(b)__ __ [I ]

(c)

A.ns(c)_ _ __ _ _ [J l

7
Smvzng/or accuracy & precision
Bukit Balok Secondary School
Mathematics Secondary One Express SA2 201 I Part I

14. The pie chart shows the composition of a fast- food product.
It is given that there are twice as much of Vitamin & minerals as Fat.
(a) Calculate the value of w.
(b) Calculate the percentage of Fats in the fast-food product. Give your answer
as a fraction.
(c) Given that one such fast-food product contains 200 grams of proteins,
calculate the total mass of the fast-food product.,

--------
....

Carbohydrates

Vitamines \
& minerals
101

w
160 Fats

Proteins

Ans:(a)_ _ _ _ [2]

(b)

Ans(b)_ _ _ _ _[2]

(c)

Ans(c) _ _ [2]
End of paper

8
Striving/or accuracy & precision
8uk11 Hatok Secondary School
Mathematics Secondary One Express SA2 201 I Part I

Answer key

QI (a) 17, I I (b) I, 64, 125 (c) IT, J2.J


Q2 (a) 22 x 32 x 5 x 11 , (b) 22

Q3 20%

Q4 3 : 25

Q5 (a) Greenland (b) 24°C (c) 3°.c

I
Q6 (a) x216 - (b) 17
2

5
07 (a) -6 (b)
6

Q8 (a) -2m-llk (b) 4-2x

13 31
09 {a) 67 ' 13 I (b)
-3 '12
7-36y
QIO
15

Qll 20cm

Q 12 (a) 48 (b) 56 (c) 59

Q13 (a) 87°. (b) 6 ~o


) . (c) 28°.

Q14 (a) 33°. (b) 9_!_% (c) 450g


6

9
S1r1v111gfor accuracy & precision
BUKIT BATOK SECONDARY SCHOOL
SECOND SEMESTRAL EXAMINATION 2011
Secondary One Express
Mathematics
BUKIT BATOK
SECONDARY SCHOOL
PART2

Class: Sec 1

Date.: 1O October 2011


60
Duration: 1 hour 30 minutes ( 11 25 - 12 55 )

READ THESE INSTRUCTIONS FIRST

1. Write your name, class and index number in the spaces provided on this page.

2. Answer all questions.

3. Write in dark blue or black pen.

4. If working is needed for any question, show it in the space below that question.

5. Omission of essential working will result in loss of marks.

6. You are expected to use an electronic calculator to evaluate explicit numerical


expressions.

7. If the degree of accuracy is not specified in the question, and if the answer is not
exact, give the answer to three significant figures.

8. This paper consists of 9 questions.

9. The intended marks for each question or each part of a question are given in
brackets [ ].

10. The total marks for this paper is 60.

This question paper consists of 13 printed pages, including this page.


Do not turn over until you are told to do so.

(Type text)
Bukit Balok Secondary School
Mathematics Secondary One £rpress SA 2 20 I I Parr 2

Answer all the questions in the spaces provided.

(a) Use a calculator to fimd t I1e va Iue o I


. ~ 3.46-' . .
, g1vmg your
0.632 x 539
answer correct to 3 significant figures.

Ans (a)_ _ _ __ _[ I]
- · ·- - - ·---------- - - - -- - - -- -- - --------- - -
(b) In 200 I, the price of one litre of petrol was 72 cents.

(i) 65% of this price is 'tax ' and the remainder is 'other costs'. Find, in its
simplest form, the ratio of tax to other costs. Give your answer in the form
m: n, where m and n are integers.

Ans (b)(i) _ _ _ _[I]

(ii) ln 2002 the price of one litre of petrol was 81 cents. Calculate the percentage
increase in the price of petrol from 2001 to 2002,

Ans (b)(ii)_ _ _ _ __ [l]

(iii) The price of petrol in 2001 was 12% less than the petrol price in 2000.
Calculate the price of one litre of petrol in ;woo. Give your answer.correct to
the nearest cent.

Ans (b)(iii) _ _ _[2]

2
Striving for accuracy & precision
Bukit Batok Seconda1y School
Mathematics Secondary One Express SA2 2011 Part 2

2 (a) Jolm wants to cover his living room with new floor tiles. The entire floor
measures 600cm by 280cm. Given that he only wants to use whole pieces of
square tiles, find
(i) the largest possible length of the side of each tile,

Ans (a)(i) _ _ _ [2]

(ii) the number of such tiles that are needed to cover the floor.

Ans (a)( ii)___ [I)

(b) Solve the following equations.


(i) 9(4 - x) = 2 + 8x .

Ans(b)(i)_ _ _ __ [2]

3
Striving.for accuracy & precision
Bukit Butok Secundury School
Mathematics Secondary One Express SA2 201 I Part 2

3b - 2 b+3 4b - 5
2(b) (ii)
2 5 3

Ans(b)(ii)_ _ _ _ _[3]

4
Striving.for accuraly & precision
Bukit Balok Secondary Schoof
Mathematics Secondary One Express SA2 201 I Part 2

3 A piece of wire is cut into three pieces and bent into three different shapcs.
The first piece is bent into a circle of radius 7 cm.
The second piece is bent to fonn a rectangle of length 4x cm and width 2, c m.
The third piece is bent to form a right-angle-triangle of side 3x cm, 4x cm and the longest
22
side, 5x cm. Take Jr= - where necessary.
7
(a) Express in le~ of x, in its s implest form, the perimeters of the rectangle and the
triangle respectively.
Second piece ( rectangle ) [l)
~------------~·

Third piece (triangle)_ _ _ __ _ _ _ __ ____ __ ___ [I]

(b) If the length of the wire is I 16cm, form an equation in terms of x. Hence solve the
equation.

(b)_ _ __ [2J

(c) Find the total area of these three shapes.

(c)_ _ __ [3]

5
Striving for accuracy & precision
Bukit Batok Secondary School
lvfathemarics Seconda1y One Express SA2 201 I Parr 2

4 (a) Factorise each of the following completely:


(i) 15nbc-6ac,

Ans(a)(i)_ __ _ __ __ _ [l l

(ii) 2ab + 4bc - 8cd - 4ad .

Ans(a)(ii)_ _______ _ _ _[2]

(b) The total cost $C, of placing an advertisement in a magazine consists of a


fixed charge of$5 and an additional charge of 50 cents per word used.
Mrs Ong wants to place an advertisement in the magazine,
(i) If it contains n words, write down a formula connecting C and n .

Ans (b)(i)_ _ ._ _ _ _ _ _ __ [1]

(ii) If the advertisement that Mrs. Ong wants to put up contains 60 words, how
much does she have to pay?

Ans (b)(ii) _ __ [1]

6
Striving for accuracy & precision
Bukit Ba10k Sewndary School
Ma1hemarics Secondary One Express SA22011 Part 2

5 (a) ABCD is a parallelogram. Stating the reasons clearly, calculate the unknown
angles p. q and r .

Ans p = ----~[!}

Ans q = _ _ _ _ _ _[2]

Ans r= - - - -- [2]

7
Striving.for accuracy & precision
B11kit Batok Secondary School
Mathematics Secondary One fj:press SA2 2011 Part 2

5 (b)( i) Find the size of each interior angle of a regular pentagon.

Ans (b)(i)_ __ _ [I]

(ii) The diagram below is made up of a square, a regular pentagon and an incomplete
regular polygon ABCDE .... of n sides. Find

(a) L'.'.BCD, the interior angle of the incomplete polygon,


A

D 1----~

Ans (b)(ii)(a) _ _ _ _ _ _[l]

(b) the value of n.

Ans (b)(ii)(b)

8
Striving/or accuracy & precision
Bukit Batok Secondary Schoof
Mathematics Secondary One Etpress SA22011 Part 2

6 Philip used matchsticks to make the patterns shown below.

·· ---·- '-· ~r

1 )( 1,
".a=.:-:-~
l>;1!Ct:rn I

(a) Draw the next pattern below. [I]

(b) Complete the table below. . [IJ

Pattern number (n) 2 3 4 5


Number of matchsticks used (A} 6 16

(c) Write down a formula, in terms of n, for the number of matchsticks K needed to
make Pattern n. [ 1J

(d) Use the formula from part (c) to find


(i) the number of matchsticks needed to make Pattern 28,

Ans(d)(i)_ _ _ _ _[ I]

(ii) the pattern number that uses a total of201 J matchsticks.

Ans(d)(ii)_ __ __ [ IJ

(e) Is there a pattern that is made up of 340 matchsticks? Explain. [ ll

9
Striving for accuracy & precision
Bukit Batok Si'Condary School
Mathematics Secondwy One Express SA2 2011 Part 2

7 Figure I below shows a cylinder that is half filled with water. The diameter of the
cylinder is 32cm and the length is 0.6 m.

32cm 32cm
Fig I Fig 2

(a) Calculate, in cm3, the volume of water in the cylinder. Leave your answer'in terms
of ff.

Ans (a) _ _ _ ___[2]

(b) Taking 7£ to be 3.142, calculate, in cm2 , the surface area of cylinder in contact with
water.

Ans (b)_ _ __ _,[3]

10
Striving for accuracy & precision
Bukil Balok Secondary School
Mathematics Secondary One Express SA22011 Part 2

7 (c) Water from the cylinder is then poured into a triangular prism. The base of the
triangular prism has length 32 cm and perpendicular height 20 cm as shO\vn in
figure 2 . Taking rr to be 3. I 42, find the height of the water level in the prism in
centimetre.

Ans (c)_ _ _ _ _[2]

II
Striving for accuracy & precision
!Jukit Balok Secondwy School
Mathematics Secondary One Express SA2 2011 Part 2

8 (a) The following frequency table shows a survey done on the speed of l 00 cars on a
certain stretch of road. (3]
--
Speed (v km/h) Number of Mid-value (x)
cars (f)
25 < v $ 35 2

35 < v $45 8

45 < v $ 55 26

55 < v ~ 65 41

65 < v $ 75 18 .
--
75 < v ~ 85 5
-----
Total
~
·-

By completing the above table of values, calculate an estimate of the mean speed
of the cars.

Ans(a)

(b) Pauline needs to travel from Bishan Park to Pasir -Ris within 84 minutes in order
to be punctual for school. The distance between Bishan Park and Pasir Ris is 63
kilometres.
(i) Will she be on time for her lesson if she travels by bus at an average speed
of 50 km/h? Explain your answer with working. [2)

Ans (b)(i)._ _ _ _ _ _ _ _ _ _ _ _ _ _ _ _ _ _ _ __

12
Striving.for accuracy & precision
Bukir Blltok Secondary Schoof
Mathematics Secondary One Erpress SA? 201 I Parr l

8(b) (ii) What time must she leave Bishan Park if she needs to reach Pasir Ris by
07 06? Assume that her travelling time is 84 minutes.

Ans (h)(ii)_ __ _ _[l]

9 Answer the whole of this question on a single sheet of graph paper.

The table below gives some values of x and the corresponding values ofy. where
y = 4-3x .

(a) Find the values ofp and q . [I]

(b) Using a scale of 2 cm to l unit, draw a horizontal x-axis for - 2 s x s 6


Using a scale of2 cm lo I unit, draw a vertical y-axis for - 3 ::; y s 7.
On your axes, plot the points given in the table and join them with a straight line.
[2]

(c) A(l .5, w) is a point on the line y = 4- 3x. Mark the point A on the graph and hence
write down the val uc of w. [ 1]

(d) On the same axes, draw a line L which passes through the points ( 6, I )
and ( -2, 5) [I]

(e) Find the gradient of the line L. [l]

(f) Write down the coordinates of the point where the line y = 4 - 3x meets the
line L [I]

End of Paper

13
Striving /or accuracy & precision
Bukit Balok Secondary School
1\1athematics Secondwy One Express SA2 2011 Part 2

Answer Keys
l(a) 0.349 (b )(i) 13:7 (ii) 12.5% (iii) 82 cents

2(a) 40cm (ii) 105 (b(i) 2 (ii) 2

3(a) 12x, 12x (b) 3 (c) 280cm2

4(a)(i) 3c(5nb-2a) (ii) 2(a+2c)(b-2d) (b)(i) 5+0.5n (ii) $35

5(a) p= 110° q = 17° r = 53° (b(i)) 108° (ii)( a) 162°

(b) 20sides

6(a) (b) 21, 26 (c) k = Sn+I (d)(i) 141 (ii) 402

(c) No, the number of match sticks should end with I or 6.

7(a) 7680cm 3 (b) 3820cm2 (c) 75.4cm

8(a) 46.Jkm/h (b)(i) she takes 75.6mins to travel by bus so she will be on tike for her
lesson. (ii) 05 42

9(a) p = 7,q = -2 (b) (c) w = -0.5 (e) (I) ( 0, 4)


2

14
Striving.for accuracy & precision
I
Class IRegister No I Name

Bukit Merah Secondary School.


End-of-Year Examination 2011
Secondary 1 Express

MATHEMATICS

Paper I 11Oct2011

lh
Candidates answer on the Question Paper.

READ THESE INSTRUCTIONS FIRST

Write your class, register number and name on all the work you hand in.
Write in dark blue or black pen.
You may use a pencil for any diagrams or graphs.
Do not use staples, paper clips, highlighters, glue or correction fluid.

Answer all questions.


The number of marks is given in brackets [ ] at the end of each question or part question.

If working is needed for any question it must be shown in the space below that question.
Omission of essential working wi II result in loss of marks.
The total of the marks for this paper is 50.

You are expected to use a scientific calculator to evaluate explicit numerical expressions.
If the degree of accuracy is not specified in the question, and if the answer is not exact, give the
answer to three significant figures. Give answer in degrees to one decimal place.
For 7r, use either your calculator value or 3.142.

For Examiner's Use

This document consists of JO printed pages including this cover page.


[Tum Over
2
· Answer ALL the questions.
(50 marks)

I. (a) Find the LCM of 18, 36 and 48.


(b) Find the smallest integer n, such that 48n is a perfect square.

Answer :(a) [2]


- -- - - - -
(b)_ _ _ __ _, [l]

2. (a) Express 2 as a recurring decimal.


II

(b) Evaluate 2.83 + 51 correct to 3 significant figures.

Answer :(a)- - -- - - - [1]

(b)_,_ _ _ __ [I]

BMSS Sec I Express End-of-Year Paperl 2011


3
3. The water level of a.Marina Barrage increases by an average of 28% to 176 cm during
the wet season. However, during the dry season the water- level decreases to an
average of 96 cm. Fmd
(a) 1be original water levd before its increase of284Y.,
(b) TbC pen:eotage decrease. coned to 3 signific3nt figures, of the origi~l water
level during the dry season.

A11SJPU :(a) _ __ _ __ _ cm [2]

(b) % [2]
- - - - -- -

4. Factorise complcteJy:
(a) I 5a?b 1 - 33ab'

(b) 8px - 1Say-6py+20ax

·fl]~

(b) _ _ _ ~----- (2]

8MSS Sa: I Express EJ>d-of..Ye.Papal 2011


4

5. The ratio of the age of Lisa to the age of her mother is 1: 4 . The ratio of the age Lisa's
mother to the age of Lisa's grandfather is s: 9 .
If Lisa's grandfather is 72 years ol~ find:
(i) the age of Lisa's mother,
(ii) the age of Lisa,
(iii) ratio ofthe age of Lisa to the age of Lisa's mother to the age of Lisa's
grandfather.

Answer -i i)
- -- - ----- ears ol<l [2]

(ii) _ __ _ _ years old {2]

(iii) _ _~----- (I]

BMSS Sec I Express End-Of- Ycar Papcrl 20 II


5
6. Solve
(a) 3x= - 21

(b) ~r - 10 =_l_r
4 2
(c) 3/-)
- - =7
2t+3

Answer:(a)x= _ _ _ _ __ [1]

(b)r= _ _ _ _ _ [2]

(c)t= _ _ _ _ _ [2)

7. In the diagram, ABCD is a trapezium. BEC is a semicircle with centre o and a radius
of 3 cm. If AB = 12 cm and D c = 5 cm, calculate the following:
(a) the area of trapezium ABCD
(b) the area of the shaded region ABE c D and correct to 1 decimal place (Take
Jr = 3. 142 ).

5cm
D~----;;--, C

E
o.Ej
:o
'(")

A£--~-~-~----'"'--'s··
12 cm

Answer :(a) - - -- - - - - cm i [2]

(b) _ _ __ _ _ _ _ cm 2 [2]

8. In the diagram, AB c is a straight line and cE is a parallel to BF . Given


BMSS Sec I Express End-of-Year Paperl 2011
6

that DB = DC , L ABF = 40° and L BEC= II4 °, calculatethevalueof

(a) p'
(b) q'
(c) r,
D

Answer:(a)p = _ _ _ _ __ (I)

(b)q= _ _ __ [I)

(c)r = (2)

9. a) Using your ruler, compasses, protractor and set squares, construct a [~)

BMSS Sec I Express End-of-Year PapeTI 2011


7
quadrilateral ABCD such that AB = 7 cm, BC= 5.5 cm and LABC = 115° . In
this quadrilateral, CD is parallel to AB and AD is parallel to BC.
(b) Construct the perpendicular bisector of AB, [I)
( c) Construct the angle bisector of angle L.BAD. [I)

Answer:

B 7 cm A

10. Consider the pattern:


BMSS Sec I Express End-of-Year Paperl 2011
8

4= 02 +4
6= 12 +5
10 = 22 +6
16 = 32 +7
a= b2 +8
?I
x= y2 +50

(a) Find the values of a and b.

(b) Hence, by observing the pattern, find the values of x and y.

Answer:(a)a= _ _ b = __ [2]

(b)x= _ _ y = __ [2]

11. The diagram shows the unifonn cross-section of a sofa. EFG is a


BMSS Sec I Express End-of-Year Papc.-1 2011·
9
semicircle, AB= 75 cm, BC= 20 cm, CD= 45 cm, AG = 63 cm and the length of
the sofa is 2.3 m. Find
(a) the area of the cross-section of the sofa,
(b) the volume of the sofa in m 3 .
F

G _:-- --~ --;:: E

63cm
D 45cm
'-'-----__,.....
,c
20cm
A....__ _ _ _ _ ____,B
75cm

Answer :(a) _ _ __ __cm2 [2]

(b) _ _ _ __ cm3 [3]

12. (i) Solve the inequality x+ 2 < 3x - l 6.


(ii) Hence, find the smallest odd number that satisfies the inequality x + 2 < 3x-16

Answer :(12(i))- -- - - - - [2]


( l 2(ii)) [1]

13. The height of 18 boys from Bukit Merah School is shown in the table below.
BMSS Sec I Express End-of-Year Paperl 2011
IO

Hei!!ht (x cm) I I 0 < x :s; 120 120 < x s 130 130 < x S l40 140 < x s 150
Number of
4 x . 5 2
boys

(a) Find the value of x.


(b) The information is to be represented in a histogram. Complete the histogram
below.

Answer :(a) - - - -- - [1 ]

Answer :(b)

,.
10

9 -

rJ)
>. 6
0
.0
0 5 --
Ci>
.0
E
4 - .__
:l
z 3

2 -

j
r . [2]
110 120 130 140 150

Height (cm)

- END OF PAPER -

llMSS Sec I Express End-<>f-Year Paperl 2011


t;:M ~ g I©) Cl) .b-= it
1~) LCM -= .Q x~ Y .;2 ll3X' tt b)x·· .=od-lbb
:. \44-~
l \) aj sI~ 3. -S" CMl-
b) () := s 6) 0·1J3 ,<,(~
..
.2 ~ D- PtS
b) o. 0556 cg f.f) (.2 Q; x> c,
01) I(
~ ~ \3lS"CM
b) 3D-)
0
/o ~s/) rsa) x ==l ·
4: ll) 3ct 62.cs- o.-ri 6) ~) l01tWf w+Jtt = ~ f
b) [:lf-tS (ii,)( l+ :t -<~0 ~ u~~ld -- i t .
s:) AO~~ ~-=====~-~~~~~~~~~~~~~

II) fO~tJ: q
Hi) $ : ).0~ 'S.b
b o.) )(.:: -1
b) r--.:ttO
c) l=--2
1 ()) b[ (,Li2.
b) ~ b~ q CMz. ( ~ ~ fJ ?cm A
Q-a.) ;, f .:~o()
b) Lt 2 Jb~
L-) Lr ::. ,-6 °
2
2
4. Two towns, A and B, are 198 km apart.
(a) Jimmy travelled by car from town A to town B at an average speed of 66
km/h. How long did the journey take? [1]
(b) He travelled back by car from town B to tOwn A in 5 hours 30 mi~utes.
Find his average speed, in kilometres per hour, on the returned journey. [I)

(c) Jimmy left town A at 07 30. He stayed in town B for ~ of an hour.


4
At what time did he arrive back in town A? [2)
(d) Calculate the average speed for the entire trip in km/h. [2]

.. l\

.5. . Mrs Loh keeps a monthly account of four items of expenditure. The figures for a _.
particular month are shown in the table below.

Item Amount{$}
Food 300
Utilities a

Entertainment 180
Transport b

The information is also illustrated using a pie chart.

Transport
Food

Entertainment

(a) How much did Mrs Loh spend on the four items in that month? (2)
(b) Calculate the angle of the sector representing entertainment. [2]
(c) Find the percentage of the amount money spent on transport. [2)

BMSS Sec I Express End-of-Year P2 2011


3
6. A 2-litre bottle of organic peanut oil is sold for $15. A 3.!. _litre bottle of olive oil is
2
sold for $28.
(a) Suppose both types of oil are equally good for cooking, which one is better [2]
value for money?
(b) A bottle of peanut oil was used up in 16 days. Find its consumption rate in [1]
I /day.
(c) If the consumption rate of the bottle of olive oil is the same as that of peanut oil, (1]
how long can it last?

7. Mrs Lim was givensome money to buy fruits for making a fruits hamper. She can
buy x oranges at a price of 3 for $1.00 and she would be left with $0.80.
Alternatively, she can buy (x + 9) apples at $0.20 each and she would be left with
$0.60.
(a) Write down an expression in terms of x for
(i) the amount of money Mrs Lim spent buying oranges in terms of x, [ 1]
(ii) the amount of money Mrs Lim spent buying apples in terms of x. [1]
(b) Hence form an algebraic equation in terms of x and solve for the value of x. [2]
(c) How much money does Mrs Lim have for buying fruits? [2]
(d) During a fund raising campaign, each fruits hamper was sold at $16.70.Find the [2]
minimum number of hampers needed to be sold if at least $5000 is to be raised
during the campaign.

BMSS Sec I Express End-of- Year P2 2011


4

8. The diagram below shows part of a tree trunk in the shape of half a cylinder. It is 4 m
long and its semi-circular end has an arc length of95 cm. (Take ;r = 3.142)

Calculate
(i) the radius of the trunk in metres, [2]
(ii) the volume of the trunk in m3 f2]

(iii) calculate the total surface area of the semi-circular trunk in m 2 , [3]
(iv) the cost of painting the trunk, correct to the nearest cent, given that it r1]
costs 70 cents to paint one square metre.

9. A company offers 2 different pay schemes for annual pay increase to their
employees:

Scheme A: Annual pay rise of 8% of present salary

Scheme B: Annual pay rise of $40 plus 6% of present salary

(a) David is an employee who earns $2400 a month. Which scheme should he [ 3)
choose in order to recei ve more pay in the following year? Jus.tify your
answer.
(b) Steven, another employee finds that both schemes give the same amount' of pay (2J
raise. What is Steven's present salary?

- END OF PAPER -

BMSS Sec I E~ press End·of-Year P2 2011


Maths 1E Paper 2 - 2011- Answer:

1 a 2
Given that x =4, y = 5 and z =-8, find the value of
.
5x-{/zi + .YZ.
x

2
5x - {/zi + yz
x

= 5(4)-~(-8)2 + 2(5)(-
4
8) [ml]

= 20-4-20 ..
= -4 [al] [2]

b s·1mp1
rfy -
x+4
- -5-3x
--
9 6

x+4 5-3x
- - - --
9 6
2(x+4)-3(5-3x)
= [ml]
18
2x+8 - 15+9x
=
18
I lx - 7
::;:; [al] [21
18

[GJ+HH- 1 ~J
c [2]

: ;:; [i- 1~]+[- 1 l] [ml]


-.
= 2l/64 [al)

2 a 180 - 156 [2]


(i) L PRQ = =12° [bl)
2

360- 156-156
(ii) L PSR = = 24° [bl]
2
b (i) Exterior L of polygon = 180 - 156 = 24° [bl] (1)
360 [I)
(ii) No. of sides of polygon= =15 sides
24

3 a
5 165 270
3 l 33
11
54
18
[ml]
HCF=5x3 [2]
[al]
=15
The longest length of each strip is 15 cm.

b 165+270
Total number of strips obtained =
15
= 29 strips [bl] (I]
·'

..

4 a T:zme=-=
198 3 h rs
. [1]
[bl]
66
b 198 [1]
Avg Speed= - =36 km/h [bl]
5.5

c Time spent travelling from A to B = 3 hrs


-
Time spent staying at B = 0.75 hrs
Time spent travelling from B to A = 5.5hrs
Total time spent on round trip back to A
= 3+0.75+5.5 [ml]
= 9.25 hrs = 9 hrs 15 mins

Jimmy arrived back at A = 07 30 hrs+ 9 15 hrs .


{2]
= 16 45 hrs (al]

d Total Distance
Average speed for entire journey=
Total time
396
= -~
(ml)
9.25
= 42.81 km/h [al J [2]

1--

5 a Amount Mrs Loh spend on the four items in that month:

II = 135° ~ $300
I
I = l0 ~ --
135
$300
[2]

= 360° ~ $ 300 x 360 (ml)


135
= $800 [al]

.
BMSS IE EOY P2 201 1
2
b Angle of the sector representing entertainment:

=$300 --+ 135°


=$-1 --+ 135
300
135
=$180 --+ - x 180 [ml]
300 [2]
= 81° [al]

c Percentage of amount money spent on transport:

360-135-54 - 81 [2]
= x lOO [ml]
360
=25% [al)

6 a) umt. pnce
. o f organic
. peanut 01., m
. = -15
2
-
= $7.50 / 1

u mt. pnce · -28


. of o 1·1ve 0 1·1 m=
3.5
= $8/ I
[ml)

Organic peanut oil is value for money. [al] (2)


--
i' b)
I Consumption rate for organic peanut oil = l:__
16

l c)
Co~sumption rate for olive oil =_!_ I I day
8
I
= - I / day
8
[blJ [11

3.5
= --
A bottle of 3.5 1 of olive oil will last for 0.125
= 28 days [bl]
[ 1]
----
1- -·

- -
7 a)i)
Amount of money spent on buying oranges = ( x )( j)
:::: $ ~ [bl ] ( I]
3
a)ii) Amount of money spent on buying
apples = ( x+ 9)(0.2)
= $(0.2x+ I.8) [bl] [ lJ

BMSS IE EOY P2 201 1


3
b) Amount of money spent on buying oranges +0.80= Amount of money
spent on buying apples+0.60
x 8 2x 18 6
- +-=- + - + - [ml]
3 IO IO IO IO
x 2x I8 6 8
---=-+---
3 10 10 10 10
-··-· 2 8 [2]
:
- x =- [al]
15 5
x=l2

-··- c)
Amount Mrs Lim has = = ( x )( ~) + 0 .8

= •: +0.8 {ml]

1---1--- -- 1 - - - - - - - - - =_ $_4_.8_o_ _ _ _ _ _ _[_a1_] _ __ _ _+--'.r._


2L _
d) Minimum number of hampers needed to be sold:

Let the number of hampers to be sold bey

16.7y > 5000

I 5000
y > 16.7
[ml)

y > 299.41

Minimum no. of hamper to be sold= 300 [al] [2]

Or

5000
= [ml]
16.7
=299.4 1

Minimum No. of hampers need to be sold= 300 [b I]


1-·--·-- - ----·--+-------~-------------------+---I
~·--f-----+-------------------------+---l

8 i) Radius of the trunk in metres: [2]


l
- x 2 x;rx r = 0.95 [ml]
2
0.95
r = --
3. 142
r = 0.30236m
r "" 0.302m [al]

BMSS IE EOY P2 20 11 4
ii) I [2]
Volumeof trunk= = - x 7r x 0.302362 x 4 (ml)
2
=0.57449
: : : 0.574m3 {al]
iii) Total Surface Area:

= ( 4x2x0.30236) + 2(~x3.142x0.302362 )+(0.95x4) fm2)

2
=6.506lm [al]

..
iv) Cost of painting the trunk = 6.5061x70 [I]
=455.42
::::::455cents [bl]

9 (a) Under scheme A, he will have a pay rise of [3)


$2400 x 8/100 = $192 [Ml]

Under scheme B, he will have a pay rise of


$2400 x 6/l 00 + $40 = $184 {Ml]

David should choose scheme A. [Al)


·-
(b) Let y be steven's present salary. [2J

8% of y = 6% of y + 40
2 3
- y = -y+40 [MJ]
I 25 50
I I
- y = 40
I 50
y = 2000 [A I]

ll Steven's present salary is $2000


SIGLAP SECONDARY SCHOOL

Mathemat ics 4016/1


End Of Year Examination 201 1
Secondary One Express

)
Name: ( ) ......Date : 11 Oct 20 11

Class: 1E( Duration : 1 h 30 mins

READ l HESE INSTRUCTIO NS FIRST

\iVt1le yovr name and index nu1nber on all tho work y\lv hand in
Write in dark ~e °' blacl< pen on bolh sides of lhc pope1
You may use a pe11C1I t0< any dia9rarns °'
o•~phs .
Do no! u ~o slaples. pllper chps, highlightcrt., glue o, correc1ion ttuie1

ArlS,vcr all questions in lhis papt:r


fhP. number of marks is givcn1n brackcls ( ) at the end ol each Quesuon Of pa1t q11es,1on

Ornission of essential walking will result in toss ol r11arks .


The total number ol matks t0< tl1is paper is 60.

f h~ use of an etecuonic calcula1or is expec1ed. whCJO: appropua1e


If lhc d egree of accuracy is no1 specilic in lhe q uestion, aud it the answer is not exact. givtJ 1t1c
ans..vcr to three signlficant figures Give answets in deg.tees to one decimal place
F0t j[ , use either your ~"'lfeulator value or 3 142. vntess the ques1ion reQu1rcs the a1's'ver •n term ~ nt n

You are remin'1ed of the need tor c lear prcscn1ation In your answers

l
Score

I
r - -Total marks 60 Parent's Date

L __ Signatu re
_ _j__
_ _ ~ turn the p:iges until you are told to do s_o _ _
This paper consists ol .!.!_printed pages and ! blank page .
. . ."8-u,,.fJ<.'"'9 0,4 'l~e fie<!.( 'J,, &uc:, Sit)laftia..,, . ...

(T um Over
2

An~wc r oil questions.


I. (3) Find. m its simple" lnnn, the f1ac11on which is e'ac1 ly h;ilf,.ay
7 8
between and
9 ')

{h) W11hou1 using cakulator. cvalu!lle ( 2 ~ + ~).;. ( - 4 ~).


Sho'"' your Vl'orkings c1early. Give your answer as :l frncli(ln in ils shnplesl

lorm

Ans: (a) _ __ _ __ _ ~ [ IJ

(b) - - -,.--- ._ 121

2. F"alua1 c each tlt the following. g1v111g your answcn; corrccl 10 3 ~1gn1ficant figures.

(a) 12352 - 1
5
x 8 ·92
11 ( l.1n)'
3;i: 26.3
(b)
Ji 2.65 : o.oi

l\ns : (a)._ __ _ __ __ 111


(b)_ - - - --
J

3. (al If ooe f1hh of2520 i< the sa!lle J ' 2' x3• x7

\vhat ~c1rc I he values of .f • y antJ : '!

lb) Hence or othe1w1\C. lind the smallest v3lue ol k whe<c 7' x3' x7 ' x k 1s a

Aos. (a)x - __ llJ


y= 111
I I]
(h) - -- [ II

•I. Con"dcr the lollowing numbers

9
J2. 0.78, ()
1

\V111c do\\"ll 1hc

(h) irrallonal numbc"

Ans: (a} _ _ _ 11 I
(b) _ _ f 11

'
r\a~yita/MJ1hs/I Exp1FOY/Plf201 I
4

5. A eye hst travels IO.& km in I hotlJ . Find its average speed in

{o) rn/min.

ib) on/s.

Ans : (a) _ _ _ ___m/min [I J

ib) _ _____ cm/s I I J

6. Given that C = F ' + .!.. pn, find the value of n whrn F ~ 5. 1> :~ 0.4, :ind C '" 65 .
2

An<: _ _ _ _ __ _ _ _ __ (2)

7. FactoriLe the (ollo,ving cornpletely

(a) 6rs 2·1st .

(h) 9gh+4~'1 - 15 - Jg .

J\11': (a) 111

(l>) _ _ _ _ (2)

Nasyita/i'v1aths/ I Exp/EOY1T'li201 I
8. (a) Simplify 213p - 2!p-2qlJ .

(b) Sul:>trart the >um of 5ab I a - .l/J and 2<> - 4b + ba from JO- ?ab.

Ans: (a) _ ___ _ _ _ _ _ 121

(b) _ (3)

'I. Eugene ha. 2x b1x>ks.


liafiz ha• 4 morr l">OOks than Et> gr nt
f)a1Ten ha~ half ns 0''11'1lY books a!- H1!1i1

(a) Wnte uuwn an exp1es.<mn, in 1rm1s of A. for lhc num~I or books Chai Darren hns

(h) 1lie l<>IJI numbet ortxx>ks is 121 Find the number or boo~s Chai Halil. ha~

Ans: (J) _ II J
(bl ._ _ _ _ _ !_ 121
6

JO. In 1ht figure, PQ II HS, LOAM - , + I0° and LABR - 3 r - 10• Find the value of x.

Q
+ 10'

-- _----
J(

~-
1 --£_ -
A - - --- ..
- -----------N
R
p

Ans: _ _ _ _ 121

II . liovcn 1hat AB = 1\C. /AC/) i .t .. . L BCA ~ 3x' aml L/JAC ~ >"',


calculate the ''alue of

(a) X.

(b) ).

Ans · (a) _ 121

(hi _ _ _ _ _ _ l lJ

Nas)na/:>101hvl 1"p/EOY/P1!20l I
7

J 2. Consider the nu111bcr p;mrm ~low .

.s. 11. 17, 23. 29. )5.4 1...


(a) \Vri1c dl)\Vn 1hc 10111 lC1n1of1hc paucn1.

(h) Write down then'" 1nm tlf 1he pntctrn.

-
,\ns · (a) _ _ _ __ _ _ II)

(h) -=._ _ _ __ 111

13 (a} During the G1ea1 Sing;iporc Sn le. the p1i,· es of Mokia handphoncs we1c
sold al 30% dis.:ot1111. William bought the phone hcfmc 1hr snl• sinned I low
much did he pay for 11 1f 1he price of 1he phone dunng rhc <al• wns S285"
<;il'f' your answ~r to lhr nearest t€'nts..

1b\ :..1r Aqeel bough1 a condominium tn 2001 for $51XIOOO H~ ~Id i11n lOH) for

$650 OOo Find the pcn.:cn1agc inc1ease in 1h~ price of tht· l ondo1ninJLun

_ ___ 121

(h) _ _ , _ _ _ % [21

Nnsy1ia/Ma1hs/f Exp/EOY/1'1120 I I
8

14. /\ s11n1 of money was divided bctv.-·ccn A. JJ and C in the ratio 2 : 3 : '1.
If instead. this money had been divided c<1ually be tween them, A would have
Jcccived an ex1ra .$20 Vii'h(ll v1a~ the 101al stun of rnoncy?

Ans:$.·_-- -- - - - - - !21

·------ - -- - -- - -- -- - - - - -
15. In a survey. 48 children were asked how they travelled to school.
The resuhs of the survey are shown in the bard1art.

30

Number 24
of
C111hlrcn 18

,
B11s
12

6 ...
0 .... .. ,
Car

(a) Express the toial numbe.r of children who 1ravc)_lcd by bus or cycled as a
frac1ion of 1he rot:ll number of children.
Give your ans"'·er in i1s h.>\vcst tcnus.
(b) The same infonn"tion is 10 be shown in a pie chan.
hnd the ani;le which represents the childien who uavellcd by car.
(c) Calc11la1e the percentage of children who travelle.d by bus to school.

Ans: (a) _ _ __ _ _ __ _ 111

(b) - -- - - -- - - - " !21

(C) - - · -- ·- ·· - - _ _% 1ll

N;isy1t;i!Maths/I fap/EOY WI/ 20 I I


'J

I b.
"
4

B
.l ><

A;.< 2

, '.! - I 0 2 3
• .t

_,
.....
-2 ><
(

(;i) Write down the coordi11:t1cs of points A and C.

(b) l'lo11hc po1n1 1)(-2,0) and label il ~~aily. loin all 1hc poini- and wri1~ dow111he
name or !he quadrilatcml ABCD.

(c) l'ind the art•n or 1he <111ndrilaicrnl AllCD.

(d) Find the 1:md1en1 of AR

1\11s : (a) 1\ ( JI I
11.J
(b) _ _ 121
k) _ _ __ .squa1e u1111Sl21

(d) l'.!J
. 10

17. --___"-.../_____
.v · ""

Samo made a Valentine's Day card which is made from a square and two

semi-circle~ I( the a"'a of 11~ squnn: 1< I 00 cm1, calculare

(a) the total are;i of the e:inJ, ""

(b) tho perirnerer of 1he rnrd.


(Take n to be 3. 14)

Ans : (a) _ _ _ _ _ _ crn' [2)


(b) _ _ __ _ cm 121

N:l>)'ita/Ma1 hs/I Ex piEOY iP.l/2U I I


II

I~

1.5 rn

LJJ 3.5 II\

i\ swim1nir11: pool measures 3.5 m by 5 111 and the heig ht of the wall 1s 15 m.

(a) Calculate the volume of the swimming pwl

(b) Uncle fared has to 1>3uu the inner surface of the pool. Given that I litre of
2
pain1 can cover 5 nl , ho\v rno:iny lit.res of pain1 docs he rc<p1ire '?

(b) The paint is sold 1n 4- liire containers. Vind the lea." numbe1 or containers of
paini that Uncle Jared needs to buy.

An.<· fol _ _ __, _ _ _ _ mJ(I)

{b) __ - _ litres 131

(c\ _ _ Ill

- -- -- - - ~- -
-- Er1d uf /'npu --
l EXPRESS EOY MATHS ANSW ER KEYS

5
,-!
- -- - -- - - - - - - -- - -
·1
l(a) -
f, Bl I
( I 5\ (
(b) 1 - 2 ., + - l o- - 4:;
1')
I. 4 3) , .>

Ml

-~--- --
5
= -
-- --26 -
·~'>
7(a) I 2 ·-'-'· -
1·-5 x -8.92- , ! Bl
11 (1.357)'

= 5.31

3r. - 26.3
(b) f12.65 ~ 0.02
Bl

= - 0 ,671

- - - - - -·- -- ·- -·

I
. 3 (a) 5
.! x 2520 = 504
504 ' 2 3 x3 1 x?
Bl for each correct answer.
Therefore • .x = 3,y = 2, z = I
(b) ror·a perft•ct cube,

50<1x k = 2; )< 31 x 7 x 3 x 7 x 7
:.k 3x 7 x 7
147 Bl

9 . ..
4 (<1) - : 0 v'M
!-·- --
' '
.) '
- -- · -- -' RJ_ _ __ _ _ _ J
r·(b) J2 _ _
· -- IBl __ _J
l
- · -·- - --·-· ····---- T-
I l
S (a) ·180 m/mln Bl
I
{b) 300 emfs Bl

•< . C' I.;


. +-I pn
2

I
65 = 25 + (0.4){n)
2
40 = 0.211 Ml
I t1 = 200 Al

1-·· -- ·--··-·-
I
! 7 (a) - 6rs - 24st Bl

I = .. 6s(r+ 41) I
II (b) 9gh+4s1i - 1s - 3.i:

I - 9h(g+5) - 3(5+g)
= = (g + 5 )( 9 lz - 3) Ml

= 3 ( g I 5 )( 3 h - I) Al

8 (a) 2f3p - 2(p - 2q)J

= 2[3p 2 p ·l 4q]
~ 6p - 4p+8q Ml
~' 2p+~q
Al

(b) Snb + n .\b. t· 2a - 4b + ha

'· 6al>+ Ja ·· 7b

- -- · -···- -
...:.._
I0 - 7ab (6ab+3a - 7b)

I 0- 7ab 6nb - 3a + 7b
=
= 10 13ab- 3a; 7b Ml

Al

9(a) r + 2 Bl

2.x + 2x i 4 I .l'. ~ 2 121


Sx 16 121
(b) Ml
5x 115
x =2>
Therefore, Hafiz has 2(23) • 4 =50 books Al

x + I0 I 3x - 30 180
4,t 20 - 180
10. Ml
~x =200
.l - so Al

I 7,\ + Jx = 180 Ml
I 11. (a) I 0 r = 180
Al
r ' 18

3(18) t 3(18)+y = l80


(b) y 180- 54 54 Bl
y 72

12.(.l) 59 Bl

(b) 611 - I 81

t 13 (al 70% ···· S285


285)(100 Al
100% ---- 70
: $407. 10

Ml
(IJ) $650 000 $500 000 = $150 000

I 50000 x l OO% ~ 30%


500000 Al

M I unit = $20 Ml

9 units= $180 Al

7
" Bl
IS (ij)
8

(b) 48 children----- 360°


Ml

360
- )<
6
6 children -- - 48
= 45~ Al
(c) ,!_ (2 +5) x 3 = I0.5 square units
2

( d) -
~
~- -'>. -- I
1- (-2) 3

1 7 (a) Area of two semi circles= 3.l4 x S x !> Ml

- 78.S cm'

Total area • 100 + 78.S = l 78.5 cm' Al

(b) Circumference o f two semi circles

= J .14 x 10

= 31 .4 nn

Perimeter = 31.4 t 10 + 10 • SIA crn

18 (a) Vol ~ l.S x o; x 3.5 = 26.25 m 3 Bl

(b) Surface ~rea of the pool

- (2 x 1.5 x 5 ) • (2 x 1.5 x 3.5) + { 5 x 3.5} Ml

= 43 m'

5 m 2 ····- 1 litre Ml

43 m' ···· 8.6 htres Al

(t) 3 tin\ Bl
L
S/GLAP SECONDARY SCHOOL
Go forth with wisdom and courage

Mathematics 4016/2

End-of-Year Examination 2011

Secondary One Express

Date: 14 Oct 2011


Class : _ _ _ _ _ Duration : 1 h 30 min

AdCticmal Ma1e1i.,.11s: A1\Sw~r Papers


Plain Paper ( 1sheet}
G"1oh Paoer ( lsheel)

READ TH ESE INSTRUCTIONS

W rilc your name and index number on all the work you hand in.
Wrile in dark blue or black pen on both sides of the paper.
Do not use staples. paper clips, highlighters. glue or correction fluid.

Answer all questions.


Write your answers on the answer space provided.
Give non-exact numerical answers correct to 3 significant figures. or 1 decimal place in the
case of angles in degrees, unless a ditrerent level ol accuracy is specified in the question.
At lhe end or the examination. fasten all your work logether securely.

The number or marks is given in brackels [ ] at the end of each question or part question.
The lolal number of marks for lhis paper is 60.
The use of an electronic (',alculator is expected. where appropriate.
You are reminded e>f the need for clear presentation in your answers.

/,,/I ---. ,
Scor e / __________ !
/ ' 60 ' Parent's Signature I Date
_ _ _ _L l I

· D o not turn the pages until you are__


to_l_d_t_o_d_o_ s_o_ _ _ __ _
Th is paper consists of~ printed pages and !!.blank page
. ... .. (!:.,~Hf O«t tit,; i?ca '" &""'"! S<9fafJ.iw<. . . . ..
2

Answer ALL questions.

I. (a) Bus Service /\ leaves lhc Pas ir Ris Interchange every 3 minulcs. Flu~ Service
13 leaves the Pa~ir Ris Interchange every:'> minutes and Bus Service C Je~vcs

the Pi~~ir Ris Interchange every 8 minutes. If all the.three bus services first
leave the bus interchange at 06 00, at what time would the th ree buses next
leave the interchange together? [3 J

(b) Mrs Tan has3 pieces of ribbons measuring 120cm, 192 cm and 252 cm
respectively. She wants to cut the ribbons into smaller pieces of equa l
lengths with no remainders.
(i) Find the greatest possible length of each of the small er pieces of
ribbons. [2}
(ii) How many such small er pi.::ccs of ribbons can she get rrorn the
original 3 pieces of ribhons? [I]

2. Given that x - I , y = 4 and : 1. find the value of the following.

(a} (.ry) ' - 8xz (2J

(b)
..ty _ fZ + XZ
[2)
z x y

3. Simplify the following:

(a) • ~X - 2x - Y [21
4 5
2(m - 1) "1(2m I)
(b) _:__ __.:. + - - (2)
3 ,,

Ahou WW/J:.MI'2/HO Y!WI I


3

4. Solve lhc following equation~:

(a) 2x - 19 3(4x - 7))=36 (JI


.3x 2
lb) 5+ - = .\ 13]
•1

----- - -- - - -- --
:'. A 1rain traveJctl a distance of625 km in 5 hours anti then without stopping.
traveled a further distance of 240 km at an average speed of I 00 km/hr.

Calq1late:
(a) the average speed for the 1st part of the journey. [ JJ

-(b)
(c)
the lime taken, in hours and minutes, for the second part of the journey.
the average speed for the whole journey.
(21
[2]

-- - - -- - -------·- --------- --------- -


6. In the diagram below, PQR is a straight line and 'IQ is the angle bisector of / PQS .

T
s

(llz - 28)"
p Q R

Fiod the values of

(a) x, [2]

(b) )' anti (2j

(c) z. [2)

KhooWW!/:Ml'HF.01'' '(/J /
4

7. PQR is a triangle Ill which l'Q = (5.r - 2) cm, QR= (6x + I) cm and PR -

rn x 4)
! cm.

(Sx - 2) on Ux+4) cm

Q R
(6x+ I) cm

(a) Given that the perimeter of triangle PQR is 53<m, fom1 an equation in tenns
of x. - (I]
(b) Find the value ofx. (21
(c) Using (b), find the length of the shortest side of the triangle. fI I

- --------~

X. On a piece of blank paper. con~truct a quadrilateral ABCD such that the base

AB = 9 cm and / B11D =85° . AIJ 7 cm . L !IBC =75° and ('{) 6.8 cm [2)

(a) Measure and write down the length of BC. [IJ

(b) On Yl>ur diagram. draw:

(i) the pcrpcndiculur bis<:ctor of AB. (I)

(ii) the angle bisector o f LAVC. [I)

{c) The perpendicular b1~ector and the angle bisector meet at X. Measure mid
write down the lenkth of AX. [I j

AhnoWWi l::\4PJ!F:OYl ?IJl I


5

9.

31.5 cm

The cross sectional area of1hc prism shown above is a quadrant of a c ircle of radius
22
6 cm and height 3 1.5 cm. Taking Tr = , find
7
(a) the area of the quadrJnt , 111
(b) the volume of th e prism . 111
-(c) the total surface area of the prism. f3l

I0. An swer rhe whole of tJ1is q ucstion on a sheet of graph paper·.

The given 1able of values is for y = 2( x - I) .

(a) Find the values of a and b. [7)

(b) Using a scale o f2 c111 to 1cprcscnl 1 unit on the x-axis and I cm to represent I
unit on the y-ax is . draw the graph of y - 2(x - 1) for 1hc va lues ofx in the

range - 3 S x < 3 . [21

(c) \J~mg )'Our graph,


(J) find the value of T when y - 5. 111
(ii) cakulah.: the grmhen l of the line. [I )

Klioo~fll'-'F..\fl' ?/l;,'O}'·JIJ/ I
6

Il. Th~ pie chart below shows how Hannah spends her allowance of $240 in a
particular rlJO!lth.

(a) Ca lculate:
(i) the nmount spent on food . II I
(ii) the IT-action of the amount spent on shopping. (1)

(iii) the pl-TCcntagc of the amount spent on transport. fI)


(b) If she managed to keep S52 as savings, calculate the angle o f the scctl.lr
repre.~enting saving.s. (2)

(c) Given that she spent twice the amount on movies than on storybooh,
calculate the amount she spent on mov.ics. [3)

·· .....

- £11J of Poper:_

Alwoll'H'il-.M/'liFOJ'i_'O/ I
7

Sec I "'rn:ss Paper 2 Solutions

(a) 3 !3 5 8
5Ll~Ll
2L 1 ~.L..!
2LL.L4
2LL.L2
LL.Ll [MIJ

LCl\.1 = Jx5x2x2x2
- JS x8
~ 120mins
~ 2 hrs IMll
0600 -< 2 hrs ~ 0800 h13 (8:00 am)I A I J

(bi(•) 21120 192.252


21 60 96 126
3L. 39 48 63
L 10 .~..._ll fM I )
-
I Ocm , 16 cm .21 cm (Al}

(ii) JICF ~ 2x2x3


= 12 p ierPs (All

(a) (xy )' - 8.rz = (- 4)2 8( l)Cl)


= 16 - ( - 24)
= 16 ~ 2·1 (Ml (
= 40 [Al)

( b)

=9.!...!. I/\ I I
12

T +y 2X V
(a)
4 5
5(x + y ) 4( 2-'__~
20

A"ho"WW/LWP2;EOY!21JI I
8

5x t 5 y - 8x+4y
- -- IMlJ
20

- 3x +9y (~y - 3x)


= 20 20
(A l)

2(m - I) 3(2111 - 3)
(b) I
3 4
2{4){m - I) 3(3X2m - 3)
= -- +
12 12
2(4)(m - I} 0 (3}(2m - .3}
=
12
8(m - 1) + 9(2m --~
= 12
8m - 8 +1 8m 27
= IMI)
12
26m - 35
= 12
[All

,, (a\ 2x - [9 - 3(4.< 7)) = :>6


h [9 - 12.x +2 1] 36
2x - [30 - 12x l = 36 !Ml)
2x - 30 Hh = 36
14r =66[M I)
66
x=
14
33
=7
5
~ 4- IAI ]
7

)x 2
(b) 5+ - - = I
4
) 1- 2
,, .r - 5 !Mil

1, - 2 4(.x - 5)
>x - 2 = 4x - 20
\.\ - 4 X c ·-20 I ;> )Ml) ,
- x - 1&
t I~

!.: lwol·VW!F::\41'! / F:()Y/1011


9

s. (•J AvcrJg< Spccd • 625 + S


= 125 km/Irr [MI

(b) Time Taken = 21\0 + I00 [Ml)

2.4 lrrs ( 2 ~ hrs) (Al]

(c) Oisrane< - 625 + 240


= 8"5 km

Tm>e Taken = 5 + 2.4


74 hrs

864
J\verage ~ed • (M J1
- 7.4
• I 1689km//,r (116 ~iA?n/hr) [AIJ

1. (a) 2x~ 17 = 69°


2x = 52° (Ml)
x = 26° (Al)

(bl 5y 24 69°
5y = ?~u )Mii

y = 18.6° (1s~·) I,\ I I

(c) 4z - 28 09°
l\z =97" IMll
C =?4 l5° (24 I~ o) (All

'/ (al (5x - 2)+(2x+ 4) ~(6x d) = 53 (Al]


2
3
(b) 5.l - 2 + - .l + 4 t 6.ro I = !\~
2
1 2~x+3 = 53
7
I
12 )
.\ = .50 (Ml]
L

x 50 -:· 12 I •
2
x 4 f t\ll

l\luH1WWiEMPl 'r:<)) ' /(!/I


JO

(c) (5x - 2) = 20 - 2 = 18 cm
3
(- x+4) = 6+4 = 10 cm
2
(6x I I)= 24 ! l = 25 <:m

leng1li cif shoni:s1 si<if' is I 0 cn1 !Al I

8. (•) 1.cngtb of BC ~ 6 cm fA II

(c) Le11g1h of AX ' 4 8 cm IAI)

l
9. (a) Arca of quadrant ~ x irri
2
l 22
= - x - x 6x(1 [Ml)
2 7
2
...
- 28 r:m' (28.29 cm 1 ) IAlJ
7

2
(b) Volume of p1ism ~ 28 x 31.5
7
1
"' 89 1 C/11 JAi]

(c) Total Surfac.e 1\rca

(31.5><6)+(31.5 >< 6}+28 -2 + 28 2 ! (I· x ·-


22 xl2x31.5) JM!)
., 7 4 7

= 189 -1189 1- 28
2
~ 28 2 ·l 297 [Mil
7 7
= 731 -
4
C!tl
, (731.57 cm') IAll
7
10. (;o) (I = - 12 fAl)
. 0
" fA l)

(c}(i) x = 2.65 (Al)


(ii) gradient = 3 rAJJ

11 . (")(i) -105 x 24 0
360
=$70 JAi)

60 I
(I j) --- = IAI)
360 6

45
(iii) x l00%
360

KhooWWIF:MP2/EOY/20! !
II

= 12.5% [Al]

·1
(h) 2::.. x 360° !Ml)
240

!All
(c) L rcprcsc1uing fnovic:~ anc1 $I01ybook.s

= 720

Lei :x be lhc L rcprcscnling storybooks


Let / ..t be the L represc;nting movies

2x + x = 72°
3x "' 72°
x - 72° ·'"3
x = 24"

2x = 2 x 24 = 48°
Arnounl spent on n1ovics
48
"' - - x $240 [Ml)
360
= $32 [Al]

Qucstioo 8:

!
.~

·.
• )~
,. .
:1.
12

I mark :cunt"t:~ drn1tnsions)


f I mark .-oort'Ct cliagrom)

Question JO


1
2

FQr
·aminer's 3 A stack of two-dollar notes, worth $6000 is 12 cm thick. Find the thickness ora For
U:;e Exan1iner'.
two-dollar note Use

(a) in centimetres, giving your answt:'r as H decimal.


(b) 111 metres, giving your answer in standard fonn.

-
Answer (a) ....... ...... ..........cm JI]

(b) .............~ ........m 121


4 (a) Find the smallest possible value of w if it leaves a rcniaindcr of one when
divided by 2, 3 or 5.

(b) Expressed as the product of prime factors,


18 = 2x3 2 and 45 =3 1 x 5.
Use the results to find
(i) the smallest integer, k, such that J 8k is a perfect cube,
(ii) the highest common factor of 18 and 45.

Answer (a).......... .... .... ... .. .. 121

(h)(i)...... .. . ..... . .. . .. .. 121

(h)(ii).. .. .. .. .. ... . .. . .. . . ll l

2011 !'ear End Ernm


3

s (a) Expand and snnplify 2b(a t b) ·• a.(a - b).


F"' . 3/ - g
~am"rer ·s (h) Express - - - - 1 as a single fraction in its simplest fonn .
Uso f +2g

Answer (a)...... .. ... .... .......... [2)

(b)........ .. ... .. .. ...... .. 131

6 (:1) Factorise co mpl etely 3a (.\ - y ) - (y - x).

(h) '" Ivet hc equation


.-.n .
a
II
I )
=J .
1

Ans wer (a) ....................... .. [21

(ll) ............... .... ....... [2)


4

For (a) Given that ,r: y = 5: 6 and y: w = 8: I I, find x: y: w. I For


0Jm1r1er's ·1 Exonlin<n
Use Us.;
(b) Vinegar and water are mixed in the ratio I : 4 by volume. If the volume of the
solution is 900 cml, i
(i) find the volume of vinegar in the solution,
(ii) how much vinegar must be added to the solution so that the ratio of
vinegar to water in the solution becomes I : 1 hy volume?

Answer (a).. ...... ........ ......... 121

(b)(i) ........ ... ...... cm 1 111


(h)(ii) ...... ........ : . . cm 1 12]

fon;ong Kllll)ng Sec School 2011 Year E>ul E.rnm


5

For
:xa11110(1r :;
18 (~) Find the 'mallest integer y that satisfies the inequality .6 y > - 54 . /•'or
Exont1ntJr'
Us. 1 2 Ust<
(h)(i) Solvc - x l "-7 .
s
I 3
(ii) I Jenee, find the largest possible value ofx if x is a prime number.

..

·A nswer (a)..... ......... ........... 121



(b)(i).... . .. . . . . ... .. . .. . . . 121

(h)(ii) ...... .. . ... . .... ... .. 111


~I\;

~(}/I l'rnr End F ., ·0111


6

r"' 9 In the figure, RSllPQ, LSRA 30", /RAQ = (7x - 40)0 and For
&-
u... / AQP = (2 x+ 25) 0 • Find the value of x. Uso

R...,, __ )11r - .,._ S


·--......_
·-- A
fl• - .JI»;:..·.
p

Answer • ............... .. .... ...... 141

limi<>n.1: Kat<-11g S.:c S< hwl


7

JO In the chagram, ABCD is a quadrilateral in whi ch AB: AD , RC =- CD. LRAD. 50° FM


£xomrnet'.
and L l{)C = 140 °. U"'1

(a) Find LRCD.


(h) Wha t is the special name given to quadrilateral ABCD'?

LI

• /111.vwer {a) ................ : .... ." [21

{h)............. ......... Ill

I t1111ong Ka1ong ::,·ec .<;<'hoof


8

For 11 The following pie cha rt shows the brand of mobile phones owned by a group of For
·aminer's E:itnn>in1
Use
teenagers . Use

/_ ... - ·--~

/
/ Nokia
LG

( 118° Samsung )
'l /
\ ./
,/
\ ,,,.,,.,"'" Sony
~ Ericsso~
----- -· ___ ...---

(a) Write down the fraction of teenagers who own a Samsung mobile phone.
(b) ·nic number of teenagers who own a Sony Ericsson mobile phone is three
times more than the number of teenagers who own a LG phone. Calculate the "
angle of the sector which represents the number of teenagers who own a Sony
Ericsson mobile phone.
(c) Calculate the total number of teenagers in the group if the number of
teenagers who own a Nokia mobile phone is 24 more than ~10se who own a
Sony Ericsson mobile phone.

Answer(a) .. ..................... JI]

(b). . .. . .. .. . ... . ... . .. 13\

(t) ... ....... ·············· 121

Jinif<>ng f.:u1ong S1.'C School :?O11 f'ear 1:;,ui £ .\an1


9

12 Constnict a quadrilateral PQRS where PQ = 6.5 cm, QR = 4.8 cm, RS = 8.5 cm, For
?-~iuuiner 's f;xiJmir>e-r.
u.w. L.PQR -· 75°and L.QRS- 98°.
Use
On the same diagram, constrncr
(a)(i) the perpendicular l>is(!{:tor of PS.
(ii) lhe angle l>isec101 of L. PSR.

(b) Jvlark the poiol X where these two bisectors meet. Measure the length of PX.

, /nswer (b) l'X = ........ . .......cm 151

1i11fjr>n.1!. Katong Sec !:>'chooJ 201 I y,,ar Fml F.wm


1
2

r., 4 (a) Find the smallest possiole value of w if 1t leaves a rcmaind(;r o f I when Fur
~amjrlef"s
.X,f1(11i06f's
divided by 2, 3 or 5. vse
USO

(b) F.xprelised a~ the product of prim.: factors.


18 . 2x3' and 45 = 31 x5.
u~e the results to find
(i) the smallest integer, k. such that I Rk is a perfoct cuhe,
(ii) TI1c highest common factor o f 18 and 45.

(a) LCM of 2,3,5 is 30 MI


w = 30 ~I
- 31 i\ I

1
(b)(i) 18.k 2x3 x2 1 x3 Ml
k - 12 Al
(b)(ii) HCF of I 8 and 45 = 9 AI

Answer (a)......................... (2)

(b)(i) ... .. ...... ... ....... 121

(b)(ii)........... . ......... ll I

(a) Expand and simplify 2b(n ·I h) n(n - h).


3f . g . ..1 1·rnc11on
l as a smwc . m . llS . Ies t r·orm .
. snnp
(b) Exprcss - - -
I +2x
2b(111 b) - a(a - b).~2ab+2b
1 1
(a) - 11 +ab Ml
2 2
=3ab+2b - a Al

Jf - g (· 3/ - g - (f+ 2g) MI single fracllon


(b)
fl 7.g f I 2g
~ 3/ - g.:_ / 2g MI- ex pand correctly
f +2g
2/ 3g Al
.r + 2g

.'ll/S IH"f" (a).......... ... ... ... .. . ... (2(

(h).. .. . ......... .. . ..... ... Pl


3

For 6 (a) Factorise completely 3o(x-y) - (y-x). I-or


,..}(lf/11lltC.' 's £.xauuner
u,,, (b) Solve the equation ° 12.. tlse
a+J 4

a) 311(x-y) - (y ·- x) - Ja(x - y) ~(x y) Ml (chaugingrcrm."n2'" brocke•)


~ (x - y)(3a+ I) Al

a I
(b) =l ·-
a+ 3 4
a 5
-- =
a+3 4
4a ~ 5(a ~ 3) M 1 - cross multiply
a =-15 Al

Answer (a) ................... ...... (2)

(b) .......................... 12]

7 (a) Given that x: y ~ 5: 6 and y: w = 8: 11, find x: y: w.

(h) Vinegar and water are mixed in the ratio 1: 4 by volume. If the volume of the
solution is 900 cm1,

(i) find the volume of vinegar in the solution.


(ii) how much vinegar must be added to the solution so that the ratio of
vi11cgar to water in the solu tion becomes I : 3 hy volume"

(a) x:y=5:6 y:w= Rx 3:1 lx3 .\ ·t I (rnulroply r•llo w11h t CM o l


= 5x4:6x4 - 24 :33 original values of YI
= 20: 24
' : y : w ~ 20: 24: 33 A I
1
(b) (i)Yol of vinegar - 900 x
5
~ 180 cm; AI

(b)(11) Let x cm'be the vol of vinegar added.


Vol of water - 720 cmJ

180+ '
Ml
720 3
540 I 3~ =
72()
x - 60 cm 3 A 1

1foSll'l'l'°(a).................... ..... (2)

(h)(i) ........... .. .... cm 3 111


. 1
(IJ)(1) ................. Clll 121
4

f-Ot 8 (a) Find the smallest integer y that satisfies the inequality 6y > -54. For
~amirier'.-; ExamifK
Us&
(b)(i) Solve ~x -~ x > - 7. Use

(ii) Hence, find the largest possihle value orx if xis a prime number.

(a) 6y > 54 Al
y> ··9 Ml

J 2 7x < l 05 MI ----changing sign


(b)(i) - x "· - x > ·- 7
) 3 x< 15 Al
_:t(3) 2x(5) > _ 7
(ii) largest possible value of x "' 13 A 1
15

Answer (a)......................... 121

(b)(i).. . .. .. .... .. ........ 121

(b)(ii). . ...... ...... ..... 111


9 In lite figure, RS!!PQ. /..SRA - 30°, L RAQ - (7x - 40 )°and
.LJIQP - (2 x + 25)0 • Find the value of x.

R ---.... )31Y. • · ---· - - - S

"--- -~:::.~;~~~ .
p 1)..- ; 2S)" Q

L RAD =.;0°(a/1Ls, RS ii l'Q) MI (use of alt. angles)

L.DAQ e- (?x - 40)0 - 30° Ml


=(7.x - 70)°
L./)AQ I L.;JQT' - 180°(inlL.s,DA//PQ) Ml (uscorinl.an&l.:s)o~useofa~J.
angles on a sir. line)

(7x - 70) 0 + (2x + 25)° . t 80"


(9x - 45)0 ~ 180°
~);r" =225"
x ~. 25 !\ l

, /ns1ver ........ ................. l4J


5

/-or 10 Jn the diagram, ABCD is a <1uadrilaterul in which AB -- AD , BC= CD, LRA/) =50''
~xwniner"s
Use and LADC :: 140 °
(a) Find LBCD .
(b) Whal is the special name given to quadrilateral ABCD?

L AL>fJ =
180' -50'
-~ -
. Or sir , 140" +MO'+ LBc.-v- 360°
2 -. (b<1.u.• L ,· n'1>.r 1\}
(sum of inc ongles of a quad) M 1
=65°
/ CDB=l40•-65° LBCD=30° Al
- 75°
/RC!) ~ 180'- 75° 7S"(Lrsum of.~) MI
" 30° A I

Quad ABCD is a kite. :\ l


Answer (a}------------·---- -------- 121

I
J I The following pie cha1t shows the biand ()f mobile phones ownctl by a group of
1ccnagcrs.
(b)--------······-···---··· 111

LG

SamsunQ

Sony
Ericsson
..._____
___
(a) Write down the fraction oftccnai.;cr~ who own a Samsung mobile phone.
(b) If the number of teenagers who own a Sony Ericsson mobile phon e is three
times more than the number of teenagers who own a LG phone. Calculate
the angle of the sector which rcprc.~enrs the number of teenagers who own
a Sony Ericsson mobile phone.
(c) Calculate the total number of irtnagers in lhe grnup if the number of •
teenagers who own a Nokia 11mbik pho1Je is 24 more than those who own a
Sony Eric~son mobile phone.
6

,,,,.
(a) fraction of teenagers who own a Samsung phone ~ ~ Al EJc 8IYIJOO(
4 ll.'>e
(b) 3x + .x = 360° - 90° 1 t 8• MI
152
x=
4
.x 38" Al
auglcofsector (Sony Ericsson) 114° Al

(c) DilTcrcm:c - (118- 114)0


- 40
24
Tot;il number of teenagers ~- x 360 MI
4
- 2 160 Al
Answer (a).... ..................... [l J

(b)....... .... ............. [JJ

(c)....... ......... .... .... [21.

12 Construct a quadrilateral PQRS where l'Q = 65 cm, QR -· 4.8 cm°, RS " 8.5 cm, [SI
,( PQR -. 75° and L QRS - 98".
On the same diagram, constmet
(a)(i) the perpendicular bisector of f'S.
(ii) the angle bisector of / PSR

(b) Mark the point X where these two bisectors meet. Measure the length nf PX.
Constnact
(a) QR •·•··•·••• l m
(b) RS · ··• •· Im
(c) Line bisector ···•· •••• I rn
(d) Angle bisector ••·•·•• I ru
(c) Length PX -······· Im
Class Re9 Number
C.cmdiclate Name
C~I =1
T ANJ ONG KATONG SECONDARY SCHOOL
2011 YEAR END EXAMINATION
SECONDARY ONE

MATHEMATICS
PAPER 2
Friday 7 Oct 2011 1 hour 15 minutes
0750 - 0905

READ THESE INSTRUCTIONS FIRST


W rite your name, class and register number on the cover page.
W rite in dark blue or black pen.
You may use a pencil for any diagrams or graphs.
Do not use staples, popcr clips. highlighters, glue or correction fluid.

Answer all the questions.


All answers are to be written on the writing paper provided.
If working is needed ror any question it must be shown with the answer.
Omission of essential working will result in loss of marks.
You are expected to use a scientific calculator to evaluate explicit numerical
expressions.
If the degree or accuracy is not specified in the question, and 1r the answer is not exact,
give the answer to three significant figures. Give answers in degrees to one decimal
place.
For n, use either your calculator value or 3.142, unless the question requires the answer
in terms of 11.

The number of marks is given in brackets [ ) at the end or each question or part
question.
The total number or marks for this paper is 50.

I rum O\"CI'
Answer !!!_questions

A football club paid $25 million for a player. A nt:w~11aper reported that the club would
have lo sell 635 382 tickets 10 recoup their investment.
(i) Write down 635 382 correct to two significant figures . (I]
(ii) lJsc your answer to (i) and lht! infonnatiun above to estimate the cost ofa ticket,
correct m the nearC"st dollm . 12}

2 The cost of an advertisement in a magaI:ine is obtained by adding together a fixed


charge of80 ct:nts and an additional charge of35 ct:nts per word .
(1) Calculate the total cost of an advertisernenl containing 11 words. (I J
(ii) The total cost of an advcrtiscrncnl containing x number of words is T cents.
Write down a formula for Tin tc1tl'}S of x. (I J
(iii) Ooncx company is intending 10 prinl an advertisement in the magazine. With a
budget of only $8.00 available. what is the greatest number of words that Donex
can use for their advertisement'! [2J

(a) Solve the equauon


. 3q I
4
- 1 =5 -2-q . [3]

(h) Factorise completely 4hx - 6a+ l2ax - 2b. (4)

-I <'handra tell home at 08 30 and headed for school. After cycling for x minutes al a
speed of 30 km/h. he experienced a flat tyre. He spent 30 minutes to change the tyre
and then continued the second part of the joumcy for (x ' 10) minull!S al a speed of

~· I k.mlh
(1) h1ul, in tenns ofx.
(~) 1he distance lrnvclled in the firs! part of his journey. [ 1J
(b) the (lislancc lrnvell cd in tire second ~)art of his journey. (I l
(1i) Given 1ha1 the distance from his home to school is I 'i.5 km, form an eq uation in
lcrms of< and solve 11. 13 I
(1i1) What time di<l Chandra finally arrive in school'? II I

lamr•11.i· J,,,11011g Sec School !ti I I Ycur bu/ Exam


"

5 1lle line graph below shows the estimated number of visitors 10 l'ulor Harbour Resort
ovtT lhe past 6 years.
Visitor arrival at Pulor Harbou r Resort

K~timated oo. of
v isitors
(in thousands)

.- -

- )~ ~~.:~~_( !? ~. ·rd ~~1fJ! ~.t'.:~U I~ · 2( , l


. -=-~~:.: _.__ .. ;._ :.:: :._._-:_J.. - .!--:-.:. ::: -==-=-:::.: ._:_ . .. .:-::.-:.
Vear
{a) Describe the graph. II I
{h) Suggest a possible reason why there was a sharp drop in the number of visitors
in 2008. 111
(c) On avaagc, guests stay for 2 days in the resort and spend $85 on e111.:h day.
Calculate the income collected in 2009 correct lo the nearest milli on (71

ra1ljOlrJ.!. Ka1onx .i{.(!(' .'><:hoof Jiii I 1'1·m End £mm


(> The table below shows Mr Tan's water bill charges for the past 2 years.
Charges
i.-- . ·~

Year Fixed Cost I cubic metre (Cu M)

___
2010 $10/ month 80 cents
201 I
__,,___ _,_, -
36 cents/ day 80 cents

(i) Calcu late the total amount he paid in fixed charges for 2010. ( l)
(ii) Calculate the total amount he paid in fixed charges for the 365 days of2011 and
the percentage increase from 20 I0 to 2011. {3]
(iii) For 20 11 , Mr Tan paid $419.40 in all for his water bill. How many Cu M of
water did he use? (2)
.(iv) There was a leaky tap in Mr Tan's house that he did not fix. Water dripped from
.it at a rate of0.6 Cu M per day. Calculate the amount of money he could have
saved in a week. (2)

7 (a) Jn a heptagon, the exterior angles are (86 - x) 0 ,x 0 , 4x 0 , (2x + 15) 0 , (x - 4) 0 ,

(J 20 7-.<) 0 an<l 28". Calculate the value of x and hence dctcnninc the value of
[3)
th<~ largest interior angle in the heptagon.

(b}

v (I
The drngrnrn above shows a regular pentagon XYCRA and a regular hexagon
XYLUVW which share a comri1on sicle XY. Givi ng your reasons, calculate

(i) / XYC. [ 1J
(ii) / XYL. [ 1)

(iii) / !'Cl.. 13}

J'anjong Ka!(mg s~" Sdrool ·201 ! l'e11r t:nd £>.am


8 A series of dia!o,'Tams consisting of black nnd white triangles is shown below. The
hlack triangles line the outLT perimeter of each figure. All the other small triangles arc
" lute.

~
~
Fig. I
£ Fig. 2 Fig. 3 Fig. <1

The table below shows the number of small triangles


I{lgurc I 2 3 4 s ... II

~- of black lriangles 3 6 9 12 c ... x


I o~whitc.iriangit::>
No. I 3
.. . - -
7 . ' 13 ti
.
... y
. -
l 'l'otal number of tn angles 4 9 16 25 e ... -- - ~

(~) Oy considenng the number pattcms in the tahle or otherwise,


state the value of r., d aJJd of £'. [31
find expressions in tem1 s o f 11 for each orx and z. [2]
(b) 1lence or otherwise. write down the relationship between x.y and z in Ute fonn
of an equatton. Ill
( c} J low many white triangles are thc1 c in figure 73? (2]
(d) Is it possible for any figure to haven ro tnl nnmber of l lj4() triangles?
Rxplai n wiU1 reason . [2]

End of raper·

21111 Year Fm/ l:wm


2011 Yr End Seel Maths P2

QJ 640, 000
11 Cosl of a ticket

$25000000
= ··· -- -
640000
= $39.0625
,,,,,'f;19

Q2 Tola I cosl = 80c -r 35c{l I)


- $4.65 or 465c
11 T = 80 + 35c
111 800 2. 80 + 35x
800 - 80 = 720
35x :-;; 720 OR
720.;.35 = 20R4
x s 20.5

Q3

~! :- J -· 12 = ~-·-
-q
4 2
(111 - ?.6 20 - 4q

I Oq ' 46

3 23
q = 4 - or -
5 5
h 4bx - 6a + 12ax ·- 2b
·- 4bx 1- 12ax- 611 - 2h
= 4x(b + 3a) - 2(3a + b)
={4x - 2)(3a + b)
= 2(2x - 1)(3a + b)

Q4 ia .
D 1slance travel! cd ··· --
x >< 30
60

= x km
2
2011 Y1· End Sec I i"1aths P2

b . (x + 10) ~
J)istaoce tr;welled = - - - x "'4
60
2x+ 20
- - -km
5
I) t 2t+ 20 -
- + - - - · = 17.)
2 5
Sx t· 'Ix;. 40
17.5
10
9x+40 = 175
9.r = 135
x = 15
111 Total time taken -· 15 + 30 + 25
= 70min or I hr I Om in
Time that Chandra finally arrives ill schoo l is 9.40am or 0940

QS a Highest visitor arrival in 20 I I


I.owest visitor arrival in 2008
Upward trend in visitor anival from 2008 lo 2011

b Financial crisis. natural disaster or other logical aoswcr


c Income collected in 2009
= <)6000x$85x 2
S16320000
.,, $16mil

Q6 Total amou.n t paid in 20 I 0 = $120


I) Total amount paid in 20 I I - 365 x 36c
= $131 .40

0/
/(>
.
increase .;-..; $131.40 - $120 x 100•/
:ru
120

= 9.5%
201J Yr End Seel Maths J>2

$ti 19.40 - $131.40


Amounl of wnlcr used ' - -- - - - -
0.8
288
0.8
~360 CuM
1v Arnounl or money he could have saved = 0.6 x 7 x 80c

" $3.36

Q7 Tola I exlerior angles = 360 °

.x +4x+86 - x+2x + 15+x-4+120- 2x+28 = 360


5:t + 245 - 360
5x = 115
x 23
Larges! inlerior angle - 180° · J9°

- 161°
bi ,(, __ J X 180'°
F \ 'V
,_, ...
5
= 108 °
II 4 x 180°
/XYZ
6
120°
Ill L C>'/, = 360° - I 08° - I 2Q 0 (angles at a pt)
- 132"
Since CY = YZ

:. ~/ }'C'Z
. = ·1800-
- 1320
-- (base ang Ies o 1··1sos lr1ang
. Ie)
2
24°


Q8 ill c = 15
d . 21
l'. 36
201J Yr End Seel Maths J>2

$ti 19.40 - $131.40


Amounl of wnlcr used ' - -- - - - -
0.8
288
0.8
~360 CuM
1v Arnounl or money he could have saved = 0.6 x 7 x 80c

" $3.36

Q7 Tola I exlerior angles = 360 °

.x +4x+86 - x+2x + 15+x-4+120- 2x+28 = 360


5:t + 245 - 360
5x = 115
x 23
Larges! inlerior angle - 180° · J9°

- 161°
bi ,(, __ J X 180'°
F \ 'V
,_, ...
5
= 108 °
II 4 x 180°
/XYZ
6
120°
Ill L C>'/, = 360° - I 08° - I 2Q 0 (angles at a pt)
- 132"
Since CY = YZ

:. ~/ }'C'Z
. = ·1800-
- 1320
-- (base ang Ies o 1··1sos lr1ang
. Ie)
2
24°


Q8 ill c = 15
d . 21
l'. 36
2011 Yr End Seel Maths P2

II .t )11

Z = (n I 1) 2
b x+y - z

c When n = 73,

w..- (74) 2
- :l(73)

5257
d Yes

J1849 = 43 (whole number) or 43•d pallcm


BEAITY SECONDARY SCHOOL
END-OF-YEAR EXAMINATION 2011

SUBJECT : Mathematics LEVEL : Sec 1 Express

PAPER :1 DURATION: 1 hour 15 minutes

SETTER : Mr Bernard Lee DATE : 07 Oct 2011

. , CLASS:
! NAME : I REG NO :
=i
READ THESE INSTRUCTIONS FIRST

Write your name, class and index number in the spaces on the top of this page.
Write in dark blue or black pen.
You may use a pencil for any diagrams or graphs.
Do not use staples. paper clips, highlighters, glue or correction fluid.

Answer all questions.

If working is needed for any question, it must be shown with the answer.
Omission of essential working will result in loss of marks.
You are expected to use a scientific calculator to evaluate explicit numerical expressions.
If the degree of accuracy is not specified in the question, and if the answer is not exact, give the answer to
three significant figures. Give answers in degrees to one decimal place.
For Jr , use either your calculator value or 3.142, unless the question requires the answer in terms of Jr .

The number of marks is given in brackets [ ] at the end of each question or part question.
1nc total number of marks for this paper is SO.
For Examiner's Use

50

This paper consists of!! printed pages (including this cover page)
[Tum over
For 2 For
ammer .~
£'<.ammer
list· Use

Answer all the questions

(a) Express each o f the following correct to 3 significant figures.


(i) 7 139.28
(ii) 0.015 89
(b) Evaluate the following.

(i) VI0.5 2
- 4<>.25

(ii) 4
-~--

13 +.Jo.16
5

Answer: (a) (i) [ I]

(ii) [I]

(b) (i) [l )

(ii) [I]
For 3 For
om1ner ·s €.xammer
Use Use

2 (a) Write down the prime factori sation of 1512.

(b) State the integer q such that J15q'l. is the largest possible prime number.
(c) Find the LCM of36 and 56.

(d) If the LCM of 36, 56 and m is 1512, state the smallest value of m.

Answer: (a) rIJ

(b) [I]

(c) [I)
- - - -- - -- -- - - - -

(<l) f IJ
For 4 For
-ammer ·s Exammer
Ufe

3 (a) ~
Expand and simplify3( + 4)+ {*x - 6).

(b) Factorise a - 2b - ac + 2bc

Answer: (a) (2)

(b) f2]

a + 2 ii 3a + I I d . ..
4 (a) Sub tract - - rom - - - , an s1mp111y your answer.
10 5
, b b+ 5
(b) Solve - + - · = 2 .
2 3

Answer: (a) [3]

(b) [2]
Fur s For
1mmer ·s F.:cammer
lhe Use

5 Peter starts driving from Town P to Town Q at the same time that Quin starts driving
from Town Q to Town P. The ratio of Peter's speed to Quin's speed is 2 : 3. The
distance between the 2 towns is 240 km.
(a) How far will they be from Town P when they meet?
(b) lf Peter took 3 hours to reach Town Q, how many hours did Quin take to arrive
at Town P?

Answer: (a) km [2)

(b) hours [3]

6 30% of people in a room were male. lf 20 more males were added into the room, the
number of males would be equal to the number of females. Find the number of
females in the room.

Answer: [2)
For 6 For
1miner·s /i:fammer
Use Use

7 Observe the following sequence of numbers.


I, 3, 5, 7, 9, l l ...
(a) Write down the next 2 terms of the sequence.
(b) Find the 501h term.
( c) If 199 is the l(h term, find the value of k.

Answer: (a) [I]

(b) [I]

(c) k = [I]
8 (a) Solve the inequality 3x - x > -6.
(b) Hence, state the
(i) smallest integer x.
(ii) smallest prime number x.

Answer: (a) [I]

(b) (i) [I]

(ii) [I]
For 7 Far
1miner 's E:mmmer
Us.-
2
9 A solid hexagonal prism has a base area of I 000 cm and a height of 2 m.

(a) Calculate the volume of the prism in m 3 .


(b) The prism is placed into a hollow cylinder of radius 0.5 m and height 2m. Then,
water is poured into the cylinder to fill the empty space. What is the amount of
water required to fill the empty space?

2m

Answer: (a) rn3 [2)

(b) m3 (2]
Fur 8 For
an11ner 's Examfoe.r
Use Use

I0 Trapezium ABCE consists of a triangle AED and a rhombus ABCD. L.EAC = 67.5°,
and D is on EC s uch that ED == 5 cm, AD = 7 cm.

E 5cm D
c
(a) (i) Find L.ACD

(ii) Find LADE.


(b) By considering your answer in (a)(ii), deduce the length of AE.

(c) Hence, calculate tht.: area of the trapezium ABCE.

Answer: (a) (i) 0 rt]

(ii) o [I)

(b) cm [I)

(c) cm 2 [I)
For 9 Fnr
am1ner "s £ 1ammf!r
Use I/ft

ll (a) On the grid below,

(i) plot the points A(O, 3) and 8(-2, -3). Then, draw a line through A and B. [I]
(ii) draw the line x = 2. [I]

(b) Calculate the gradient of AB. l 1]


,..,. • ..- . • . . ..... . ..... ......._ .,......._.,. ,..,,,.,_, . _.....,.
~- ···--......,...,.1·
· • ···~-- .,~ ~ ,.. ~ ~..,.-~r'

' ·- • .. • ·I. ! ..,


~ ,. • ,,· _ . I
.. - f ' • 1

' · ·-' .• "t ' • · ·-

'• I tt .'
,.
' ..
'
I~" t • -
' •· •
._ I'

~ ·I'
--~ j'
~ i- .i

"' '.
-- ,j
. ~1
t.
.i.

.l
:L
.,
t
i
I ...
.
- · ~ ;~j !
· - - ......... . !'"-' ~.
~ ).
j
_+ .,_ • . · t:
.
t l
·! ·.•
.

'. ·•;

--~
.i

~
...

---· +- • ?-

'
. -..-.. r- ...... ____1_
.....

~~
-r---:---:---- -·3
•+ ..

1
...

. t
~

,,
,1.. . .I,
~.L., . : •.•
. .l f
-· l 1
~

...

Answer: (b) 375°


For IO For
amrnt'r '.t Examiner
Un• Use

12 120 students voted for their favourite colour. The pie chart below shows the results.

Gree11

(a) Calc ul ate the percentage of students who voted for red colour.
(b) Given that green and orange colours received the same number of votes,
calculate the number of votes for green colour_

Answer : (a) [2]

( b) [2]
For
JI For
ammer ·s f.:rammer
Use U<e

13 The diagram below shows a figure ABCDEFGH/J with equal sides.


8

H F

ACEGI is a regular pentagon. Calculate

(a) LACE. [2)

(b) LABC. 121


(c) the s um of the interior angles in fi gure ABCDEFGH!J. Ill

Answer: (a) 0 [2]

(b) 0 [2]

(c) 0 [1]

END OF PAPER
For 12 For
aminer ·s E.wminer
U,-e Use

Answer K ev
Q!! Answer ·-
J_(a) 7140 ·---
l(b) 0.0159
l(c) 4
l(d) 2

2(a) 1512 = 2 3 x3 3 x7
2(b) 168 --
2(c) 504
2(d) 27
>---- - -- ·
3(a) x
3(b) (a - 2b)(l- c)
1-------- - ·- - ·
4(a) a +4
--
-- ~--7 - ..
4(b) b = 0.4

5(a) 96km
5(b) 2 hours
--·-~

--
6 35

7(a) 13, 15
7(b) 99 - ~- -· --

7(c) k = 100
-- --- -----------
8(a) x > -3
8(b)(i) -2
-
8(b)(ii) 2

9(a) 0.2m5
9(b) 1.37 m 3

JO(a)(i) 22.5 °
lO(a)(ii) 45 °
·-
JO(b) 5cm
JO(c) 47.5 cm.:

1 l(b) 3

I 2(a) 108°
......
12(b) 36°
12{c) 1440°
I

BEATTY SECONDARY SCHOOL


END-OF-YEAR EXAMINATION 2011

SUBJECT : Mathematics LEVEL : Sec 1 Express

PAPER :2 DURATION : 1 hour 30 minutes

SE'fTER : Mr Lim CH DATE : 13 October 2010

[ CLASS : [ NAME :
................... ...•••.•••••••••..................•.........•.....
REG NO :
J
READ THESE INSTRUCTIONS FIRST

Write your name, class and index number in the spaces on the top of this page.
· Write in dark blue or black pen.
You may use a penci l for any diagrams or graphs.
Do not use staples, paper clips, highlighters, glue or correction flu id.

Answer all questions.


If working is needed for any question, it must be shown with the answer.
. . Omission of essential working wiJI result in loss of marks.
Calculators should be used where appropriate.
If the degree of accuracy is not specifi ed in the question, and if the answer is not exact,
give the answer to three significant figures. G ive answers in degrees to one decimal
place.
For 7r, use either your calculator value or 3.142, unless the question requires the
answer in tenns of TL.

At the end of the examination, fasten all your work securely together.
The number of marks is given in brackets [ ] at the end of each question or part
question.
The total number of marks for this paper is 50.

This paper consists of§. printed pages (including this cover page)

[Turn over
2

. J"fy 3g +4h 1g-2h


). (a) S 1mp1 [2}
8 3
(b) (i) Factorise I 2j - 40ef - I Oe + 3 completely. [2}
(ii) If 12f - 40ef - l Oe + 3 represents the area of a rectangle, write
a similar expression for the perimeter of the rectangle. [I]

2. Leonard left home at 6:40 am and cycled at an average speed of 5 mis from his
house to school. He reached school at 7:00 am.
(a) Find the distance between his house and the school . l2J
(b) After waiting for 30 minutes, Leonard cycled home at 10 mis.
Calculate the average speed in m/s, for the entire journey. · [2]

3. The bar chart below shows the total ticket sales for the concert 'Lion King'
that was held in Marina Bay Sands concert hall.
(a) Find the total number of tickets sold. l1]
(b) Calculate the total ticket sales. [2]
(c) If the hall has a seating capacity of 350, ca1culate the percentage of the
seats that were not occupied. [2]

Total Ticket sales for 'Lion King'


120
:2 100
0

...."'"'
QJ 80
.:ic
u

-.......
0
60
QJ
..0 40
E

• I
::l
z 20

0
10 20 30 40 50

Price of Ticket($)
3

4. 1be volwne ratio of tea, bubbles and jelly in a bubble tea drink is 3 : I : 4.
(a) Calculate the percentage of tea in the bubble tea drink. [1]
_--~~ _ ~culate the "'.olu;rne_of~u_l_>l_>les i~ a~50_m/b~b~l~ ~~~- [1)
(c) Tea costs $5 per litre. bubbles cost $6.80 per litre and jelly costs $6.50
per litre. Calculate the cost price ofeach 250 ml bubble tea drink,
correcting your answer to the nearest cenl [2]
(d) ·If the drink-is sold at a profit ofS0.10, find the selling price after
applyin_g G .S .T of 7%. [2]

5. The diagrams below show a sequence of dot patterns.



• • ••• •
•••
•• •
• •
Pattern I Pattern 2 Pattern 3

(a) The information from the sequence ofdots is tabulated below.


Complete the table. [2]

Pattern n Formula Nwnber ofdots

1 I l

2 4+1 5

3 4+4 + 1 9

(b) Write ~own a formula for the nth ~ [I]


(c) Hence or otherwise, find the number ofdots in the 25th pattern. [I ]
(d) In pattern k, the number of dots is 5()C)_ Find k. [2]
4

6. Answer this wllok question oa a slteet of graph papu.

During a science practical lesson. a bot test tube with temperature y °C, is left
--·-- - ----·---- ·-··~~. - ·~· · ·- -
to cool. At time.x minutes, the test tube's temperature is given by
y = - 5x+80for 0 5 x :S IO . Some values of x andyare recorded in the table

below.

(a) Copy and complete the table below. {2)

.x 4

y =-5x+80

(d) Using a scale of l cm for I unit on the x-axis and I cm for 5 units on
the y-axis, draw the graph of y = - 5.x + 80 for 0 :S .r :S 10 . (3)
(e) Using your graph, find
(i) the value of y when .r = 5, [I]
(ii) the gradient of the graph. [2]
(f) If a fan was used to cool the test tube, describe how the gradient of the
graph weuld change. (I]
5

7. Answer this question on a single page of plain paper. Start your


constructions in the centre of the paper.

Construct a quadrilateral ABCD such that AB = 6 cm, BC= 6 cm,


LABC = 144°, LBAD = 36° andLBCD = 36°. (3)
(a) On the same diagram, construct
- (i) the angle bisector ofLABC, [1]
(ii) the perpendicular bisector of BC. [ 1]
(b) The two bisectors in (a)(i) and (a)(ii)intersect at point P. Label P and
find the length ofPB. [l]
(c) ABCD is a special quadrilateral. State the name of this special
quadrilateral. (1)
6

8. Figure 1 below shows a water trough with a uniform cross section of


trapezium shape. Given that FG = 20 cm, GH = 80 cm, CD = 50 cm and FM =
-20-cm. ------ - -- --- - ·--- - · - - ----

(a) Find the volume of water in the trough. [2]

(b) Figure 2 shows an open cylindrical container of height 70 cm. When


all the water from the trough is poured into the cylindrical container,
the water reaches the brim. Find

(i) . the radius r of the cylindrical container, [2]

(ii) the total interior surface area of the open cylindrical container
that is in contact with the water, leaving your answer to the
nearest cm 2 • [2]

(c) It costs $50 to paint one square metr e of the cylindrical container,
find the total cost needed to paint the interior surface area of the open
cylindrical container that is in contact with the water. [2]

50cm
c

70 cm

-------- ----
,'
F 20cm

Figure 1
Figure 2

End of Paper
Solutions to 1E Paper 2
. ---- -- --- ·- - - -- ~- --- - -
3g+4h - 1g-2h 3(3g + 4h) 8(7 g-2h)
1. (a) = 24 - 24 (Ml]
8 3
= 9g+l2h-56g+l6h
24
= - 47g+38h (Al]
24

(b) (i) 12J - 40ef - IOe + 3 = (12/ - 40ef)-(10e - 3)


= 4/(3 - lOe) - (lOe-3)
= 4/(3 - 10e) +(3 - 10e) (Ml]
= (4f + 1)(3 - l Oe) (Al]

(ii) Perimeter of rectangle = 2(4/ + l} + 2(3- IOe)


= - 20e+8f +8 (Bl]

(a) Distance between house and school = 5 x 20 x 60 (Ml]


2.
=6000m (Al]

(b) Time taken for return journey = 6000+10


=600s (Ml)
6000x 2
Average speed for whole journey = - - - - -- -
1200+ (30x 60) + 600
I
= 3- mis (Al]
3

3. (a) No. of tickets sold = 120 +70+50+20 +30


= 290 (Bl]

(b) Total collection from the ticket sales


=(I 0 x 120) + (20 x 70) + (30 x 50) + (40 x 20) + (50 x 30) IMl](~
== $6400
(Al]

(c) Percentage of seats unoccupied = 35o- 290 x lOOo/o (Ml](.../)


350
=17_!_% (Al]
7
3
4. (a) Percentage of tea = - x 100%
- &_ ____ - .. ·- -· -- ··-
----- - - --- --- - - - - - - ---
=37.5% {Bl)

(b) Volume of bubbles =..!_x250m/


8
=31.25m/ (Bl)

(c) Volume of tea =~x250ml


8
= 93.75ml

Volume ofjelly =.!..x 250ml


2
= 125ml
Cost price of each bubble tea drink
= (93.75 x 5) + (31.25 x 6.80) + ( 125 x 6.50) (Ml)
1000 1000 1000
=$1.49 (3.s.f.) (Al]

107 .
(d) Selling price of bubble tea drink = - x ($1.49375+0. 10) (Ml]
100 .
~ $1.71(2.d.p.) (Al]

5. (a)
-
Pattern n Formula Number of dots

I I I

2 4+1 5

3 4+4 + 1 9

4 4+4+4+1 13

5 4+4+4 +4+1 17

Each correct pair of Formula and Number of dots.- (Bl I

2
(b-)---- Fomrnla--for number of_dots ------- -- ·-=· 4n -~3 -- [B-IJ- - - -- - -

(c) Number of dots in the 25th pattern = 4(25) - 3


=97 [Bl]

(d) 4n - 3 =509 [Ml}


4n = 509+3
n =128 (Al]

6. (a)

x
y =-5x+80

Every group of 3 correct values [Bl]

(b) Correct Scale and Axes & Origin. (Bl]


All points correctly plotted with 'cross'. [Bl]
All lines correctly labeled. (Bl]

Graph shown on next page.

(c) (i) y= 55 (Bl)

70 - 30
(ii) Gradient (Ml)
2 -10
= -5 (Al]
No marks would be given ifno workings were shown as the question
requires students to use the graph for solution.

(d) The gradient gets steeper/


the magnitude of the gradient increase/
the gradient becomes more negative. (Bl}

3
+-i~~rt tt.-~:J=-"i i-:-~-- ~-'0t +7-1--:-r-:-r-i~r-i=-~J=r~1~ ~: -~ l --1+-:~ J:-:-!-+,-•-- i ---- .9 - ! &:::-~-~-----:~

E~J~U~in:; jytiJ ~trfil 1=~fl tit:1,_t_:41~ --~ L&0i ;=IT ;j I L·

- -

4
7. Not drawn to scale
·- --- -------- .. -- . - ------- ------ . - - - - --· -- - --
--·-·

c
36°

cm

36"
A 6 cm B

Correct lengths for sides (BJ]


Correct angles at A and B (Bl J
Construction arc shown at C (Bl]

(a) (i) Bl (.../) - construction arcs shown

(ii) Bl (.../) - construction arcs shown

(b) PB= 9.7(±0.1) cm (Bl]

(c) Rhombus. (Bl]

5
8. (a) Volume of water in trough = -1 (20- + 50)(20) x 80 [Ml]
- - 2 ---- --- --
= 56000 cmJ (Al]

(b) (i) Radius, r =.J56000+70+;r [Ml]

=16.0cm (3.s.J.) [Al]

6
Index Number: _ _ __

Class: - - --
Clementi Town Secondary School
End-of-Year Examination 2011
Secondary 1 Express

Mathematics
Paper 1 1 hour

Additional Materials provided : Writing Paper for rough working

ClaEKTI TOWN SECONOARY SCHOOi. ClEMENTI TOWN SECONDARY SCHOOi. CLEMENTI TOWN SECONDARY SCHOOi.. CLEMENTI TOWN SECOfo.OARY SCHOOi.
CLEMENTI TOWN SECONDARY SCHOOL ClEMENTl TOWN SECONDARY SCHOOL CLEMENTI TOWN SECONOARY SCHOOL CLEMENTI TOWN SECONOl.RY SCHOOi.
CLEMENTI TOWN SECONOARY SOJOOL CLEMENTI TOWN SECONDARY SCHOOL Cl.EMl:NTI TOWN SECONDARY SCHOOL CLEMENTI TOWN SECONOARY SCHOOL
ClEMENTI TOWN SECONDARY SCHOOL CLEMENTI TOWN SECONDARY SCHOOL CLEMENTI TOWN SECONDARY SCHOOl CLEMENTI TOWN SECONDARY SCHOO\.
CLEMENTI TOWN SECONDARY SCHOOi. CLEMENTI TOWN SECONDARY SO!OOL CLEMENTI TOWN SECONDARY SCHOOL CLEMENTI TOWN SECONDARY SCHOOL

READ THESE INSTRUCTIONS FIRST

Do not open the booklets until you are told to do so.


Write your name, register number and class on all the work you hand in.
Write in dark blue or black pen on both sides of the answer paper.
Do not use staples. paper clips, highlighters, glue or correction fluid.

Answer all the questions.

Write your answers in the spaces provided on the question paper.


If working is needed for any question it must be shown with the answer.
Omission of essential working will result in loss of marks.

CALCULATO~S ARE NOT ALLOWED TO BE USED IN THIS PAPER

The number of marks is given in brackets [ ] at the end of each question or part question.
The total number of marks tor this paper is 50.

FOR EXAMINER'S
USE

50

This question paper consists of I printed pages, including this cover page.
(Turn ovef]
2 Clementi Town Secondary School
Mathematics I Paper 1 Secondary 1 Express End--0f-Year Examination 2011

Answer all the questions in the spaces provided.

1 Estimate I 0.45 x ..J22.2 , giving your answer correct to one significant figure . .

Answer [2]

2 Evaluate the following 34 - 4x(28-30}+2.

Answer [2]

Write down a simple fraction which is halfway between i and ~,in the form of~.
7 7 b

Answer [2]

4 Given that 3a = 4b, find the ratio of a : b.

Answer [2 ]
Clementi Town Secondary School 3
End-of-Year Examination 2011 Secondary 1 Express Mathem atics I Paper 1

x- 4y2
5 Given that x = 2 and y = - 3 , evaluate - --'--
x- y

Answer (21

6 Given that a and b are integers such that 2 < a s 5 and -3 S b s 2, find

(a) the greatest possible value of ab2 ,

(b) the least possible value of _!_ + .!.. .


a b

Answer (a) [IJ

(b)

7 The average speed of a car for the first I hour of its j o urney is 80 km/h. T he driver made J :

hour stopover for petrol and lunch. Its average speed for the remaining 2 hours is 100 km/h.
Find its average speed for the whole j ourney.

Answer ................. .... ..... km/h [2 J

8 The first four terms of a sequence are 6, 10, 14, 18, .... . .

(a) Write down the next tenn in the sequence.

(b) Write down the' n1h term in the sequence.

(c) Write down the 100111 term in the sequence.

A nswer (a) [ I]

(b) [l ]

(c) [I ]

[Turn over
4 Clementi Town Secondary School
Mathematics I Paper 1 Secondary 1 Express End-of-Year Examination 2011

9 Solve the following equations

(a) - 3(2- Sx) = 9,

2
(b) - y - 3=3.
3

Answer (a) x = -····-· · ·· · ······· · · -·· ·· (2]

(b) y = ........ ... .............. [2]

10 (a) Expand and simplify c - 3 {3c - 2) .

(b) Factorise completely d + 2de - 2f - 4ef.

Answer (a) [2J

f21
(b)
- - - - - -- - - - -- - - - -- - - - -- - - - -- - - -- -- -·-- ------
11 A polygon has n sides. Two of its exterior angles are 21° and 54°.

The remaining {n - 2) exterior angles are 15° each. Find

(a) the value of n,


(b) the sum of interior angles of the polygon.

Answer (a) [2]


0

(b) (2)
aementi Town Secondary School 5
End-of-Year Examination 2011 Secondary 1 Express Mattiematics I Paper 1

12 (a) Express 80 as a product of its prime factors, giving your answer in index notation.

(b) Find the smallest positive integer k such that 80k is a perfect square.

(c) Find the value of .V&OxlOO.

Answer (a) [2J

{b) [ l]

(c) (2]

13 In a certain polling station, there were 1 000 voters.


(a) During the General Election, 20% of them failed to cast their votes. Calculate the
number of people who voted.
(b) During the Presidential Election, there was a percentage increase of 12.5% in the
number of people who voted. Calculate the number of people who voted.

Answer (a) (2)

{b) (2)

[Turn over
6 Clementi Town Secondary School
Mathematics I Paper 1 Secondary 1 Express End-of-Year Examination 2011

14 The graph below shows a straight line AB.


(a) Write down the coordinates of point C.
(b) Find the gradient of AB.
(c) Write down the equation of the line that is parallel to AB and pass through point C.

Answer (a) [I]

(b) [2]

(c) [11
Clementi Town Secondary School 7
End-<>f-Year Examination 2011 Secondary 1 Express Mathematics I Paper 1

15 In the figure, ABCD is a parallelogram. Calculate


(a) the area of the parallelogram,
(b) the value of h.
12 cm
.. '

..
.:hem

Answer (a) ....................... .. cm2 [2]

(b) h =...... .. . ... .......... .... [2]

16 The solid shown below is a prism. Find the


(a) volume of the prism,
Scm
(b) total surface area of the prism.

) ---- -- -- ---- --- -- -


,'
,'

3 cm ,'

4cm

Answer (a) .... . : .. ... ............. ..cm3 [2]

(b) .......................... cm2 [2]

- End ofpaper -

(Turn over
2011 IEXP
2011 IEXP EOY Exam Pl A
I) 50
l lb) 3420°

2) 44
4
12a) 80 == 2 x 5

9
3)
14 12b) 5

4) a:b=4:3 12c) 20

4 l3a) 860
5) - 6-
5
l3b) P=900
6a) 45
14a) (0, 1)
4
6b) --
5
14b) gradient of AB = _!_
2
7) Average speed = 70km I h
I
14c) y=-x+ I
8a) 22 2

2
Sb) 4n +2 !Sa) Area of parallelogram = 72cm

8c) 402 15b) h ==. 6

9a) x =1 16a) Volume of prism= 48cm 3

9b) y=9 l6b) Total surface area of prism


2
= 108cm
!Oa) - 8c+6

I Ob) (d - 21X1+2e)

Ila) n =2 1
Index Number : - - - -

Class : - - -

Clementi Town Secondary School


End-of-Year Examination 2011
Secondary 1 Express

MATHEMATICS
Paper 2 1 hour 30 minutes

Additional Materials provided : Writing Paper, Graph Paper and Plain Paper

CLEMENTI TOWN SECONDARY SOIOOI. CLEMENTI TOWN SECONOARY SCHOOl CLEMENTI TOWN SECONDARY SCHOO!. CLEMENTI TOWN SECONOARY SCHOOL
CLEMENTI TOWN SECONOARY SCHOOL CLEMENTI TOWN SECONDARY SCHOOL CLEMENTI TOWN SECONDARY SCHOOt. CLEMENTI TOWN SECONDARY SCHOOL
CLEMENTI TOWN SECONDARY SCHOOL CLEMENTI TOWN SECONOARY SCHOOL CLEMENTI TOWN SECONDARY SCHOOL CLEMENTI TOWN SECONDARY SCHOOL
CLEMENTI TOWN SECONOARY SCHOOL CLEMENTI TOWN SECONDARY SCHOOL CLEMENTI TOWN SECONDARY SCHOOL CLEMENTI TOWN SECONDARY SCHOOL
CLEMENTI TOWN SECONDARY SCHOOL CLEMENTI TOWN SECONDARY SCHOOL CLEMENTI TOWN SECONDARY SCHOOl CLEMENTI TOWN SECONDARY SCHOOL

READ THESE INSTRUCTIONS FIRST

Do not open the booklets until you are told to do so.


Write your name, register number and class on all the work you hand in.
Write in dark blue or black pen on both sides of the answer paper.
Do not use staples, paper clips, highlighters, glue or correction fluid.

Answer all the questions.

If working is needed for any question it must be shown with the answer.
Omission of essential working will result in loss of marks.
Calculators should be used where appropriate.
If the degree of accuracy is not specified in the question, and if the answer is not exact,
give the answer to three significant figures.
Give answers in degrees to one decimal place.
For n, use either your calculator value or 3.142, unless the question requires the answer
in terms of re.

At the end of the examination, fasten all your work securely together.
The number of marks is given in brackets [ ) at the end of each question or part question.
The total number of marks for this paper is 50.

For Examiner's Use

This Question Paper consists of~ printed pages.


[Turn over
2 Clementi Town Secondary School
Mathematics I Paper 2 Secondary 1 Express End-of-Year Examination 2011

Answer all the questions.

1. Solve the following equations

2
(a) 3x= - [I]
7'

(b) [3]

2. There are three numbers such that the second is three times the first and the.third
is three less than four times the first. It is given that x is the first number and the
sum of the three numbers is 37.
(a) Write down an algebraic expression for the second and third numbers. (2J
(b) Form an equation connecting the three algebraic expressions. (lj
(c) Solve the equation and find the three numbers. [2]

3. (a) It is given that V = 2:rr.J1 2 + h 2 • Given that :rr = 3.142, I = 20and h = 9.8 ,
calculate the value of V to 3 significant figures. f2]

a b
(b) Subtract the sum of - and - from 2(a - b). [2]
3 4

4. Answer the following questions by referring to the advertisement below

Multi system Hi Fi Player


Cash
Original Price : $2 500

NOW: $1 798

or
$55 monthly instalments x 38 months
$200 DEPOSIT!

(a) Find the percentage discount for payment in cash compared to the original price,

giving your answer correct to one decimal place. (31

(b) What is the difference between the instalment price and payment by cash? [2]
Clementi Town Secondary School 3
End-of-Year Examination 2011 Secondary 1 Express Mathematics I Paper 2

Sm 7
5. (a) (i) Solve -~-- [2)
6 3

(ii) If m is an integer, what is the smallest value of m ? [I)

(b) Ariff wants to buy pizzas for his birthday party. If one pizza costs $19,
find the maximum number of pizzas Ari ff can buy if he has $350. [2]

6.

A x D

In the diagram, AB CD is a quadrilateral in which LABC = 120°, L BCD = 110°, LCDA = 75°

and BC= CD. Point Xis o n AD is such that BX is parallel to CD.


Calculate
(a) L.BAX, [2]
(b) L.BDX, [2]
(c) L.BXD. [2]

7. Answer the whole of this question on a sheet of plain paper.

(a) Construct a triangle ABC such that AB= BC = 7 cm and AC = 9 cm. [3]

(b) Measure and write down the size of L.BAC. (I )

(c) Construct the angle bisecto r of L.BA C . [I)

(d) Construct the perpendicular bisector of AC. (I]

(e) Label the point of intersection of the 2 bisectors from ( c) and (d) as M.
Measure and write down the length of AM. (1]
4 Cleme.nti Town Secondary School
Mathematics I Paper 2 Secondary 1 Express End-of-Year Examination 2011

8. In a factory, waste liquid is poured into cylindrical drums. The area of the base of each drum
is 0.2 m2 and its height is 90 cm.

(a) Calculate the volume of each drum, giving your answer in m3• [2]

--When each drum is full, the waste liquid is emptied into a tank as shown.below.
ADHE, ABFE and DCGfl are rectangles, while ABCD and EFGH are trapeziums.
EF = 4.5 m, DC = 3.5 m, AE = 2 m and the perpendicular height of the tank is 1.5 m.

c
(b) Calculate

(i) the area of the trapezium ABCD, [2]


(ii) the volume of the tank. [2]

(c) How many full drums of waste can be emptied into the tank? [I]
Clementi Town Secondary School 5
End-of-Year Examination 201 1 Secondary 1 Express Mathematics I Paper 2

9. Answer the whole of this question on a sheet of graph paper.

The table below shows the corresponding x and y values for the equation y =- 2x + I .
x -2 -1 0 2
y p 3 q -3

(a) Find the value of p and of q. [2]

(b) Using a scale of2 cm to represent 1 unit on both axes, draw the graph of
y =-2x + l for - 2 .:s; x .:s; 2 . [3)

(c) On the same axes, draw and label the line x =1.5 . [I]

(d) From the graph drawn, write down the coordinates of the point of intersection of
x = l.5 and y= - 2x+I. [1]

END OF PAPER
Answer Key

2
1 (a) x= -
21

1 (b) x=4

2 (a) Second number is 3x


Third number is 4x - 3.

2 (b) x+3x+4x-3=37 ·

2 (c) The three numbers are 5, 15 and 17.

3 (a) V = 140 (3 sig. fig.)

20a-27b
3 (b)
12

702
4 (a) Percentage discount= x 100% = 28.1%
2500
4 (b) Difference= $2290 - $1798 = $492

4
5 (a)(i) m ~ 2- or 2.8 [A1]
5
5 (a)(ii) Smallest value of mis 3

5 (b) Maximum number of pizzas is 18.

6 (a) LBAX = 360°-120° -110° - 75° (sum of interior angles in a quadrilateral)


= 55°
6 (b) LBDX = 75° - 35° (sum of angles)
= 40°
6 (c) LBXD = 180° - 35° - 40° (angle sum of triangle)
= 105°

7 See attached.
8(a) Vol of each drum
= 0.2 x0.9 M1
3
=0.18m
A1

8(b)(i) Area of trapezium ABCD


l
= - (3.5 + 4.5)x l.S
2 M1
=6m 2
A1
8(b)(ii) Vol of tank
=6x2 M1
3
=12m
A1
8(c) _g_ = 663_
0.18 3

66 full drums can be emptied into the 81


tank

9 See attached.
Geylang Methodist School (Secondary)
End-of-Year Examination 2011

Candidate
Name

Class Index Number


.... ~·- ~ .. -

MATHEMATICS

1 Express
Paper1

Candidates answer on the Question Paper. 1 hour

Setter: Ms Grace Yap 3 October 2011

READ THESE INSTRUCTIONS FIRST

Write your name, index number and class on all the work you hand in.
Write in dark blue or black pen in both sides of the paper.
You may use a pencil for any diagrams or graphs.
Do not use staples, paper clips, highlighters, glue or correction fluid.

Answer all questions


If working is needed for any question, it must be shown with the answer.
Omission of essential working will result in the loss of marks.
Calculators should be used where appropriate.
If the degree of accuracy is not specified in the question. and if the answer is not exact, give
the answer to 3 significant figures. Give answers in degrees to one decimal place.
For n, use either your calculator value or 3.142, unless the question requires the answer in
terms of 11:.

At the end of the examination, fasten all your work securely together.
The number of marks is given in brackets [ ] at the end of each question or part question.
The total number of marks for this paper is 50.

For Examiner's Use

50
This document _consists of 10 printed pages.

[Turn over
GMS(S)/Math/Pl/EOY201 l/1E

Total Score: (SO marks]


Answer ALL questions in this paper.

1 Consider the numbers stated below. Write down the prime numbers.
0.2,~' jj, 7, 9, 13, 85, 144
5

Answer: _ _ __ _ _[1)

2 A distributor has 48 apples, 72 oranges and 96 pears in his storeroom. If each type of

fruit is distributed equally among a certain number of fruit baskets, what is the

greatest number of fruit baskets that can be prepared?

Answer: _ _ _ __ _ ,[2)

3 A restaurant owner estimated that he had 200 customers on a particular day.

If the restaurant owner had rounded off the number of customers to 1 significant

figure,

(a) what was the smallest possible number of customers?

(b) what was the largest possible number of customers?

Answer: (a) _ _ _ _ _ _ _,[ I]

(b) _ _ _ _ _ _,[1]

2
GMS(S)/Math/P l/EOY201111 E

4 A car uses 5 ~ litres of petrol for 48 km. How far can it travel with 23 ·litres of
4

petrol.

Answer: _ __ _ __ km[2)

5 If a = 3, b = - 1 and c = 1, find the value of


(a) 2a - 3b+ c,

a - 2b 2b+c
(b) --+--
3 4

Answer: (a)_ _ _ _ _ __ _ [l)

(b)_ _ _ _ _ _[2]

3
GMS(S)/Math/Pl/EOY201 l/1E

6 Simplify 7(x - Sy) - 4(y - 3x).

Answer: _ __ _ _ _ _[2]

2(5a-7) 3{a-3) fi ...


7 Express 4a + - as a raction m its 1owest terms.
6 4

Answer: _ __ __ _ _[31

4
GMS(S)/Math/Pl / EOY201 l/IE

2
8 Solve - x + 15 = 4x.
3

Answer: _ _ _ __ _ _ [2]

9 The sum of interior angles of a polygon is 1620°. Find the number of sides of the

polygon.

Answer: _ _ _ _ _ _[2]

IO Mr Lai bought a condominium in 2005 for $675000. He sold it in 2007 for $856000.

Find the percentage increase in price.

Answer: _ _ _ _ _ _[2]

5
GMS(S)/Math/P1/EOY201 l/1 E

11 The graph below shows a straight line passing through two points P and Q.
y
l.
'

':
'.
' ~ - f •

--~- ..._' .. I :_ ·__


.. •.. --- :: :'.:: .... J : __ -:-;- .. t :·r ::.,·.
(a) Write down the coordinates of the points P and Q.

(b) Calculate the gradient of the straight line PQ.

Answer: (a) ~P~<~_).__....Q~<-~~)[1]

(b)_ _ _ __ _[2)

12 The first four terms in a sequence are 2, 5, 8 and 11. Write down an expression, in

terms of n, for the n 1h term of this sequence.

Answer: _ _ _ __ __ [2]

6
GMS(S)/Math/PI/EOY201 I/IE

2
13 (a) Solve the inequality ; 2: 6.

(b) State the smallest possible value of x ifxis an integer.

Answer: (a)_ _ _ __ _ _ _ [2]

(b)_ _ _ _ _ _[l]

14 In the diagram, find


12 cm
(a) the area of the parallelogram, ·············· ....
9 c~···
(b) the value of h. ~h cm

Answer: (a)_ _ _ __ _ _ _[2]

(b)_ _ __ _ _[2]

7
GMS(S)/Math/P I/EOY2011 I l E

15 Factorise 9at - 45t - 15 + 3a completely.

Answer: _ _ __ _ _ _[3]

16 In the diagram, AB, CD and EFG are parallel. L BAF = 43° and L DCF= 114°.

Stating the reasons dearly, find the value of

{a) x, A ..--...-~~~--~~~~

(b) y,

(c) z.

E F G

Answer: x = _ _ __ _ __ [l]

y = _ _ _ _ _ _ _[1]

z =_ _ _ _ _ __ [2]

8
GMS(S)/Math/P 1/EOY20 l l/1E

17 The histogram below illustrates the results of a survey to find the number of

passengers in cars crossing a bridge.

Number of
cars

5· -

3 -+--~

o.___....___. . . . ____..__________________
2 3 4 5
Number of passengers

(a) Calculate the total number of cars.

(b) Calculate the total number of passengers.

(c) Find the percentage of cars which have more than 2 passengers.

(d) If the information is to be illustrated in a pie chart, calculate the angle of the
sector representing cars with 5 passengers.

Answer: (a)_ _ _ _ __ _ _ _ [1]

(b)_ _ _ _ _ _ [2]

(c)_ _ _ _ _ _ _ [2]

(d)_ _ _ _ _ _[l]

9
GMS(S)/Math/Pl /EOY2011 / 1E

18 (a) Construct a triangle ABC given AB = 13cm, BC = 5cm and AC = 12cm.

(b) Construct the perpendicular bisector ofAC.


[4)

- - End of Paper - -

10
GMS(S)/Math/Pl/EOY201 l/l E

Answer Keys (Paper 1)

I 7, 13 13b x =:' 9
2 24 14a 72etnL

3a ISO 14b 6'cm


3b 249 IS (a - S)(9t +3)
4 192km 16a x = 43°
Sa IO 16b y=66°
Sb I 2_ 16c z = 71°
12
6 19x - 39y 17a 20
7 59a-1 17b 57
12
8 4.S 17c 60%
9 n = 11 17d 36°
10 26.8% 18 Drawing is in marking scheme
·-
Ila P(4, 4) Q(- 4, 2)
I lb 1
0.2S or -
4
-
12 Tn = 3n - I
13a x~9
- ·-
Geylang Methodist School (Secondary)
End-of-Year Examination 2011

MATHEMATICS
1 Express
Paper2

Additional materials : Writing Paper


Pl~in -Paper 1 hour 3 0 m in utes
Graph Paper

Setter : Miss Tan Lay Ming


3 Oct 2011

READ THESE INSTRUCTIONS FIRST

Write your name, index number and class on all the work you hand in.
Write in dark blue or black pen in both sides of the paper.
You may use a pencil for any diagrams or graphs.
Do not use staples, paper clips, highlighters, glue or correction fluid.

Answer all questions


If working is needed for any question, it must be shown with the answer.
Omission of essential working will result in the loss of marks.
Calculators should be used where appropriate.
If the degree of accuracy is not specified in the question, and if the answer is not
exact, give the answer to 3 significant figures. Give answers in degrees to one
decimal place.
For TI, use either your calculator value or 3.142, unless the question requires the
answer in terms of TI.

At the end of the examination, fasten all your work securely together.
The number of marks is given in brackets [ ] at the end of each question or part
question. ·
The total number of marks for this paper is 50.

This document consists of 5 printed pages and 1 blank page.

[Turn over]
GMS(S)/Math/P2/EOY2011/1 E

Paper 2 (50 marks)

1 There are 20 questions m a quiz. For every co1:Tect answer, incorrect answer and

unanswered question, 2 points will be awarded, I point wil1 be deducted and no mark

will be awarded respectively. Jericho has 13 correct answers, 3 unanswered questions

and the rest are incorrect answers. What is Jericho's score for this quiz? [2]

2 (a) · The area of a trapezium is 1500 cm2 . Find the height, h, if the lengths of the
..-14c~
parallel sides are 14 cm and 36 cm. [3]

;I L\
(b) The figure below shows a square of sides 14 cm. It contains a pattern made up of

a semicircle and a quadrant. Calculate the area of the shaded region. (3]
.-~~ 14cm~~-+
22
(Use n = - )
7

2
GMS(S)/Math/P2/EOY2011/ 1E

3 Dexter is going on a school field trip to Indonesia. He plans to buy toy aeroplanes and

toy cars for the orphans at an orphanage. Dexter has a $30 budget and he cannot exceed

this amount. The cost of the toy aeroplane and toy car is shown in the table below.

Cost
80
60

(a) Form an inequality to represent the total cost for buying toy aeroplanes and toy

cars. [2]

(b) What is the maximum possible number of toy cars he can buy with $30? [2]

4 Tim has some red and blue marbles. He started with 350 blue marbles, which was 62.5%

of the total marbles he had. He lost some of his marbles and now, the total number of

marbles Tim has is 469. Given the percentage of red marbles is now 20% less than what

he had originally, find the

(a) number of red marbles left, [3]

(b) loss of blue marbles as a percentage of the original number of blue marbles. [3]

3
GMS(S)/Math/P2/EOY20l 1/ 1E

S Three angles, a, b and c are shown in the diagram. Given a= 2x· , b =( %x + 30} and

c =3(x + 20 r,find the values of


(a) x, [3]

(b) Lb. [2]

You are to state the necessary geometrical reason(s) in your answe~. [1)

6 A solid is made up of a cuboid and a cylinder. The cuboid has a square base of sides

12 cm and a height of 9 cm. The cylinder has a diameter of 21 cm and a height of 7 cm.

22 12cm
Taken = - , find the
7

(a) volume of the solid, [3)

(b) total surface area of the solid. [4]

21 cm

7 Consider the following number pattern.

Line 1 : I 12
Line 2: I+3 4 22
Line 3: 1+ 3 + 5 9 32
Line 4 : 1 + 3 + 5+7 16 = 42

(a) Write down the S1h line in the pattern. (1]

(b) Find the value of l + 3 + 5 + ...... + 175. [3]

4
GMS(S)/Math/P2/EOY201 l / 1 E

Answer the whole ofthis question on a piece ofplain paper,

8 Construct and label a quadrilateral ABCD such that AB = 8 cm, L.DAB = 55° ,

AD= 4.5 cm, AC = 9 cm and BC= 6 cm. Measure and write down the length of BD and

value of L.ABC . [6]

Answer the whole ofthis question on a sheet ofgraph paper,

9 Using a scale of2 cm to 1 unit for both axes, draw a graph of y =_!_ x - 1 for - 4 ~ x ~7.
2
[4]

x - 3 2 6
y m 0 n

(a) Find the values of m and n . [2)

(b) On the same graph paper, draw

(i) x= - 3, and (1)

(ii) y =2. [ 1]

(c) What is the polygon formed by y = _!_ x - I , x = - 3 and y = 2? [ 1]


2

END OF PAPER

5
Qn Answers Total
marks
1 13(2) + 3(0) + 4(- 1) [1] 2
= 22 (1]

2(a) 50h = 1500 3


[1]
2
50h = 3000 [1)
h =60cm [1]

2(b) Area of quadrant = 154 cm:.i [1] 3


2
Area of semicircle = 77 cm [1]
Area of shaded reQion' = 77 cm2 [1]

3(a) x represents no. of sets of toy cars ahd toy aeroplanes bought. 2
x (80) + x(60) = 140x [1]
140x::; 3000 or 1.40x::; 30 [1]
-·-
3(b) x::>2L4 [1] 2
Max. possible number of toy cars= 21 [11

4(a) Initially: 37.5% - 210 red marbles [1] 3


Now left: 80% of 210 [1]
= 168 red marbles [1]
. -
4(b) 49 blue marbles lost (1 J 3
49
- x l00% (1 J
350
=14% (1 J
---
5(a)
. (52 )
2x + x+30 =3(x +20) . [I] Reason: ext. L of fl [1]
4

3
- x=30° (I]
2
x=20° [1]

S(b) 2
Substitute value of x into ( %x + 30 )° (1)

L b = 80° [1]

6(a) Vol. of cuboid = 1296cmJ (1] 3


Vol. of cylinder= 2425.5cm3 (1 J
Vol. of solid = 3721.5cm3 [1 J

6(b) . 22 21 21 4
Surface area of 2 circular bases = 2(- x - x - ) = 693cm
2
[11
7 2 2
Qn Answers Total
marks
6(b) Surface area of 4 rectangular faces= 4(9x12)= 432cm [1]
22
Surface area of curved surface of cylinder= x 21x7 = 462cm2 [1]
7
Total surface area of solid= 1587cm2 (1]

7(a) 1 + 3 + 5 + 7 + 9 = 5 = 25 (1] 1

7(b) 175+1 3
No. of terms = - - [1]
2
882 (1]
= 7744 [1]

8 Construction steps using compass [2] 6


Labelling (1]
Accuracy [1]
Length of BO ± 0.1 cm [1]
Value of LABC± 1" [1]

8
9 m= - 2.5 [1] 9
n=2 [1]
Label x - &y - axis [1]
Scale: 2cm to 1 unit for both axes [1]
Label graph y = 1 x- 1, y=2 and x= - 3 [1]
2
1
Correct graph y =- x- l [1]
2
Correct graph y = 2 [1]
Correct graph x = - 3 [1]
Polygon formed = triangle [1 J

+::::;. ::·:·,.::·r ::·:·:··


-----~····. ; .....·:···· ··~·····•· ...

--· ·---··----------+---
... ;
.
:

:
-=-
;

~4

_j ----~ ··
!

9
Class Index no.
IName I I I
Jurong West Secondary School
End-of-Year Examination 2011 /50
. ..
Subject Mathematics (Part 1) 4016/1
Level/Stream : Secondary One (Express)
Date · · 10 October 2011 (Monday)
Time 0800 - 0915
Duration · 1h15 min

Instructions to candidates

1 Do not turn over the Question Paper unless you are asked to do so.

2 Write your name, class and index number in the space provided at the
top of this page.
f

3. Answer ALL questions.

4. Write your answers in the spaces provided on the question paper.


ALL essential working must be shown in the spaces provided.
Omission of working will result in loss of marks.

5. The number of marks is given in brackets [ ) at the end of each question


or part question .

6. NO CALCULATOR may be used in this Paper.

If the degree of accuracy is not specified After checking of answer script


in the question and if the answer is not exact , Checked by Si~:mature Date
the answer should be given to
three significant figures . Student
Answers in degrees should be given to
one decima l place .
Parent

This question paper consists of 10 printed pages, including this page.


JWSS End-of-Year Examination 2011 Mathematics (Part 1) Secondary 1 Express

Answer ALL the questions. [SO marks]

1 Evaluate the following

(a) 2(5 - 17) + (- 4) + 108 + 12

(b) ~+ (-%r '.

Ans: (a) _ _ __ _ __ [2]


(b) - - - - - - - - [2] '
2 list the following numbers in ascending order

3 5 66
0.6,
5' 7' 100

Ans: _ _ _ __ _ _ _ [1]

3 Given the formula C =2(x - y }, find the value of x when y =12 and C = 4

Ans: x = - -- - - - [2]

Page 2of 10
JWSS End-of-Year Examination 2011 Mathematics (Part 1) Secondary 1 Express

4 Find t if t : 100 = 12 : 25

.. I '

Ans: t= _ _ __ _ _ _ [1]

5 (a) Express 252 as product of its prime factors.

(b) Hence, find the smallest integer, k, such that 60k is a multiple of 252.

Ans: (a) _ _ _ _ __ _ [2]


(b) _ _ _ _ __ _ [2]

Page 3 of 10
JWSS End-of-Year Examination 2011 Mathematics (Part 1l Secondary 1 Express

6 (a)(i) Factorise ab - be

I '

'
Ans: (a)(i) _ _ _ _ __ _ [1]

(a)(ii) Hence, evaluate 153 x 479 - 479 x 53

Ans: (a)(ii) _ _ _ _ _ _ _ [2}

(b) Factorise 4eh + 12hk - e - 3k

Ans : (c) _ _ _ _ __ _ [2]

Page 4 of 10
JWSS End-of-Year Examination.2011 Mathematics (Part 1) Secondary 1 Express

7 Solve the following equations

(a) 7 - x = 3(x - 1) - (2 - x)

. ' .

Ans: (a) x =- - - - - - - [2] /


(b)

Ans : (b) x= - - - - -- [3]

Page 5of10
JWSS End-of-Year Examination 2011 Mathematics (Part 1) Secondary 1 Express

1
8 If ; < : , find the greatest value of m such that m is a prime number.

.• I t

Ans: m =- -- - - - [2]
9 lf - 1<p<8 and 6 ~ q < 10 and p and q are integers, find

(a) the smallest value of p + q,


(b) the smallest value of q - p.

Ans: (a) _ _ _ _ _ __ [1]


(b) _ _ _ _ _ _ _ [1]

10 Find the number of sides of a regular polygon whose exterior angle is 36°.

Ans : _ __ _ _ _ _ sides [2]

Page 6 of 10
JWSS End-of-Year Examination 2011 Mathematics (Part 1) Secondary 1 Express

11 In the diagram, the lines AB and CD are parallel. Given that L EFD = 90°
and L. CDF = 20°, calculate the values of x and y.
E

Ans : x = _ _ __ ____ [3]


y =_ __ __ _ [2]
Page 7 of 10
JWSS End-of-Year Examination 2011 Mathematics {Part 1) Secondary 1 Express

12 On the diagram shown below A. B and Care the vertices of a triangle.

y
- - - ~- ; --- ~- - - ~ - --~--- -:- -
- A- -- ~- - - I t I I
'
- -- 1 --- ~ ---~----~--- ~ -- - ~ - - - ~ - -- ~ - --
I I
I I

1
I

I
I

I
I I
___ 1 __ _ 1 _ _ _ J - - -~ - -- J -- -- ~ - -
l t
-- - L ---
I

---.' ---
' .
4_ ___ J ___ J ___ J

I
'
___ 1 ___ ! ___ } ____
__ __ L

t
___ L ___ L _ _ _ l' ___ J'

I
~--- ~- -- ~-- -l
I I I
__ _

__ _ ! __ _
I I I I I I I I
I I I I I I I

' . . -2
-- -r-. I I I I I I
--- ,--- , --- ,--- -r- ~ - ~-~ -r -- -~- - - ~ ---
I I

. .
- -- ~ --- !s. ---L-- -
0

0
---~--- ~ --- ~- - - - L ---L - -- L---~--- ~ ---
t •

'
'

' r
-8 0
t
:s
-- - 'r ---
I I I I I I I
---, - - - , --- , -- -- ~-- - r ---r- - - r --- 1 - --

' -2
- -- 1-- --
I I I f I I

-- - ~ - - - ~ - - --·---- ·-- -- ~ --- ~ - -- · - - - ~ ---


___ L _ _ _ I I '
I '
I '
I I I I
___ J ___ J __ _ J _ _ __ L - - - L -- -L -- - l - -- l - - -

' 1

' -4
- - -r· --
I I I I I
---1-- -~--- ~ -- - -r---r ---;---r--- 1 -- -
I I I

--- t --- ~ -- - ~ - - - 1 ----1-- --~ -- --- 1 ---~----1--- - ~---~---~---·---~---

' ' ' I I I

(a) Write down the coordinates of point C .

(b) Find the area of triangle ABC.


/
(c) Find the coordinates of a fourth point, D, such that ABCD is a
rectangle.

Ans (a) C( _ _ _ _ ) [1]


(b) _ _ _ _ __ _ units2 [2J.
(c) O( _ _ _ _ _ ) [1]

Page 8 of 10
JWSS End-of-Year Examination 2011 Mathematics (Part 1) Secondary 1 Express

13 (a) Construct f'lQRS in which QR= 5.5 cm, RS= 10 cm and [3]
QS= 5.Scm.

(b) Construct the angle bisector of L. RQS. [2]

(c) Classify f'lQRS according to the number of equal sides it has.


:. l '

Ans : (c) _ _ _ _ __ _ triangle [1]

Page 9of 10
JWSS End-of-Year Examination 2011 Mathematics (Part 1) Secondary 1 Express

14 The following diagram shows a series of patterns formed by dots.


• •
••• •
• ••••
• •
• '.

(a) Draw the 4th diagram of the sequence in the space provided above.[1]
(b) Complete the table below. [2]

Patte rn Formula Number of dots


1 1 1
2 4+1 5
3 4+4+1 9
4
5

(c) Write down a formula for the number of dots in the nth pattern.
(d) Find the number of dots in the 25th pattern.
( e) Find the nth pattern that has 37 dots.

Ans : (c) _ _ _ _ _ _ _ [2]


(d) _ _ _ _ _ _ _ _ [1]

(e) _ _ __ _ __ [1]

End of Part I

Page 10of10
JURONG WEST SECONDARY SCHOOL
End-of-Year Examination 2011
Mathematics Part 1
Secondary 1 Express

Answer Key

On Answer
..
1 a 15
b 1]_
16
2 3 66 5
s· 0 6
100 • · • 7
3 x= 14
4 t= 48
5 a 252 = 2L X 3L X 7
b 21
6 ai b(a-c)
aii 47900
b 4h - 1)(e + 3k}
7 a 2
x=2 -
5
b x=24
8 23
9 a 5
b - 2
10 10 sides
1~
11 L x= 70°, L. V = 110°
j 12 a C(-2, - 5}
b 18 units.£
c D (2, - 2)
13 a See marking scheme
b See markinq scheme
c Isosceles triangle
14 a ••

•••••••
••
r --

b Pattern Formula Number of dots
4 4+4+4+1 13
5 4+4+4+4+1 17
c Tn= 4n - 3
d Tzs =97
e n = 10
Class Index no.
IName I I I
Jurong West Secondary School
/50
End-of-Year Examination 2011
' .

Subject Mathematics (Part 2) 4016/2


Level/Stream : Secondary One (Express)
Date 10 October 2011 (Monday)
Time 1030 - 1145
Duration 1h15min

Instructions to candidates

1 Do not turn over the Question Paper unless you are asked to do so.

2 Write your name, class and index number in the space provided at the
top of this page.

3. Answer ALL questions.

4. Write your answers in the spaces provided on the question paper.


ALL essential working must be shown in the spaces provided.
Omission of working will result in loss of marks.

5. The number of marks is given in brackets [] at the end of each question


or part question .

6. Silent, Electronic Calculator may be used in this Paper.

If the degree of accuracy is not specified After checking of answer script


in the question and if the answer is not exact , Checked by Signature Date
the answer should be given to
three significant figures . Student
Answers in degrees should be given to
one decimal place .
Parent
--

This question paper consists of 10 printed pages, including this page.


JWSS End-of-Year Examination 2011 Mathematics (Part 2) Secondary 1 Express

Answer ALL the questions. rso marks]


.J5600
1 Evaluate , correct your answer to 2 significant figures.
41
(4.1)3 - 3

. ' .

Ans: _ __ _ __ _ [2J

2 Solve the following equation


w 3
- +57 = - - W
5 4

Ans: _ _ __ __ _ [2J

Page 2 of 10
JWSS End-of-Year Examination 2011 Mathematics (Part 2} Secondary 1 Express

3 Wayne bought 3 bottles of wine at $12.50 per bottle, 5 packets of flour


at $3.50 per packet and 3 trays of eggs at $2.80 per tray. The GST levied is
7%.

(a) Calculate his total bill, inclusive of GST, correct to 2 decimal places.

(b) If the GST levied is increased to 10%, calculate the percentage


increase in his total bill, correct to 3 significant figures.

Ans : (a) $_ _ _ __ _ [3]


(b) ~~~~~~~~-
% [3]

Page 3 of 10
JWSS End-of-Year Examination 2011 Mathematics (Part 2) Secondary 1 Express

4 In the diagram, AB is parallel to CD and EF is parallel to DB. Given that


L ABO = 35°. Find the values of x and y in the figure below, stating the
reasons clearly.

'.

0
y

Ans: x: _ _ __ _ _ __ 0
[2]
Y : _ _ _ __ _ _ _o [2]

Page 4 of 10
JWSS End-of-Year Examination 2011 Mathematics (Part 2) Secondary 1 Express

5 T aufiq drove from Pasir Ris at an average speed of 66 km/h. Haron left
Pasir Ris 20 minutes later and drove at an average speed of x km/h. If
Haron overtook T aufiq in half an hour. Find

(a) the distance Taufiq travelled before being over taken by Haron.

(b} the value of x.


'.

Ans : (a ) _ _ __ __ _ _km [3]

(b) km/h [2]

Page 5 of 10
JWSS End-of-Year Examination 2011 Mathematics (Part 2) Secondary 1 Express

6 In the diagram, 4 circles are drawn to fit into a rectangle. If the radius of
each circle is 7 cm, find

(a) the length and breadth of the rectangle,

(b) the area of the shaded region,


.-. ' .
(c) the perimeter of the shaded region.
22
(Take rr= - )
7

,
/

Ans: (a) cm [2]


(b) cm 2 [2]
(c) cm [2]

Page 6 of 10
JWSS End-of-Year Examination 2011 Mathematics (Part 2) Secondary 1 Express

7 The size of 3 interior angles of a 9-sided polygon are 116°, 108° and 130°
respectively. The size of each remaining interior angle of the polygon is x 0 •

(a) Find the sum of interior angles of the 9-sided polygon.

(b) Hence, calculate the value of x.


..

Ans: (a) ~~~~~~~~~


0 [2]
(b) [2]

Page 7of10
JWSS End-of-Year Examination 2011 Mathematics (Part 2) Secondary 1 Express

8 A solid prism whose cross section is a trapezium is moulded into a solid


cylinder of radius 10 cm. All dimensions given in the diagram are in
centimetres.

(a) Find the volume of the cylinder.

(b) Find the height of the cylinder.


..
( c) The mass of the solid is 1.2 kg. Find the density of the solid in g/cm3.

(Take rr = 3.14)
JO

Ans: (a) --~~~~--


cm3 [2]
(b) _ _ __ _ _ _cm [2]
(c) _ _ _ _ _ _ _ g/cm3 [2]

Page 8 of 10
JWSS End-of-Year Examination 2011 Mathematics (Part 2) Secondary 1 Express

9 Students of a class were asked about their favourite sports in a survey. The
1
results are represented in the pie chart below. Given that of-the students
6
like swimming, find

(a) the value of x,


• I ' ,.
(b} the percentage of students whose favourite sports is badminton,

(c) the number of students in the class if 4 more students prefer soccer
to swimming.

Soccer

Badminton

Ans: (a) _ ______


' [3]

(b) _ __ _ _ _ _ _ % [2]
(c) _ _ _ __ _ students [3}

Page 9 of 10
JWSS End-of-Year Examination 2011 Mathematics (Part 2) Secondary 1 Express

10 Answer the whole of this question on a sheet of graph paper.

(a) Copy and complete the table of values for y=x - 4 [1]

x 1 4
y - 3 0 4
:. I •

(b) Using a scale of 2 cm to represent 1 unit on both axes, draw the


graph of y =x - 4 for 0 :5 x :5 8 [2]

(c) Write down the gradient of the graph. [1]

(d) From the graph, find the value of y when x =2.5 [1]

(e) The point (p, - 2.5) lies on y =x - 4. What is the value of p? [1]

{f) Write down the coordinates of they - intercept of the graph. [1]

Page 10 of 10
JURONG WEST SECONDARY SCHOOL
End-of-Year Examination 2011
Mathematics Part 2
Secondary 1 Express

Answer key

Qn Answer
.
1 1.4
2 w =-60
3 a $67.84
b 2.80%
4 L x =145°, L y =215°
5 a 55 km
b 110 km/h
6 a lenqth = 56 cm, Breadth = 14 cm
b 168 cml
c 31 6 cm
7 a 1260°
b x = 151
8 a 3024 cmJ
/
,_ b 9.63cm
c 0 .397 g/cm~
-
9 a x= 50°
b
41 3. %
3
c 36 students
110 a x 0 4 8
y -4 0 4
b Refer to marking scheme
c 1
d y = -1.5
e p = -1.5
f (0, - 4)
Class: Candidate Name: Candidate Index Number: I

SHUQUNSECONDARYSCHOOL
2011 End-of-Year Examination
Secondary 1 Expre·ss

MATHEMATICS
Paper1 1O October 2011

Candidates answer on the Question Paper 1 Hour 30 Minutes

INSTRUCTIONS TO CANDIDATES

Write your name, class and class index number in the spaces at the lop of this page
and all the work you hand in.
Write in blue or black pen.
You may use a pencil for any d iagrams or graphs.
Do not use staples, paper clips highlighters, glue or correction nuid.

Answer all questions.

If working is needed for any question. it must be shown with the answer.
Omission of essential working will result in loss of marks.

You arc expected to use a scientific calculator to evaluate explicit numerical


expressions.

If the degree of accuracy is not specified in the question. and if the answer is not
exact, give the answer to three significant figures. Give answers on degrees to one
decimal place.

For n. use either your calculator value or 3.142, unless the question requires the
answer in terms of n.

At the end of the examination, fasten all your work securely together.
The number of marks is given in brackets [ ] at lhe end of each question or part
question.

The total of marks for this paper is 50.

This question paper consists of 1 0 printed pages.

[Tum over
2

Answer all questions.

1. Evaluate

V12s - 3
a) ----sr--
b) 6 2 + J9x 1.6 .

Ans: a) _ _ _ _ _ [1 1
b) _ _ __ _ (1J

2. Write down the next term for each sequence.

a) 1,5,11,19, 29, __ .

b) -1 , -4. -9. -16, - - .

Ans: a) - ·- - - - - (1]
b) _ [I)

3. If p : q = 5: 3 and p : r = 2: 7, find p : q : r.

(2)

SQSS EOY 20 11 Mathematics


3

4. Arrange the following numbers in ascending order.


a) 9, 7, - 5, - 7
21
b) 50 ' 0.4, 0.44, 47% .

'.

Ans: a) _ _ _ _ _ _ _ _ _ (1)
b) (1)

5. The temperature in a town in a day al noon was 37 °C. The temperature


drops to -3 °C at midnight. Find
a) the difference between the two temperatures,
b) the temperature which is exactly half-way between the two temperatures.

Ans: a ) ,_ _ _ _ _ °C (1)
b) _ _ __ __ °C [1)

6. Sharafi worked part time in a fast food restaurant and was paid $22.80 for 4
hours. During the December holidays, he worked 128 hours. How much did
he earn during the December holidays?

Ans:$_ __ _ _ __ _ [2)

SQSS EOY / 011 f\ta thcmutics


1
4

7. The diagram below shows a parallelogram with an area of 180 cm~ and base
15 cm long. Calculate the height of the parallelogram.

/ ''

/\ns: _ __ __ _ _cm (2)

8. A gift shop uses 1155 boxes of chocolates, 462 botues or wine and 693 tins of
cookies to make as many 91rt hampers as possible. Each hamper has the
same number of boxes of chocolates, the same number of bottles of wine and
the same number of tins or cookies. What is the greatest number of gift
hampers that can be made?

Ans: _ __ _ _ __ _ (3)

SOSS EOY 2011 Malhcm.abcs


5

9. A regular polygon has n sides. The size of each interior angle is seven times
the size of each exterior angle.
a) Find the size of each exterior angle.
b) Calculate the value of n.

'.

Ans: a) _ _ _ _ _ __ 11]
b) [2)

10. Given that - 5 < x 5; -2 and I S y :5 4, find

a) the greatest possible value of x + y,

b) the grealest possible value of (x - y)2 •

c) the smallest possible value of x .


y

Ans: a) _ _ __ _ _ _ (1J
b) [1)
c) [1)

SQSS FOY 20 11 P~tnthnmnlics


6

11. Factorise
a) 2? + 6rh.
b) ac - 3c + 2ab - 6b.

: .

Ans: a) - - - - -- - [1) ,i
b) [2)

12. Solve the inequalities


a) 3y ~ 14,

2 1 7
b} x - 3> x - -· .
3 3 10

Ans: a) _ __ __ __ [1)
b) (2)

SOSS EOY 2011 Mathematics


7

13. Calculate the values of x and y in the following diagram .

. '.

Ans: a) x = - - -- - [2)
b) y = [2)

14. Solve the following equations


a) 2x - 7 =-2(x-2),
b) 3(4a + 5) = 0.5a .

Ans: a) x =- - -- - [2j
b) a= [2]

sass EOY 2011 Mathematics


8

15. The following is a receipt or Sean's dinner at Lay Garden's Restaurant.


a) Calculate the amount or service charge.

b) The GST is taxed on the sum or (I) and {II) of the receipt. Calculate the
amount of GST incurred.

c) Hence, find tho Grand Total which Sean has to pay.


'.

L:iy Garden's Retaurant

I Fish and Chips $19.80


I Fruits Salad $8.90
I Cola drink $2.50

Total $31.20 {I)


I 0% Service C harge (II)
7%GST
i

Grand Total

Ans : a)$_ ----~ [1}


b) $ [2]
c) $ (1)

sass EOY 2011 MJlhernatics


9

16. The following diagram shows a solid that takes a shape of half a cylinder of
diameter 20 cm and height 8 cm. Calculate
a) the volume of the solid,
b) the total surface area of the solid.

..

, ii> - - .. - '

Ans: a) _ _ _ _ _cm 3 [2]


b) cm 2 [2]

sass Fov 7011 ,,1.>thematocs


10

17. In the space below,


a) construct a triangle PQR in which PQ = 8 cm, PR= 10 cm and
L QPR = 75"-, [3)
b) construct the perpendicular bisector or PQ, (1)
c) construct the angle bisector of LPQR .
.. . (1]

F:nd of Pa per

sass EOY 20 11 Ma ll1ematics


J\ nswer s

r- -1~: 0.04
b. 40.8
f1 .
2a. 41
b. -25
3. JO: 6: 35
4a. -7, -5, 7, 9
b. 21 i50, 0.44, 0.4, 4 7%.
5a. l 40
b. I 17
6. I

$729.60
7. 12
-' -
i 8. 231
r
- --- -- -
.r 9a .
I
22.5° i
' h. 16
IOa. 2
b. 8J
c. -5
1 I a~i-2r(r-·I 3h)

I b. ' (a-3)(c+2b)
L__ -
; 12a. 4.67
- - - -------
I

'--~·--
. 13a. 6.9
26 - - --

b. 108
1 - --1-- - -----·-- - ---I
14a. 2.75
b. + J.30
! 1sa. 1$3.12
I b. . $2.40
c. $36.72
l6a. 1256.8
b. 725.56

SOSS EOY 2011 rv13thern<llics


Class: Candidate Name: Candidate Index Number:

SHUQUN SECONDARY SCHOOL


2011 End-of-Year Examination
Secondary 1 Express ·

MATHEMATICS
Paper2 11 October 2011 ·

Additional Material: Answer Paper 1 Hour 30 Minutes

Graph Paper

INSTRUCTIONS TO CANDID A TES

Write your name, class and class index number in the spaces at the top of this page
and all the work you hand in.
Write in blue or black pen.
You may use a pencil for any diagrams or graphs.
Do not use staples, paper clips highlighters, glue or correction fluid.

Answer all questions.

If working is needed for any question. ii must be shown with the answer.
Omission of essential working will result in loss of marks.

You are expected to use a scientific calculator to evaluate explicit numerical


expressions.

If the degree of accuracy is not specified in the question. and if the answer is not
exact, give the answer to three significant figures. Give answers in degrees lo one
decimal place.

For Jt , use either your calculator value or 3.142, unless the question requires the
answer in terms of :1r .

Al the end of the examination, fasten all your work securely together.
The number of marks is given in brackets []at the end of each question or part
question.

The total of marks for this paper Is 50.

This question paper consists of 5 printed pages.


2

Answer all questions.

1. a) Express, correct to two significant figures,


i) 208.897 (I]
ii) 0.020859 [I]

.
b) Hence estimate, . .f.1cant fi1gure, th e va Iue o f (208.897)2
correct to one s1gni [ J)
0.020859

2. a) Express 180 and 792 as a product of prime factors. Leave your answers in index
notation. 12]

b) Hence, find
i) the LCM of 180 and 792, (I)
ii) the least value of n such that 792n is a perfect cube. [I)

3. Study the sequence of shaded and unshaded triangles in the diagram below.

A .
'

2nd p~111c:rn -1th pa11C"111

a) The information from the pattern sequence is tabulated below.


Find the values of x and y. (2)

Pattern (n) J 2 3 4 5 6
Total numbe1· of triangles (1) 4 9 16 25 x y

b) Write down an expression in terms of n. for the total number of triangles in the
nth pattern.
[I)
c) Hence, find the number of triangles when n = 25. (I)
d) Find the value of n when the pattern is made up of 400 triangles. [I)

SQSS EOY ?011 lvl(lthc mntic,s


3

4. In the diagram, BDC, AHGB, AJJC, FIJII.::. EJD and FGD are straight lines.
Calculate the value of
a) x, (I]
b) y,
c) z.
0)312° {2]
[2]

I · -E
F_
-~~~--3/
H _L--r-·---~
~
J7"
\
R D
c

- - . -- -- -- - -- - -- - - - - - - - - - - -- - -
5. A track and field stadium consists of a central field surrounded by a 6 m wide running
track (shaded region). The central field has two semicircular ends, each with radius
30 m and two straight sides. The perimeter of the central field is 350 m.

+ Central Field

Find

a) the length of each straight side of the central field. [2]


b) the area of the central field, [2}
c) the area of the running track. [2}

SOSS EOY 201 l tv1alh~ffli:llic~


4

6. a) Expre ss
3
-- ~ as a single fraction in its lowest term. [3)
x+2 x- 1

2 3
b) Solve - - =--. [3]
x- 1 2x+1

7. A group of students vyere surveyed to determine which countries they would like
to visit the most. Their choices w ere represented on a pie chart as given below. -·

---------......
Japan

Korea
.___-·
2x --
3x
liSA

Aus1ralia

....___ - ------

a) Find the value of x. [2)


b) Hence, calculate the percentage of the group who would like to visit Korea the
most. [2)
c) If 400 students would like to visit Australia the most, find the total number of
students surveyed. (2)

sass cov 2011 M athe matics


5

8. a) An aircraft Oew a distance of S-756 km from London to Los Angeles afan


average speed of v km/h. Write down the expression for the time in hours
that it took for the journey.
111

b) The aircraft returned non-stop by the same route at an average speed of


3v km/h. Write down lhe expression for the time in hours that it took for the
return journey. [I)

c} Given that the difference between these.two times was 15 hours, form an
equation in v. Show that the equation will reduce to 17512 = 45v. [2)

d) Hence. solve this equation. [ 1)

- e) Thus. find the time in hours the aircraft took on the return journey. (I J

9. Answer the whole or this question on a sheet of graph paper.


The table of values for the linear function y = -3x + 2 is given below.

-2 0 4

a 2 - 10

a} Find the value of a. (1)


b) Using a scale of 2 cm to represent 1 unit on the x-axis and 1 cm to represent

1 unit on the y-axis, draw the graph of y = - 3x + 2 for the values of x from

- 4 to 4. [3]
c) From the graph, find the value of x when y = 3. (I)

d) Draw the line y = 8 and state the gradient. [2)


e) Find the coordinates of the point when the two graphs intersect. [l)
f) Find the gradient of the line AB where A is (1 , 6) and Bis (-2, -4). [J)

End of Pape1·

SQSS EOY 2011 Mathematics


Answers
- - - - ·-··-

JI. I0.02
"- 2a.
b.
bi
-1-;~ ~2 ?.) .,t. · --
2 x .) x 5, _ x 3 x 11
3960
-

II. ; 363
--· -- - - - ·
3a. 36, 49
!
b. (n + I )2

c. 676
d. 19
-
4a. 56
b. 73
c. 105
5a.
-
80.7 4
b. 767 2.2
c. 221 3.112
- - - - - ---- - - - -
6a. (-1 I - x)/[(x + 2)(x - I)]

b. x~ -5
·--
7a. 28
b. 15.6%
c. 1200
8a. 875 6/v.
b. 875 6/3v
c. 875 6/v - 8756/3v = 15
d. 389
e. 7.50 h
9a. a ~ 8
b. graph
c. -0.3

d. 0
e. (-2 , 8)
f. I; 3.33
.

sass eov 201 1 ~ArJth cmatics


~lame.
- -- - -- - - - - - ( Class· _ _ _ __

WHITLEY SECONDARY SCHOOL


A Caring and Learning Community

Discipline •Integrity •Respect •Responsibility

END-OF-YEAR EXAMINATION 2011


SUBJECT Mathematics Paper 1

LEVEL Sec 1 Express

DATE 4 October 2011

DURATION 1 hr1 5min

SETTER(S) Jaime Ong

Eddie Yong, Yong Kwee Fah,


VETTER(S)
YapW. M.

READ THESE INSTRUCTIONS FIRST

Wrile your name and index number on all lhe work you hand in.
Wrile in dark blue or black pen on bolh sides of lhe paper.
You may use a pencil for any diagrams or graphs.
Do nol use staples, paper clips. highlighters. glue or correction fluid.
Answer all lhe questions on writing paper provided.
If working is needed for any question it must be shown wilh the answer.
Omission or essential working will resull in loss of marks.
Calculalors should be used where appropriate.
If the de91ee of accuracy is not specified in the queslion, and ii the answer is not exact, give
the answer to three significant figures. Give answers in degree to one decimal place.
For n. use either your calculator value or 3.142. unless the question requires the answer in
terms or ,7.

At lhe end of lhe examination. laslcn all your work securely togethe1.
The number of marks is given in brackels [ Jal the end of each qucslion or part question.
The total marks for this popcr is 50. ·

This p<lpcr con~1sls of2 printed pages. indu<ling lhe covt:1 pngc.
I. Corrccl 0 .099841 to
(a) J significant figures,
(b) 2 dt.'Cimal place:;.

Answer : (a) _ _ _ _ 111


(b) 111

2 Which or the following two munbers are not prime numbers'!

I, .5, 17, 19, 33, 37

Answer: & 121


.>. Express \ · x s 2 on the numbrr line below:

+ ·.I._1---1
1 l"--1 I I I I I I I I-- ~
-9 -8 -7 -6 5 -4 -3 -2 -I () 2 3 4 5 6

fl I

-t Given th:u I~ r - I LJ 044 - 2-.


4.
I
(2)

..
Fill in the boxes with '+', '-', '><' or '
so that the res ult 1s 1.22 (corrected 10 3
s1g11!fica111 figures} .

Write down the next two terms in the 11umher scquchcc.

I l
(~) I. 2, 2, f 1I
2 - -- --
(l>) 10.5, 8, 5.5, - 3~ [I]
0
(>. lf1hc density or the liqmd is 90 kg/111', calculate the mass of the llquid in a tank with a
volume of 40 m'. Giv..: your an~wer in kiloemms.

Answer: _ __ _ _ kg [!}

7. (i1ven that the area of the parallelogram below is 45 cm2, calculate its height.

/. 7
I :
8.2cm - J

Answer: - - - - - - - cm [I]

8. (a) Convert I 2nl to crn7 .


(b) Convert 5m 3 to cm' .

/.\11swcr : (<1) _ _ _ _· _ ___ crn~[ IJ

.. _I ..
(b) __,_ _ cm3 {I)

9. Find the Highesl Comnwu Factor (!Kl') and Lowest Comm<>n 'vlultiplc (LCM) vf

&

Leave your ans\vers tn index notation.

Answer: HCf - [11

LCM - - - - - ·-- - - - - - (I)

IO. Evaluate lht~ follo wing. giving y<>ur answers in tcnninat ing or recurring decimal fom1.

"j
(ii) (2.85 ) x (- 0.8)
II
i2.4
(b) i/-
~ 0.4
·

;\ns'.vcr: (a) _·-·--·- - -- - ' l]

.. 4 - -
(b) Ill
I I. Express 22 rninutc~ as a percentage of3 hours.

Answer:

12 . Showing your steps clearly and without using your calculator, evaluate~+ 3. x 1
3 9 4
I .cave your answers in the lowest terms.

Answer: - - - - - -- - - [2}

13. ..., .
,-.stimate thc va lue o fJI02 + 112.9 . Icavmg
. your answer .m two s1gn1 •1ca.nt .1gures.
1 1
10.55x0.94

..5.
Answer: - - - - - - - - - [2)
2
1·1. Solve the cqoation · (2x ·· o) = 0.
3

An~-wer: x= - 121

I )- . s·imp1·1· . -I x + J .6y + -2 x - y.
1 y t l1c expression
4 J

Answer: [2J

16. John's tuition lesson starts al 10:30 am and ends al 1:05 pm. He has a 20 minutes break in
-
between . Dctcnninc the lime in hours and minutes John spends on tuilion excluding !he
break.


,1\ns\1i.:cr: _ _ _ h _ _ _ mins 121
17. Give n the algebraic equation W =- x + gz' , whal is 1hc vnfue of W when x - 3, l' ·- -·2 a11tl
~ - 2~ -

Ans wer: W ~-
- -- - -- (2 ]
- 2(x+y) x
I K. Express - - ·- - -- as a single fr.iclion in its simples! fonn.
6 3

Answer: _ _ _ _ _ __ [2]
. . • '.\a c
19. Snnp111y -
4ce

/\ns\\'t~ r: _ _ 121

- • 7 ••
20. A pipt: can fill a tank at the rate of250 litres per h-Our. A tank has a capacity of900
7
litres. Find the time needed to fill of the tank.
8

Answer: _ __ _ hours 121

2 I. Expand and simplify JO - 3(2\\\1' t 3).

Answer: _ _ _ [2]

.l2 (a) Soh·e the inequality 2(2 ~ ·') ~ - 13.


(b) J lcnce, write down the least possible value of x ifx is rationnl.


Answer: (a) (2)

(h) [I]
. ~ ..
23. (a) Express 1.25 : } in its simplest form
16

(b) The ratio orthc numb..:r of stamps Jason has to what Melvin has is 3 · 4.
The ratio nf 1k number of s1amps Melvin has 10 what Harry has is 5 : 6.
If Jason and Melvin ll>gethcr have 595 s tamps, how many stamps does Harry have?

/\nswcr: (a) . - - · ·- : _ .· - ·· fl J
(b) - _Stamps {1J

24. Using prime factori7ation, find


(a) the square mot of 484 ,
(b) the cube root of2744.

/\nswcr: (a) _ _ 121


(b)_ 121
'

-- End of l'apcr -

-. 9 . •
Answers

Ql (a) 0.0998

(b)0.10

Q2. 1&33

Q3.

-I I I I I
-9 . B -7 -6 -5
1- • + + + + + I I I I
-4 .3 -2 -1 0 1 2 3 4 5 6

Q4. - and x'

) 5 _',
Q5. (a 2'
(b) -0.5, 2

Q6. 3600kg

Q7. .S.49 Clll

2
<.)8. (a) I 20 000cm

(b) 5 000 000 cm'

Q9 JICF - 2 x J'

LCM - 2 1 x 3 1 x 5 x 7 x 13

QIO. (a) - f.770'>

(b) 1.82

. 10 . -
QI J. 12.2

Ii
Q1 2.
1~

Q13 II

Ql<l, 3

II
QIS. -- x+0.6y
17

QJ6. 2 h I S min

-
Q1 7. 65.S
(2.x I }')
QI&.
3
t•d
Q19. ,,.,

()20. 1. 1S hours

Q2 I. I (1 11y

()22. (a) x ?:'. - 8.5

(b) -8.5

Q23. (a) 20: J

(h) 408

Q24. (:i) 22

(h) ,,,

11 •
Name: Class: - - - -

WHITLEY SECONDARY SCHOOL


A Caring and Learning Community

Discipline •Integrity ·Respect •Responsibility

END-OF-YEAR EXAMINATION 2011


SUBJECT Mathematics Paper 2

LEVEL Sec 1 f.lpress

DATE 7 Octobe r 2011

DURATION 1 hr1 5 min

SETTER(S) J ai me Ong

Eddie Yong, Yong Kwee Fah,


VETTE R(S)
Yap W . M.

R E AD THESE INSTRUCTIONS FIRST

Write your name imd index number on all the work you hand in.
W rite in dark btue O< black pen on both sides of the paper.
You may use a pencil for any diagrams or graplis
Do not use staples, paper dip:;. highlighters, glue or correction lluid.

Answer all the queslions on writing paper provided.


If working is needed for any question it must be shown with the answer.
Omission of essential working will result in loss of marks.
Calculators should be used where appropriate.
If the degree of accuracy is nol specified in the question, and if the answer is not ex;:ict. give
the answer to three significant figures. Give answers in degree to one decimal place. ·
For 7T. use either your calculator value or 3.142, unless the question requires the answer in
terms of 7T.

At the end of the examonalion. fasten all your work securely together.
The number of marks is given in brackets [ I a l the end of each question or part question.
The tot<ol marks for this paper is 50.

This paper COIJSISIS or 11 pnnted pages inducltng the t:OVCI pag,e


I. Given the equation of a line 1s y = •3:c + 2, write down the gradient of the line.

Answer:
- -- -- --- [I ]
2. W1i1e down the nth term ofthi~ ~cqucm:c.

I. 4, 7, 10, n. 16, 19.

Answer: _ [2)

~. Factonse the following completely:

(a) I Say I 20by.


(hl 9/u - 3rv - 3ur 1 n-

Answer: (a) [ l)

{b) l '.! J
4. A reclangular swinuning pool, 16 m long and 14 m wide, is fully filled with waler. II i~ I rn
deep at one end and slopes uniformly down to 3.5 mat the olhcr end.

16 m
~~-=·-=-=====-=::::..=::=== - - - - -+
l 1 Ill

/
- -- · - -
.: --~
! ~-..=.·;;..;··-
.. -- -~----"-
·---- -~-<

I Im
1.5 m ,- ---~--
-+-
-~=====:::;:::::::==---
16.2 m
(a) Delermine the cros~·sectional area of the pool.
(h) Citlculule the capacity of the 1>001.
(c) Calculale lhe tolal surface area of the pool in conlact with the water.

Answer: (a) m'llJ


(b) __ m3 [ I]
- - -
(c) - m' (21

. }
5. Nancy bou~ht a watch for $JJ5 inclusive of a 7% good~ and services tax. Calculate the
amount of tax to the ncan:st cent Nancy paid.

Answer: $ - -- - 12J

6. A metal rod has a diameter of 4.2 crn. Given that the volume of the rod is 225 cm3,
determine ilw lcngih as well as !he totul cxicrnal surface area of the rod. (Take rr - 3.142)
'

.,._._____
4.2 .._..
CO'I

--
----
I

____ ___...,..

Answer: l.englh _cm [21

Surface 1\1ca - __ _ crn' 121

·I
7. Jao;on travels by car at an average speed of72 kmib and completes a journey in 4S minutes_
On his return joumcy. he travels at an average speed of 45 km/h.
(a) How Jong does the return journey take?
(b) l f he rested for 35 minutes before making his return journey, what is the average
speed of his entire journey?

Answer: (a) _ _ _ __ _ hour [IJ

(b) ---~ km/h [2]

5
8. NOPQ is a parnlldogram in "h1d1 the diagonals intersect at M. Risa point on OP such that
(JR = ()P. It is given that L ON/' 25°' LNQM = 70° ancl NOi' - I 05° . Stating your
n:asons clearly, calculate the vahu; or
(a) /J'NQ. N\ .. 0
I

\2s
\\
( l>) LRQO.

' H
,'· p

Answer: (a) 12}


(h) _ __ _ [31

"
9. Mr Lin bought 25 markers. Some arc red markers at 30¢ each and the rest arc blue markers
al 45¢ ea<:h. The totai bill for the 25 markers wa.~ $9.45.
(a) If there are x red markers, write down and simplify an expression in x for
(i) the cost of the red markers,
(ii) the cost of the blue markers.
(b) Form an ClJUation in x and solve it lo fi.nd tJ1c number of blue markers.

Answer: (a) (i) $ - -· -. - · - f I]

(ii)$ Ill
(h) _ ·_ hlut: markers (2]

.i
I 0. A regular hexagon with six ~ide:; is shown below.
(a) C:alcolatc the value oft he sum of all interior angles within the hexagon,
(h) Calculate the value of' each e.~tcrior angle of the hexagon.
(c) Given that L Jl;J C ~- 32°, find the valu<: of x, the exterior ru1gle of triangle ABC.

-·-\
- \

\ x
B c

Answer: (a) - - · - - - · -- [2]

(b) - - [2)

(c) _ _ _ _ fl]
I I. 160 students wen: asked to narne their favourite subjttts. The pie chart below shows
the result of this ~urvey.

(a) Calculate the value of x if J 6 stud~TilS chose Art a.~ their favourite subject.

(b) I low man y more students chose Geography msli:ad of Scicn,·c as their
favourite ~ubject'I

/
----..__.....
.,......

GfOGIVI Pll V
81 °

Fl'vUSll

"""'""" v
-~---- - ---·-
ART

Answer: (u) x = _ _ _ _ __ (2)

(b) _ _ _ students (21

<)
12. Complete the following in a smgle diagram

(a) Construct and label rhombus l'QRS such that PQ is 7cm and / PQR is 60". (2}
(b) Constmct the perpendicular bisoctor of QR. [2}
(c) Construct the angle biS<'~tor of/ QRS. [2)
(d) The perpendicular bisector of QH cuts the the angle bisector nf /QRS at X.
\.1easure and wri te dow11 the l~ngth of RX.

Answer: (d) RX · _ _ cm fl]

• I0 •
J 1. With n:fc rence to the grid below,
(a) Wri te down the coordi nate~ of A .
(b) Po int B has cwrdinates (-2. - 1} and point C has coordinates (4, 5).
Cal>.ulate the gradient o f th~ lme DC.

5 \< c ... ;
!
1
4 - --·;· .L -· r--·-'---- .,_ -·--1
3 1
2 - -----

., -- .f - --1-·- . .;- ·- J

.__.__ _ _ _ 7_.j ;
2 -I 0 2 3 4 5 6

B
x -i -------
-: - -
-2
·-'

Answ er: (a} 1l = ( ) (I (

(b) _ _ __ _ [2]

. II
14. Complett: the table & draw the li ne for y ~ x + 2 on the grid below:

[ I)

)'

...... 5

· ~ ··• •·•

. ................... .............:........... ' .. ; ... ...


~

[2)

.,

... End of Paper -·

. I? .
QI. -.>
Q2. Jn 2

Q> (a) Sy (Ja + 4b)


(b) (111 ·- v)(}r - r)

?
Q4 (a) 36 m

504m'

-
(b)

(c) 362m 2

Q5. $8.&:l

Q6 (a) 16.2 cm

(h) 242 c1n 2

()7 (ii) l.2 hour

(bJ 42.6 km/h

Q8 (a) 50"

(b) 5"

Q9 (a) $0.3x

(b) $0.45x

(c) 13 blue nrnrkers

QIO(<t) 720°

(b) 60" •

(c) 122°

- 1:; .
QI I (a} r 36

(bl ~

Ql2 (d) RX 7 ~m

Qll(a) A - (6.~)

(h) Gradicnl

....

'


Index
Class Number
Caindid ilte Name :=J
YUHUA SECONDARY SCHOOL


End-of-Year Examination 2011
Secondary One Express

MATHEMATICS 4016/01
PAPER 1

TI !\IE I hour I 5 minutes

Candidates answer on the Question Paper

RF:AI> TllESE INSTl{ UCflONS FJRST

\.Vi 1t1" your name, class and index number on the work you hand 111.
W11tc 111 dark blue or black pen.
Y 011 111<1y use a pencil for an y diagrams or gr;iphs.

1\nswcr nil questions.


If worl..ini: is needed for any question it must be shown with the answer.
Om1s~ion of essential work mg will result in loss of marks.
You arc expected to use a scientific calculato r to evaluate explicit numerical cxpn:ss1ons.
If the d egree of accuracy is not specified in the quest ism, and if th e answer is not exact, give
the answer to three sig.nificnnt figures. <>ivc ans wers in degree~ to one decimal place.
For n, use either your ca lculator value or 3. 142. unless the question requires the answer in
term:; of l!

The munher of marks is g iven in brackets [ J at the end.of each question or part question.
Tht: total of the marks for this paper is 50.

Fo r F:x:inuner·s Use

Total

T J1h qu rstion paper con ~isls of I 0 printed pages.


. . I I . h I f '>.954x 2.703
J. (a) Without usmg a ca cu ator, cs11matc I e va ue o correct 10 one
0.152
signi ficant figure.

Answer: (a) _ _ _ _ __ _ (2]

(b) .
F..~t 1ma1c tIic value o 1· J9.g
-;=- .
'.J9

Answer: (b) [2]

2. Arrange !he· following numbers in as~rnding order:

I
o.S2§, - o.§ . 3 ' 0.333

.lf11siver: 111

2
3. (a) Express 56 as a pro<)ucl if its prime factors.

Answer: (a)_ _ _ _ _____ • [I)

(b) Fi11<l the highest common factor, 1/CF and the lowest common 111ulti1>le,
LCM or 6, 14 and 56.

Answer: (b) HCF ~ ··- - _ ___I II

LCM~ -·--·· _ _ I11

4. Sirnplify
(a) 5ab - 3hc • 2ba ~ 1rb,

Answer (a) ____ _ _ _ _ _ II I


7 21
(b)
g// + gJ},• '

'

,111swer (b) _ __ -- [2]

3
c-- --
. -b+..Jb' - 4oc
5. Given a = 5 , b =- 6 an<l c = 1 , find the value of - -
·
2a

Ans\ver: -- -----·- _ [1]


---- -·-·-- - - - --·-----·----
6. Factorise the foJlowing expression~.

(a) - 2cd' +&de,

Al!SW<'r: (a) _ _ __ - (J)

(h) a - b - ak + bk .

Answer: (b) _ - (2]

.. ···------···-- - -- -
7. (a) Find t11c possible values ofy for which 2y:;; 18 ify is a square number.

Answer: (a) _ _____ _ _ Ill

(b) Find the possible value$ of x fc>r which : x > - :2 irx is.a negative integer .
.)

- [I J

4
. . 3x 2+x
8. (a) SoI ve t he equahon - ·• --- = I.
4 2

Answer: (a) x - ·- -- -- - ....,....___ (3)

5x ·t v 6 . fx
(h) If----·- = - . find the value o .
3.x-2y 7 . y

x
..fns wer: (b) - ·
v - -- --- ·-·----- 131
9. The diagram shows a regular hexagon, a square and an incomplele regular polygon of
11 sides.

(a) Find the value of one interior angle of the regular hexagon.

An~wcr: (a) _
0
111

(b) Find the value ofn .

-· 131
Jo. (a) Express 1.25 : 2-·in its simplest form.
16

Answer: (a) __ ____ _ _ , __ _ __ _ [! )

(b) The ratio of the number of b!iokmarks Tom has to what Dick has is 3 : 4
The ratio of the number of bookmark.<; Dick has to what Harry has is S · 6.
'- l fTom and Dick together have 595 bookmarks, how many bookmarks does
Harry have?

Answer: tb) __ _ _hook marks [2]

7
11. The <listance between Boon Lay Miff station and Simci MRT station is 35 km. A bus

takes 210 h to travel from. Boon Lay MRT station to Simei MRT station.
(a) Calculate the speed of the bus in
(i) km/h

Answer: (a) (i) _ _ _ _ _ km/h [ t)

(ii) mis

Answer: (a)(ii) _ _ _ _mis 11)

(b) lfa car travels at an average speed of80 km/h, how long will 11take10 travel
hctw<..>t:n the 2 stations'/ (Give your answer m mmutcs.)

Answer: (b) minutes (I J

12. Gi vcn the equation of a line is - 8x 1 4y = t 5. find

(a) the gradient of the line,

Answer: (a) __ [2)

(h) the v-inicn:cpt.


..fn,wer: (b) _ [II

8
13. A sales promoter of an elretronics shop is paid a commission of 5% for every
computer set that be sells. The seUing price of a computer set is $2 420.

(a) In a particular month, the sales promoter sold 15 computer sets

(i) Calculate the total connnission received by the sales promoter.

Answer: (ai) $ _ _ _ _ _(2]

(ii) In that month, after paying the sales promoter his commission, the
shop owner gets to keep 8% of the remaining amount as his earning.~.
Find the total amount the shop owner earned from this sale.

A11.nvcr(a1i) $
--- -_ f2)

(b) The selling price of a computer set is inclusive of a 10% GST . hnd the price
o f a computer set w11hou1 the inclusion of Gs· 1.

_ _ 121
14. On the clrngram shown ht:low, ti , 8 and Care points on the grid .

... . ' -- -·- - -·-


. . .. -·· -
I
• •
. . '
_ _ _ , __ _ J _ _ _ _. _ •. . . . ... . . . l . - ~---1 •••

·, ·. - . . . . '
-·.- ·-·t••• ; -· · ; · · · ;·· -
. . . .
., . . . ••· - - ·.- • • • r ••• f' • · • • • .

-.. -. -- . -. -·-. -... .. . . ..


,;
' ' . --.---
• '- \..

·~----'-~~-'---''- · J
- .'ff • -t
· · · 1 ··· ~ ·- · - · ---. - - - ·. - ·
··.~ -.~ :
:·· · -~·-·
u
r: :2
- - - ~ · - ·: · ·· :
:
_;___;, __,.___,_--''---Ht
:~ : :t, :
···:· ··- ~ --- ~ · - ·:·:.f:··-
~$

. · · ·· ·· ··· · : ... ~.:l, .. : ~ ....:.. .. ~ . ..: ... : . .. : . :.


.. ' .
'
. . . . t ., ' · ' · ·- · · ··· ._ . -.... .J----·----·---·"" ··· ' · . \ J • ••

. , . --·.. ---.--. -.- . --.- -.


I - . • ' :5 -. - ;.. -. --..--- -.-
. ---.,· -....· - .: ..

:' ... .... .... :..s:... -~ ... .:

(a) Find the coordinates of II and JI.

Answer: (a) A= ( ). LI = ( ), [2 J

(b) On the grid, mark the point C (0, 5).

Answer: (b) shown in diagram [ 11


(c) 1-11111 the area of!!.BOC.

Answer: (c) _ _ _ _ __,sq. units [2)

(d) Find th<.: gradient of AH.

Answer: (d) _ _ [2)

(e) What 1s the v~lue ofp if the line AH passes through( p ,- 1)·'

Ans wer: (e) p - _ _ __ ___ Ill

F ncl o f Papc1

10
Yuhuo Secondary School
Encl-of-Year Examination 2011
Scrnnrfory One Express

Mathcmatin
P3per 1(1016/1)

Marking Scheme

r··N~=fu-· _--=..:....:
Working Marks allocated
- - -!

1t~.0x27
Al
1 1.
' 0.15
J 27
= --
0.15
"'180

Al

3
2
r
Bl

{ :: 0.3,- 0..323,0.333. '.

(a) 56=2'x7
.)

Bl

f-- l j.
(b) HCF =._ 2; LCM= ..168
(a) 7ah+ 4/x:
Al . Al
Bl

(b)
7 g'1i·• Ml
- x- - ·
gli' 2J
Al

9:
(gh)'
= -- -

5. I - - - - Bl
f-'.- - - · - - - - - - - -
I6. 3 4 - d)
(a) 2cd( Bl

(b} I Ml
11(1 - k) - b(I · k) ! Al
/_ = (a - b)(J - k) _ __ I _
- Marks allo<:ated
No Working .
I. (il}
)' < <)
y = 1.4,<) Bl

(b}
r ., 3
Bl
:. x - 2.- 1
-
8. (..i}
3' - ?(2 t .r)
I
4
tt 4 2x . I
4
-- Ml
x 4
=I Ml
4
' - t1 ~ 4

x =8 Al

(b)
7 (51 • J') = 6('1.1 - 2y) Ml
1)r~ 7y 18r - 12y Ml
17\ 19y
.I 19
v 17 Al
9. (a)

illl I
(6 - 2)~0
6
I
- 1200 I
I
s-1
I
(b)
int L - 160- 90- 120
Bl
150°
I . (II
2)180 = I SO
,, Ml
18011 - 3~0 15011
10n =- 160
II 12 Al
rl~o. w~....
--- ~--

Marks allocated
I IMO' 3 Bl

(b)

I. Dick'~ bookmark =
~95 x 4
7 Ml

I 340

l I Harry's bookmark =
340 6
5 x

ll I. (a)
- - __:: 408
7 .
Al

15. --
(i) Speed = · · 10
= 50kmlli Bl

50km
Iii
(ii) = 50x 1000
l x 3600

I' s
13 9 m ls
Bl

(b)
I ).5 x60
I Time taken= SO
1 Bl
I ' 26 min.1
_ 4
112.- (a)
IS
y= 2x+ ·-
4 Ml
gradient = 2 Al

I __
(b) y-intercept = 1~ Bl
- - - - ·4 - - . --- - . --- -

J
-
No. Working Marks
allocated
- - - -- -----
13. (a)
(i) Total commission
Ml
-~ x (2420x 15)
- IOO
= $1815 Al

(ii) Total amounl


8
-
x(36300 1815) Ml
= I00
Al
= $2758.80

(b)
Price of compul er
2420 Ml
--·-- x lOO
= 110
= $2200 Al
14. (a) A(8, 0.5), 8(- 4,- 4.5) Bl. Bl

(c) Area of /\BOC


I
= - x5x4
2
= IOsq1mils

(d)
gradient of AB
- 0.5+4.5
- --
8- 4

=-
3

(e) p =6 J
End of Paper l
Index
C lass Number

Candidalc Name: 1-_


'-· _ ___.__ - -- :J

YUHUASECONDARYSCHOOL
END-OF-YE AR EXAMINATIO N 2011
Secondary One Express

MATHEMATICS
PAPER 2
• 4016/02

Tll\IE I hour IS m m utes

Candidalt:s answer on lhe Question Paper.

READ T HESE INSTRUCfJONS FIRST


Write you r name, class and index mrrnber 0 11 the work you haml in.
W rite in dark blue or black pen .
You may use a pencil 1<1r any dia~Jms or graphs.

Answer all questions.


If wo rk ing is needed for an y qucslion it must be shown w ith the answer.
Omission of e.-;sential working will result in loss of marks .
You arc cx pecled to use a scientific calculato r to evaluate ex plicit numerical expressions.
If the degree of accuracy is no t speci fi ed in the question, ancl if the answer is no t exuct, give
the ans wer lo three significa11t figurt!$. Give answers in d egrees to one decimal place.
For 11, use either your calculator value o r J. 142, unless the question requires the answer in
tcnns of II'.

T he number o f marks is given in brackets ( I al the e nd of each 4uestion or part question .


The total of the marks for this paper 1s 50.

r For F.~ anun er 's ll sc

Tot~I -~~ 50

This qut>stion paper ron si~ts of J J prin ted pages.

ll' UHN oyn i


I. The selling price of a Toyot;1 Vws is $52 250 and there ar(: two different payment
schemes available to Peter who wants to bny a car.

Payment Scheme A:
1.5 % discount if payrncnt by cash.

Payment Scheme B:
20% down payment, balance at simple interest of2.5 % per annum payahle in 3 years

(a) For Payment Scheme R, calculate

(i) the down payment payable.

Answer: (ai) $ _ ___,_ _ __ - [l)

(ii) Peter's monthly instalment.

Answer: (aii)$ _ _ __ 121

(b) Find his savings as a percentage of the selling price for Pa~rncnt Scheme 8 if
he cl1ooses Payment Scheme A.

tins 11·er:(b) · - -- - --· - "'/<> [.~)

2
2. ·me tickets for a conct-rt were priced at $3, $5 and $8. The number of S8 tickets
wld wasx. The number ofS5 tickets sold was twice the number of$8
1ickets. The number of$3 tickets sold was 30 more than the number of$5 tickets.

(a) Find an expression in terms of x for the total amount of nmney received
from sale of the tickets.

An.~wer: (a)$_ _ [I]

(b) Given that the total sale of tickets was $8010, fonn an equation in-'·
Solve this equation and hence lind the total number of tickets sold.

A11$wer: (h) _ _ _ ti ckets 12]

3. (a) Two numbers we-re given as m ' x 11 and m x 11' . Find the cube root of their
produt:I in ter111s of m and n.

Annver: (a) _ _ _ _ _ 121

(h) 11irce light hou~e.5 Oash their lights every 20 seconds. 30 seconds and 45
seconds respectivel y. Given that they flash together at 7p111, when will they
next flash together?

tfnrnw: (h) _ _ _ (2)


4. (a) F,valuate the following, rounding off your answer to z·significant figures.

5.15i"lli
., I.Ix 42.2

Answer (a) _ __ _ [I J

(b) Mr Lo wishes· to spend at mos t $ 55 000 in an advertising campaign. $38 500


is to be spent on T V conuncrcials and the remainder on T-shirts which will be.
given away to customers. The T-shirts will cnst the company $2.80 each.

(1) Form an 1ncquali1y inx.

tlnswl'r: (hi) _ _ _ [J]

(1i) Find the greatest number of T-shirts that can he given away .

Answer: (b11) _ _ - - · _ Ill

·I
5. David walks his brother to school everyday al a speed of (x -J I)mis. If the distance
between his house and the school is 6000m,

(a) Find, in tenns ofx, the time taken in seconds for Oavi1J to reach school.

Answer: (a) _______ _ _ __ seconds [I}

(h) Jf1he time taken for David to reach the school is 40 minutes, form an
equation in ierms of x and show that it reduces 10 --- ~ "' 2.
·x+ I

Answer.· (b) [I}

(c) Solve the equation _2__ = 2 .


x+I .

Answer: (c) x = ______ [2}

(d) If David takes Sminutes to drop his brother off then spenl 30 minutes walking
home, find the average speed for his journey from home to school and back .

_ _ mis (?.)

5
6. (a) Constnict a parallelogram PQRS in which PQ = 8 cm, /,.PQR = J 00° and
QR ~ 5cm.

Answer: (a) 12J

(b) Measure L QRS .

Answer (b) / QRS = __ _o [IJ

(c) Constrnct the perpendicular bisector of PQ. [ 11


(d) Constrnct the angle bisector of L SJ'Q . J1I

- . ·------····- - -

(,
7. In the diagram below, AB is parallel to FF..A F is parallel to EG, / ABC= 135°,
L BCD - 115°, LCDE = 85°. LFAB 95°

)is•
--- - -----
D
85''

n
z"
- G

F111d the values or

(u) x,

A11s11er (a) x~ _ _ [I]

(b) y.

Answer: (b) y --- [2J

(c) z.

A l/SWl'r: ( c) :: - II I
7
l!. A square construction paper was cut as shown below so that a symmetrical star was
obtained. Find the area of the star that was cut out.
12cm

1 ?' CO\

5 t;Ul

2 cm 8 cm

CIJl 15)
9. A svlid cylinder is made from plas1ic. The height and diamclcr of the cylimJcr
measured arc 16cm and IS cm respectively. (Use n 3.142)

------ 1'5-c1;1 ~,
. /
·------

Fig I
I
16 cm
~ ,(/

,/ -~ .
I
t
/ '
·- - - /

1 Fig 2
8cm
i
.
(a)
15 cm-..
-1scm
-
Calculate lhe volume of plastic needed to make the cylinder (Fig 1 as shown
above) correct to 3 significanl figures.

Answer: (a) _ _ cm' l21

(b) Ten such cylinders arc melted to make cubes whose sides are 13 mm
eacl1. What is the maximum number of cu hes I hat can be mndc from lhc 10
cylinders?

A11.rn·l'r :(b) _ _ _ i.:ubcs [2)

(c) A quarter of the cylinder is cut off from the cylinder. Calculate the toial
surface area of the cylmder Jell after the cut. (Fig 2 as shown ahove)

. 111.rn-er :(<•) _ _ _ _ cm ' 131


I0. Answer the whole oftlris question 011 the ~·heet ofgraph p11per providl'd 0 11 page I J.

Given the equation y . 3.t + 8.

12::__1-~~=r.L. IQTI_ I
~-·=u=_ii~

(a) Find the value of"·

,,1nsl>ver :(a) a= [I]

(b) Using a scale of2 cm lo represent I unit on the x-ax is and I cm to represent
I unit on the y-axis, draw rhe graph o fy = 3x + 8 for the values of x in !he range
· 3 S x:52. [3J

(c) U~c your graph 10 find !he value of y when x - I .:'\,

Answer :(c) y = _ _ - Ill

(d) On the same graph. draw the hnr y ,.. 5. (l]

(e) Find the coordinate~ of the point of inlersection of !he lines y - 5 and y - 3x 1 8 .

Answer :(c) ( _ _ _ _ [I)

Enrl of Paper 2

10
Yuhua Secondary School
ExJmination 2011
End-of -Yoar
S~condary One fxprcs<

1'.\iithematio.
Paper 2(4016 I 2)

Marking Scheme

Marks allocated
I lNo_, l(a) 20·- -- -

1(.) .100- x 52250


) I

= $10450 Bl

(ii) Balance amount


= 52250 -- 10450
"' $41800
Ml

I = .:±_1800x 2.5~ . $J l 35
100

I rnonlhly imt<illrnent
41800+ 3135
36
= $1248.19 Al

(b)
Amount paid under Scheme A $51466.25
Amount paid under Scheme B =- $55385 Ml

1
%savings
! = 55385 _:- 5 I 466.25 x JOO= ?.OS%
Ml. Al
...4-_ 55385 - - - - -- - - - - - - -
__
2. (a) 24x + 90 Bl
(b)
i 24.x+90 "' 8010 IMl
24x= 7920
x 330 !
Total number of lickets = 330 ~ 2(330) + (30 + 330) · 1680 I Al
_ _.L._
·-- -- - - --- .
•' No. Workin~ M arks allocated
3. (a)
ll j • Mt
'I Ill X n
1
Al

I
n111

(b) LCM = 180 seconds = 3 minutes


Time = 7.0 3 pm
II Al
Ml
1-..- - l . - -
4. ( a) 0 .0018 Bl

(b)
{i) 2.Sx + 38500 s; 55000

(ii) .t s 5892.85
1-- =1-..:·:.::
:. c X
:_"';..5
::.:8:.::9..:
2_ __ __ _ _ __ _ .______ Al
5.
(a ) 1 1me ta ke n = -6000
..
- Bl
.r + J

6000 =40 x 60
x+ I
6000
(b) - 2400
r+I
5 Al
2(show11)
~ 1 I

(c)
5 = 2(x+ I) Ml
5 h+2
2x = 3
Al
x 1.5

(d)
ToJal distance !raveled = 6000 x 2 = 12000 m
Tota l time take n = 40 ~ 5 + 30 = ·75 mi n> = 4500 secs

12000

l
Ml
4500
Average speed = J
= 2 ::.. 111is Al
}
~- --
'NQ.- ~-- ___
W o rking ____.._ Marks a llocated
1·· --1 ·•).(<).(d)
B2. Bl. Bl

. l

(b) LQRS 18°


Bl
!-=-'---- - -- ·· -
(a ) .x ~ 180 - 9s c·mt L s. 11 1.mes>
7. Bl
= 85

(b} sum of int. a ngles in hexagon = (6 2)180 "' 720 Ml


:.y=7200- 5 15
Al
205

2 = 160 - 205 8'\


{cl {ult /: s. II lines. mm of L s. in /\) Bl
70
8.
-
7
-- - ·
. Area o f square = 12 x 12 = 144 cm~
-- ~--
Ml
Area of top 2 larger trapeziums = 2" t x (S ~ 6)x S = 55 cm' Ml
2
· Area of3 tria ngles = x 2 x 3x 2 ~
1 1
x Rx 2 = M cm' Ml
2 2
1
Area of smaller trapeLiums = (2 ~3)x5x2= 25cm2 Ml
2
Area o f star . 144 - 55- 14 - 25 = 50 cm '
Al
1
9. (a} Vo lume o f cyl inder =3. 142x7 5 x l 6 = 2Bi7.8 cm' Al

±
28278
(b) M ax number o f cube~ , - - 1'.'87 I Ml. Al
l. 3'
, I
(<) Toial surface area
1
= (3 1,12>'7.5 1 H (15 x 8)+(3. 142x7.5) I · (7x'.1. 142 x7.5xl6) M2 •
I 4 Al
I _ 1Q_'10 cm' ___ __ _ __ _
r- - - --
l __No.
10. (a) a = -1
_ _ ----- -- ~{()rking _ - -- - ·-···Marks
81
--
allocated
~

(b) See graph below

(c) y = 12.4

j (d) See graph below Bl

IfeH-1 . 5 ) Bl
---~·--· - -·-· ·~------ ---

I~
)
) - .,: /

/
I
I
/

I
Calculator Model:
Name: Index No: Class:

Mark Awarded :
~ Parent' s Signature:

:. ' .

NORTHLAND SECONDARY SCHOOL


Motivated Learners, Assets to Community
Nurturing Minds, Shaping Character, Strengthening Vigour

FINAL EXAMINATION 2011


Subject : MATHEMATICS Paper: I
Level : Secondary I Express Date: I0 October 2011
Duration : I hour 15 minutes Time : 0740 - 0855
Setter: Miss Joanne Yap Vetter: Mr Low K. M.
/
READ THESE INSTRUCTIONS FIRST
Write your name, index number, class and calculator model in the spaces at the lop of the
page.
Write in dark blue or black pen.
You may use a pencil for any diagrams or graphs.
Do not use staples, paper clips, highlighters, glue or correction fluid.
Answer all questions.
If working is needed for any question it must be shown neatly and clearly in the space below
the question.
Omission of essential working will result in loss of marks.
Calculators should be used where appropriate.
If the degree of accuracy is not specified in the question, and if the answer is not exact, give
the answer to three significant figures. Give answers in degrees to one decimal place.
For 1! , use either your calculator value or 3.142, unless th<? question requires the answer in
terms of 1! .
At the end of the examination, fasten all your work securely together.
The number of marks is given in brackets [ J at the end of each question or part question.
The total of the marks for this paper is 50.
For Examiner's Use

This paper consists of 8 printed pages including the cover page


I. 4
lt is given that 240 = 2 x 3x 5.

(a) Express 168 as a product of prime factors.


(b) Hence, by using your answer in part (a) and the given information, find
(i) the highest common factor of 168 and 240.
(ii) the lowest common multiple of 168 and 240.
• I '

Ans: (a) [1)


(b)(i)_ _ _ _ _ _ [l]
{
.
(ii) [I)

2. Using a calculator, evaluate the following and correct your answer to 3 significant
figures.
2
(a) {(5 - 12_) 2 +0.2 2
v' 3 12

I
4.8+
(b) 6
2.71 - 1
2

Ans: (a) _ __ _ __ _ [l]

(b) [l)

2
3 q r
(a)If p= - - ,findpwhenq = 101,r = 5andt = l.
rt +3

(b) Danny is x years old, his sister Dahlia is thrice as old as Danny and his brother
David is l 0 years older than Dahlia. Write down in tenns ofx
l .

(i) Dahlia's age

(ii) David's age

(iii) Given that their mother's age is twice the sum of Danny's and Dahlia's age.
Write down their mother's age in terms of x .

(iv) Given that their mother's age is 48 years old, find Danny's age_

Ans: (a) [IJ


(b)(i) earsold[l]
(ii) ears old [1]
(iii) years old [1}
(iv) years old [I]

4 (a) Simplify
(i) 7m + 2n + lm -4(111 + n)]
.. ) 3x+2 2(1-x)
( II - - - - - -
5 3

(b) Factorise
Sax - 6by + I Obx - 3ay

Ans: (a)(i) [ 1l,

(ii) [2]
(b) [2]
3\}?

3
5 Solve the equations
(a) 9x - 4 = 5(x+ I2) ,
2x - 5 4x+l
(b) - = - -
8 2

..

Ans:(a)x= _ _ __ __ [2)
(b)x = [2]

6 A boy is 24 years younger than his father. In 3 years' time, the sum of their ages
would be 42 years old.
(a) Write an equation to represent the sum of their ages in 3 years' time.
/
(b) Find the father's current age.

Ans: (a)._ _ __ __ _ [1]


(b) [ l]

7 (a) Solve the inequality - 8::::; 4(3 - x) + 2x.


(b) Hence, write down the largest possible value of x ifxis a prime number.

Ans: (a) _ ___ _ _ _ __ [ l J


(b) [I]

4
8 A plumber works 40 hours in order to earn $150. Assuming the same rate of pay,
calculate
(a) how much he would earn in 16 hours.
(b) the number of hours he would have to work to earn $67.50.

Ans: (a)$ _ _ _ _ _ _ _ [!]


(b) hours[ l]
/

9 (a) A salesman sold 120 digital cameras in June. In July, the sale of
digital cameras increased by 15%. Find the number of digital cameras sold in
July.

(b) James and his friends paid $135.90 for a meal. This amount included the 10%
service charge. Find the amount James and his friends paid for service charge.

Ans: (a) _ _ _ __ _ _ _cameras [2]


(b) $ _ _ _ _ _ _ _ (21

5
IO. (a) Find L. z in the following figure .

.. 135~~
z

'.

(b) Find the gradient of the line which passes through points (2 , -5) and (5,7).

Ans: (a)_ _ __ _ _0 [3)

(b) [2]

11 Jn the diagram, ARC and AED are straight lines. EB is parallel to DC and BD =DC.
Angle AEB = 100° and angle BDC = 70°. Find the values of

(a) x.
(b) y.
(c) z.

Ans: (a) - - - -- - -- 0 [l]


(b) 0 [1]
- - - - -- --
(c) O[J )

6
12 (a) Four angles in a hexagon are 110° each. The remaining two angles are in the
ratio 2: 3. Find the smaller of these two angles.

(b) A regular polygon has n sides. The size of each interior angle is 11 times the
size of each exterior angle. Find n.

;. ) ,-

Ans: (a) _ _ _ _ _ _ 0 [2]


(b) sides [2]

!3 Figure ABCD is a parallelogram. The perpendicular from A meets CD produced


/
at£. AE = 8.8m, AB = Sm, BC = I Im and BF = 4m. Calculate

(a) the area of paralleiogramABCD,


2
(b) the length of DE given that the area of trapezium ABCDE is 70.4m ,
(c) the area oftriangleAED,

5
A B
1"-~ ./~,
-~

',,
/ 4 "-.
. .< "'-,

?;<< ~'-11
8.~ 1
"'-, ,
'"' D
tl................... -···--······-······ ···· ...................> ~. . c
E

Ans: (a) ------~ m [l]


2

~) m[~

(tj m[2]

7
14 The pie chart shows the proportion of students of different mother tongues in an
International school. There are 45 students whose mother tongue is Italian. Find

(a) the total number of students in the school,


(b) the value of x,
(c) the number of students who speak French.
>.

Ans: (a) _ _ _ _ _ _ _ _students [1)


(b) _ _ _ _ _ _ _ 0 [2]

(c) _ _ _ _ __ _ _ students [I]

Have you checked your work?©


End of Paper

8
\ o.) ~~x-~xl::-(bg
6) \\) !-\CF :: .llf--
~\\ LCM ~:;_tt(~'f~~l
== f b~O
10 o.) Lz =~2°
!1 t\) 11· cq
6) 0 -lS"°' . b) 4
s o,) ~ ={2. 11 q) Lx = 3~~
b)~> s.)C b) L~ :: 55~ 0
\)\) ~~ t { 0 c.) L ~- .:: ?-~
~II) 2-)c..
QV) .)l ::c b· 12 (A) 1121)
.\ O\~I) ;)_{ !) M_ -I'\) b) (\:: 14-
(\'1 ) \q )( ~4
s- I~ a_) . 4- ~ 'M )_ .

6) ~2( -sl{) ( Q +d-b) lo) 6 M


Sl-~ · 4
> ~ y__ .=: (b
C) Ml.

bJ :::( ;:, - j
llf-

~ ~) If- 2...

b) ~o
1 o.) x ~lD
6) )(_ . : : 1
~ C\) ~bO
b) I~~

1 q) !~ 2 CQAetf1J
. b) ~l2· b~
Calculator Model:
Name: Index No: Class:

Mark Awarded: SO Parent's Signature:

. ;

NORTHLAND SECONDARY SCHOOL


Motivated Learners, Assets to Community
Nurturing Minds, Shaping Character, Strengthening Vigour

.. ~ - - ··

~-
FINAL EXAMINATION 2011
Subject: MATHEMATICS Paper: 2
Level : Secondary 1 Express Date: I 0 October 2011
-- -··--
Duration : 1 hour 15 minutes Time : 0900 - I 015
--
Setter : Miss Joanne Yap Vetter: Mr Low K. M.
---·
/
READ THESE INSTRUCTIONS FIRST

Write your index number and name on all the work you hand in.
Write in dark blue or black pen on both sides of the paper.
You may use a pencil for any diagrams or graphs.
Do not use staples, paper clips, highlighters, glue or correction fluid.

Answer all questions.


ff working is needed for any question it must be shown with the answer.
Omission of essential working will result in loss of marks.
Calculators should be used where appropriate.
If the degree of accuracy is not specified in the question, and if the answer is not exact, give
lhe answer to three significant figures. Give answers in degrees to one decimal place.
For Tl, use either your calculator value or 3.142, unless the question requires the answer in
terms of Tl.

At the end of the examination, fasten all your work securely together.
The number of marks is given in brackets [ ] at the end of each question or part question.
The lotal of the marks for this paper is 50.

- -···--·- -- -- - - - - -- -- - - -- -- - - - - - - - - - - -- - - -

This paper consists of 6 printed pages including the cover page


l. Derek used some sticks to form a series of shapes as shown below.

!51 shape 2nd shape 3r<t shape

(a) Write down the number of slicks required to form the.61h shape. [I]

(b) Given that to form the nth shape (an+ b) sticks are required, where a
and b are constants. Write down the values of a and b. [2]

(c) Hence, find out which shape is made up of 164 sticks. [2]

2. ln the rhombus ABCD, DB cuts AC atX andLDAC = 50°. !lAXP is isosceles


such that AX= PX. The line PX produced meets BC at Q. Find, stating reasons
clearly,

(a) L BXA [l]

(b) L AXP [1]

(c) L CXQ . [1]

(d) /BQP . [I]

2
3. The diagram below is made up of a square, a regular pentagon and an incomplete
regular polygon ABCDE of n sides. Find

. ;

E
(a) the value of each interior angle of the pentagon, [I)

(b) the value of obtuse LBCD, [1]


I/
(c) the value of each exterior angle of polygon ABCDE, [l]

(<l) the value of n. [1)

4. A triangular piece of paper is removed from the original piece to fom1 a new piece
as shown below (diagram is not drawn to scale). Find ·

15 cm

6cm

(a) the perimeter of the original piece of paper, [2]

(h) the area of the new piece of paper, [2}

(c) the cost of the new piece of paper if it costs $0.25 for every one square [I]
centimetre. Convert your answer to the nearest cents.

3
5. The chocolate bar is made up of an outer chocolate biscuit enclosing chocolate
cream in its hollow.

4cm ..
'

- -11- --6-cm

If the length of the biscuit is 6 cm, calculate

(a) the amount of chocolate cream needed to fill up the biscuit, [2]

(b) the volume of the outer biscuit. [2]

(c) the curved surface area of the biscuit that is in contact with the chocolate [2]
cream.

6. Figure l and 2 show two types of prisms. All dimensions are given in cm.
Calculate
3

1
8

1
7

Figure 1 Figure 2

(a) the volume of the prism (shown by figure I), [2]

(b) the total surface area of the prism (shown by figure 1), [2]

(c) the whole prism is then melted completely to form solid cuboids as shown [2]
in figure 2. Given that the dimensions of a cuboid are 2cm by '1cm by x cm,
find the value of x.

4
7. The histogram below illustrates the results of a survey to find the number of
passengers in cars crossing a bridge.

6 ~

5 -
Number '.
of
cars 4 -

2 -
1 -
0
1 2 3 4 5
Number of passengers

(a) Calculate the total number of cars. [I]

(b) Calculate the total number of passengers. (2)

(c) Find the fraction of cars with Jess than 3 passengers. (1]

(d) Find the percentage of cars with more than 3 passengers. [ 1)

(e) If the information is to be illustrated in a pie chart, calculate the angle of the [I]
sector representing cars with 5 passengers.

8. (a) Construct triangle ABC such that AB = 8cm, BC = 6.5cm and AC= 9cm. [2]

(b) Measure and write down LABC. (I]

(c) On the same diagram, construct with compasses, the


(i) angle bisector of angle A CB [l]
(ii) perpendicular bisector of line AC [I)

(d) Label the intersection of th_e two bisectors as T and measure the length of BT. [ I)

5
9. Answer the whole of this question on a graph paper.

The table below shows the values for the equation Y =4 - 3x.

- 1 0 2
a 4 b
I '

(a) Find the values of a and b. (I]

(b) Using a scale of2 cm to I unit on the x-axis and JCm to represent 1 unit on [3]
the y-axis, plot the graph of y = 4 - 3x for - 1 :S x ~ 2 .

(c) From your graph, find

(i) the value ofy when x = 0.5, [ 1J

(ii) the value of x wheny = 4. [I]

( d) Draw the graph, y = x. Find the coordinates of the point of intersection of (2]
the two lines.

Have you checked your work? ©


End of Paper

6
ANSWER SCHEME
Final examination 2011
Sec One Express Mathematics Paper 2

I. (a) 60 sticks (Bl]


(b) a = 8,
b= 12
:. .
,· (Bl)
(BI)
(no working is required)
(c) 8n+12=164 (MIJ
Sn = 152
n = 19 (AIJ ·

2. (a) LEXA = 90° (Diagonals in a rhombus intersects at 90°) [Bl)

(b) L AXP =180° - 50° - 50° = 80°(L sum of triangle) (BJ)

(c) LCXQ = LAXP = 80° (vert opp. Ls) [BI]

(d) LPQC = LAPQ = 50°(altLs) [BI)


LBQP =180° - 50° = I 30°(Ls on a str. line)

3. (a) . (5 - 2)xl 80° JBI]


Intenor angle of pentagon = = 108°
. . 5
(b) LBCD = 360° - I 08° - 90° = 162°( L s at the point) (BlJ
(c) Exterior angle of polygon ABCDE = 180° -162° =18° IBIJ
(d) 800
J8n = 360° OR (n - 2)xl = 162° (Bl!
n
n = 20 I 80°n - 360° =162n°
l8°n = 360°
ti = 20

4. (a) 6 (Ml]
Perimeter of original piece = Jr( ) + 15+15 + 6
2
= 50.08510965cm
=50. lcm (3 s.f.) (Alf
(b) (Ml)
Areaofnew piece = (15)(6)+7r(3) 2 _ .!_(6)(3)
2
= 109.2743339cm 2
[Al]
= I 09cm 2 (3 s.f)
(c) Cost = $(109.2743339x0.25) = $27.31858347 [Ml)
= $27.32 (Al] _

5. (a) Amount of chocolate cream needed = n(l 2 )(6) [MJ}


= I 8.84955592cm 3
= I 8.8cm 3 (3 s.f.) (Al]
(b) 2
Volume of the outer biscuit~ 7r(2) {6) (Ml}
= 75.39822369cm 3
=75.4cm 3 (3 s.f.)
(Al)
(c) Curved surface area = 2.7r(l)(6) (MI)
=37.699 l l 184cm 2
(Al]
= 37.7cm 2 (3 s.f.)

Volume of prism = ((3x8) + _!_(3 + 6)(4)),~6


6. (a) [MJ)
. 2
= 252cm 3 IAII
(b) Total surface area o(prism =
{42x2) + (8x6) + (3x6) + (2x6) + (5x6) + (6x3) + (7x6) (Ml)
= 252cm 2 (At)
(c) 252 (Ml)
x= -
4x2
= 31.5cm (Al]

7. (a) Total number of cars = 3 + 5 + 6 + 4 + 2 = 20 (Bii


(b) Total number of passengers =
(3)(1) + (5)(2) + (6)(3) + (4)(4) + (2)(5) (Ml)
= 57 (All
(c) (Bl)
Fraction of cars with less than 3 passengers = _!_ =3.
20 5
(d) Percentage of cars with more than 3 passengers = ·
6 JBll
- xl00% = 30%
20
(e) {Bii
Angle of sector = 2-x360° = 36°
20

8. (a) For accurate measurements and drawings of AB, BC and AC. (Bl)
For labeling of all points and given values.
(Bl)
(b) LABC = 75° accept values 74.5°,75°,75.5° (Bl)
(c)(i) For compass markings and straight line through C for angle (Pl]
bisector.
(c )(ii) For compass markings and straight line centre of AC for IPIJ
perpendicular bisector.
(d) Length BT= 3.3cm accept values (3.3 ± O. l)cm (Bii
fBl} only if students label point T and measure length BT
accurately.

9. (a) a = 7, b - -2 (Bl) for two correct answers IBI]


(b) All points correctly plotted (Pl)

2
A correct scale is chosen (Bl)
Drawing a straight line through all the points (Bl)

(c)(i) y = 2.5 (Bl) if answer is exact (Bl]


(c)(ii) x = 0 (Bl) if answer is exact IBl)
(d) All points correctly plotted (Pl)
Drawing a straight line through all the points '.
Point of intersection = (I, I) (Bl)

Answer Scheme for Question 8

..·

I
I
"
1 •
I
~

'
~ .

,I
I
/ -..
I

:'

I
I
Name
I
Class: Sec 1
I Class Register No _ _ _

CHUNG CHENG
50
HIGH SCHOOL (MAIN)

2011 END-OF-YEAR EXAMINATION


SECONDARY 1
MATHEMATICS 4016/02
Paper 2 7 October 2011

Additional Materials: Writing paper(4 sheets) 1 hour 15 minutes


Graph paper (1 sheet)

READ THESE INSTRUCTIONS FIRST


Write your name, class and class register number on all the work you hand in.
Write in dark blue or black pen.
You may use a pencil for any diagrams or graphs.
Do not use staples, paper clips, highlighters, glue or correction fluid.

Answer all questions.


If working is needed for any question it must be shown with the answer.
Omission of essential working will result in loss of marks. ·
You are expected to use a scientific calculator to evaluate explicit numerical expressions.
If the degree of accuracy-is not specified in the question, and if the answer is not exact, give the
answer to three significant figures. Give answers in degrees to one decimal place.
For Jr, use either your calculator value or 3.142, unless the question requires the answer in
terms of 7r .

At the end of the examination, fasten all your work securely together.
The number of marks is given in brackets [ . ] at the end of each question or part question. The
total number of marks for this paper is 50.

DO NOT TURN THE PAGE OVER UNTIL YOU ARE TOLD TO DO SO

(Tu rn Over
CCHMS £0 Y £.ram 201 1 Sec I Maths Paper 1
2

l. The interior angle of a regular polygon is 165°. How many sides does it have ? [3]

2. Simplify 4x - [5(4y - x) - 9(y+2x)] [3]

3. x+y [3]
If k =- -, find x when k = 12 , y = 4.
3

4. Factorise 6mq - rp + 3mp- 2qr completely. [3]

5. In preparation for the Great Singapore Sale, a store assistant accidentally raised the [3]
price of digital camera by 15% instead of lowering it by 20%. As a result, a customer
who bought this camera was overcharged by $ 161. Find the original price of the
digital camera.

6 Solve the inequality 3x-4 :$ 16 [2]

(a) If xis a natural number, represent the solution on a number line. [2]

(b) State the greatest value of x if

(i) x is a prime number, ( I]

(ii) x is a rational number. [1 l

7. (a) A farmer has x tomato plants. He estimates that each plant will produce 5.5 kg [I]
of tomatoes. Write an expression, in terms of x, for the total mass of tomatoes
that he expects to be produced.

(b) He intends to apply 300ml of liquid fertiliser to each plant. The fertiliser is sold in [2]
containers each holding 6 litres and costing $135 each. Write down an expression,
in terms of x, for the number of containers of fertiliser he must buy.

(c) If the total cost of the fertiliser is $810, form an equation in x and solve it to find the [3]
number of tomato plants that the fanner has.
[llitre = lOOOml]

jTurn Over
CCHMS EOY Exam 2011 Sec I Maths Paper 2
3

8. The diagram shows a closed metal storage container made up of a cuboid joined to half
of a cylinder. Given that AB = 5.6 m, BC= 8 m and AE = 12 m.
22
[fake 1t to be - ]
7

B - E

8 -G
--'"' ...
--- ---
c H
D

1 (a) Calculate the volume of the container, giving your answer in m 3 • (3)

(b) The exterior surfaces of the container is to be painted. Find the total surface area [3]
to be painted in m 2 • ·

9. (a) Given that x : y = 2 : 3 and y : z = 4 : 5, find x: y: z. ll ]

(b) If z has $14 more than x, find the total sum of money that is shared between x, y [2]
and z?

I0 . Solve the equation


2· 6 [31
3+ - - = l - - -
x+ 6 x +6

11 . Linda had sweets of 3 different flavours. There were 84 mango flavoured, 98 grape
flavoured and 11 2 kiwi flavoured sweets. She packed an equal number of each flavo ur
into sma ll packets without any leftover.

(a) What was the maximum number of packets? [2]

(b) How many grape flavoured sweets were there in each packet? [I]

!Turn Ove r
CCHMS EOY Exam 201 1 Sec I Maths f'aper 2
4

12. Answer the whole of this question on a sheet of graph pa per.

Given the equation y = - 3x + 4,

x - 4 0 2
y 16 4 p

(a) find p. [1]

(b) Using a scale of2cm to represent l unit onx-axis and tern to represent I unit on [3]
the y-axis, draw the graph ofy = - 3x + 4 .

,....•·(c) By using your graph, find the values of

(i) y when x =2 [l]

(ii) x when y = 6.4 [1]

(d) From your graph, is ( 1 , 1 ) a solution to y= - 3x + 4 ? Give an explana_tion for your [2]
answer.

- End of Paper -

CCHMS EOY Exam 2011 Sec I Maths Paper 2


Marking Scheme
Sec I EOY Paper2 (2011)
I. The interior angle of a regular polygon is 165°: How many sides does it have ? [3]

(n-2)180 =
Exterior L = 180° - 165°
165 Mlcao
n
= 15° Mlcao (n - 2)180=165n

if "des= -360 180n - 165n =360


Al
JVO.O Sl Mlcao
. 15 15n =360
=24 Al 360
n= - Mlcao
15
=24
No.of sides = 24 Al

2. Simplify 4x -[5(4y -x) - 9(y + 2x)] [3]

4x - [5(4y -x) -9(y + 2x)] = 4x-(20y -5x - 9y - 18x) Mlcao


= 4x - (l ly - 23x)
= 4x-l ly+ 23x Ml
=27x-lly Al

3.
If k
x+y
= - - , find
x .when k =12, y = 4. [31
3
when K = 12, y = 4.
K =x +y
3
l2 =x +4 Mlcao
3
12x3 =x+ 4 Mlcao
x=36- 4
=32 Al

4. Factorise 6mq - rp + 3mp- 2qr completely. [3]

6mq-rp+3mp-2qr = 6mq+3mp - 2qr - rp


= 3m(2q + p)-r(2q + p) Mlcao, Mlcao
=(2p + q)(3tn - r) Al

(Turn Over
CCHMS EOY Exam 201 1 Sec I Maths Paper 2
3

5. In preparation for the Great Singapore Sale, a store assistant accidentally raised the [3)
price of digital camera by 15% instead of lowering it by 20%. As a result, a customer
who bought this camera was overcharged by $ 161. Find the original price of the
digital camera.

Let $x be the original price of the camera.

Price when raised by 15% = $1.15 x


Price when reduced by 20% = $0.8 x
Price difference= $1.15 x - $0.8 x Mlcao
= 0 .35 x
0.35x = 161 Mlcao
x = 460
The original price of the digital camera is $460 Al

6 Solve the inequality 3x - 4 S 16 [2]

(a) If xis a natural number, represent the solution on a number line. [2J

(b) State the greatest value of x if

(i) x is a prime number, [I)

(ii) x is a rational number. [I)

3x- 4 S 16
3x s l6+4 Mlcao
xS 20
3
2
xS6- Al
3
(a)

Missing or wrong points,x, --+


-0.5 marks each, max -2marks
Presentation

;.
o· 2 3 4 5 6 x

(b)(i)greatestvalue x =5 Al OR (i) 5

(ii)greatestvalue x =6~ Al (ii) 6-


2
3 3

7. (a) A farmer has x tomato plants. He estimates that each plant will produce 5.5 kg [I]
[Turn Over
CCHMS £0Y Exam 2011 Sec I Ma1hs Paper 2
4

of tomatoes. Write an expression, in terms of x, for the total mass of tomatoes


that he expects to be produced.

(b) He intends to apply 300ml of liquid fertiliser to each plant. The fertiliser is sold in (2]
containers each holding 6 litres and costing $135 each. Write down an expression,
in terms of x, for the number of containers of fertiliser he must buy.

(c) If the total cost of the fertiliser is $810, form an equation in x and solve it to find the [3]
number of tomato plants that the farmer has.
( llitre = I OOOml]

(a) Total mass= 5 .5x kg Bl

(b)
.
Amount of fertiliser needed= 300x ml Ml cao
, ..;.»

. 300x
No of contamer = - - Al
6000
x
20

(c)
x
- x 135 = 810 Ml
20
810x 20
x=--- Mlcao
135
= 120
:. no.of tomato plants farmer has = 120 Al

8. The diagram shows a closed metal storage container made up of a cuboid j oined to half
fTurn Over
CCHMS EOY Exam 201 1 Sec I Mai/is Paper 2
5

of a cylinder. Given that AB = 5.6 m, BC = 8 m and AE = 12 m.


. 22
[Take 1t to be - )
7

8 G
.. __.
H
D
1 (a) Calculate the volume of the container, giving your answer in m 3• [3]

(b) The exterior surfaces of the container is to be painted. Find the total surface area (3]
to be painted in m 2 •

(a)
1 5.6 2 8
Basearea = - 7!(-) + x5.6 Mlcao
2 2
= 12.32 + 44.8
2
= 57.12m
Vol.of container = 57. 12x 12 Ml
3
= 685.44 m Al

(b)
I 22
Total surface area = 2 x 57 .12 + [8 + 8 + 5.6 + - (- )(5.6)] x 12 Ml, M lcao
2 7
= 114.24+30.4x 12
= 114.24 + 364.8
= 479.04 m 3 Al

9. (a) G ive n that x : y = 2 : 3 and y: z = 4 : 5, find x: y: z. [1)

(b) If z has $ 14 more than x, find the total sum of mo ney that is shared between x, y [2]
and z ?
7 units - - - $ 14
(a) x : y = 2: 3
= 8 : 12 $(~x35)
: . 35 units 7
y:z = 4 : 5
= $70 .
= 12 : 15
. . x:y:z = 8: 12:15 Bl
14
(b) Total sim1 of money = 7 x (8 + 15 + 12) Mlcao

= $70 Al

(Turn Over
CCHMS EOY Exam 2011 Sec I Maths Paper l
6

I 0. Solve the equation


(3]
2 6
3+ - - = l - - -
x +6 x+6

2 6
3+ - - = 1- - -
x+6 x+6
2 6
- - + - - =l-3 Mlcao
x+6 x+6
2+6 =-2
x+6
8 =-2(x+6) Mlcao
8=-2x-12
2x =-12 - 8
x=-10 Al

11. Linda had sweets of 3 different flavours. There were 84 mango flavoured, 98 grape
flavoured and 112 kiwi flavoured sweets. She packed an equal number of each flavour
into small packets without any leftover.

(a) What was the maximum number of packets? [2 1

(b) How many grape flavoured sweets were there in each packet? [1)

(a) 84 = 22 x 3x 7
98 = 2 x 72 Mlcao

112 = 24 x 7
HCF of (84,98,112) = 2 x7
= 14
:. Max.noof packets= 14 Al

(b) No.of grape sweets in each packet = 98 + 14


=7 Al

fTurn Over
CCHMS £0 Y Exam 2011 Sec 1 Maths Paper 2
7

12. Answer the whole of this question on a sheet of graph paper.

Given the equation y = - 3x + 4,

x · - 4 0 2
y 16 4 p

(a) find p. [l)

(b) Using a scale of 2cm to r.epresent l unit on x-axis and 1cm to represent I unit on [3]
the y-axis, draw the graph ofy = -3x + 4.

( c) By using your graph, find the values of

(i) y when x =2 [1 J

(ii) xwheny = 6.4 [I]

(d) From your graph, is ( I , l ) a solution toy= - 3x + 4? Give an explanation for your [2]
answer.

(T urn O ver
CdiMS EOY Exam 2011 Sec I Maths Paper l
8

mJ::; ,.·,_

~·· .... . . . ·:. ·..~


.

fTurn Over
CCHMS EOY Exam 2011 Sec I Maths Paper 2

Das könnte Ihnen auch gefallen